+ All Categories
Home > Documents > JOIN THE DOTS! - Career Launcher · dots! A current affairs series for UPSC Examination’ series...

JOIN THE DOTS! - Career Launcher · dots! A current affairs series for UPSC Examination’ series...

Date post: 12-Apr-2020
Category:
Upload: others
View: 0 times
Download: 0 times
Share this document with a friend
115
Dear Students, With the present examination pattern of UPSC Civil Services Examination, General Studies papers require a lot of specialization with ‘Current Affairs’. Moreover, following the recent trend of UPSC, almost all the questions are based on news as well as issues. CL IAS has now come up with ‘Join the dots! A current affairs series for UPSC Examination’ series which will help you pick up relevant news items of the day from various national dailies such as The Hindu, Indian Express, Business Standard, LiveMint, PIB and other important sources. ‘Join the dots! A current affairs series for UPSC Examination’ series will be helpful for prelims as well as Mains Examination. We are covering every issue in a holistic manner and covered every dimension with detailed facts. This edition covers all important issues that were in news in the month of March 2019. Also, we have introduced Prelim base question for Test Your Knowledge which shall guide you for better revision. In addition, it would benefit all those who are preparing for other competitive examinations. We have prepared this series of documents after some rigorous deliberations with Toppers and also with aspirants who have wide experience of preparations in the Civil Services Examination. For more information and more knowledge, you can go to our website https://www.careerlauncher. com/upsc/ “Set your goals high, and don’t stop till you get there” All the best!! Team CL JOIN THE DOTS! A CURRENT AFFAIRS SERIES FOR UPSC EXAMINATION Compendium – March 2019
Transcript
Page 1: JOIN THE DOTS! - Career Launcher · dots! A current affairs series for UPSC Examination’ series which will help you pick up relevant news items of the day from various national

Dear Students,

With the present examination pattern of UPSC Civil Services Examination, General Studies papers require a lot of specialization with ‘Current Affairs’. Moreover, following the recent trend of UPSC, almost all the questions are based on news as well as issues. CL IAS has now come up with ‘Join the dots! A current affairs series for UPSC Examination’ series which will help you pick up relevant news items of the day from various national dailies such as The Hindu, Indian Express, Business Standard, LiveMint, PIB and other important sources.

‘Join the dots! A current affairs series for UPSC Examination’ series will be helpful for prelims as well as Mains Examination. We are covering every issue in a holistic manner and covered every dimension with detailed facts. This edition covers all important issues that were in news in the month of March 2019. Also, we have introduced Prelim base question for Test Your Knowledge which shall guide you for better revision. In addition, it would benefit all those who are preparing for other competitive examinations.

We have prepared this series of documents after some rigorous deliberations with Toppers and also with aspirants who have wide experience of preparations in the Civil Services Examination.

For more information and more knowledge, you can go to our website https://www.careerlauncher.com/upsc/

“Set your goals high, and don’t stop till you get there”

All the best!!

Team CL

JOIN THE DOTS!A CURRENT AFFAIRS SERIES FOR UPSC EXAMINATION

Compendium – March 2019

Page 2: JOIN THE DOTS! - Career Launcher · dots! A current affairs series for UPSC Examination’ series which will help you pick up relevant news items of the day from various national
Page 3: JOIN THE DOTS! - Career Launcher · dots! A current affairs series for UPSC Examination’ series which will help you pick up relevant news items of the day from various national

Contents1. 20th Session of India-Italy JCEC ................................................................................... 12. Union HRD Minister launches the Scheme for Higher Education Youth in

Apprenticeship and Skills (SHREYAS) ......................................................................... 23. National Science Day: HRD Ministry launched Rs 250 crore scheme

‘STARS’ to fund science projects .................................................................................... 24. ISRO Special Mission For DRDO .................................................................................. 35. Cabinet approves Scheme for FAME India Phase II .................................................... 46. TECH-SOP 2019 for MSMEs ......................................................................................... 47. MainamatiMaitree Exercise 2019 ................................................................................. 58. 7th RCEP Inter-Sessional Ministerial Meeting ............................................................. 69. Construction Technology India -2019 ........................................................................... 610. Setting up of Centre for Disability Sports at Gwalior .................................................. 711. Government to celebrate ‘PoshanPakhwada’ across the Country .............................. 712. Cabinet approves setting up of a Railway Zone at Vishakhapatnam ......................... 813. Cabinet approves”PradhanMantriJl-VAN yojana” ..................................................... 914. Arun-3 Hydro Electric Project (Nepal portion) ........................................................... 1015. Indo-Pacific Regional Dialogue ................................................................................... 1116. EASE Reforms for Public Sector Banks ...................................................................... 1117. Construction Technology India-2019 Expo-cum-conference ...................................... 1218. National Policy on Software Products – 2019 ............................................................. 1219. Crop burning raises risk of respiratory illness threefold, says IFPRI study ............ 1320. India to tie-up with 4 nations to save rhinos .............................................................. 1421. World Wildlife Day 2019 ............................................................................................. 1522. PSBloansin59minutes.com .......................................................................................... 1523. One Nation One Card ................................................................................................... 1624. US to withdraw duty benefits on $5.6 billion exports from India:

Generalized System of Preferences (GSP) .................................................................. 1725. QS World Ranking: Five Indian institutes in top 300,

IIT Bombay leads India chart ...................................................................................... 1826. National Rural Economic Transformation Project (NRETP)

to boost Rural Incomes across 13 States in India ...................................................... 1927. AL NAGAH 2019 .......................................................................................................... 2028. Prime Minister Launches PM-SYM in Ahmedabad ................................................... 2029. Smart Fencing on Indo-Bangladesh border ................................................................ 2130. Textiles Minister inaugurates Refurbished Handloom Haat .................................... 2231. Independent Verification of Swachh Bharat Grameen .............................................. 2332. Kudo International Federation India as National Sport Federation ........................ 2433. 7th March 2019 to be celebrated as ‘JanaushadhiDiwas’ across India ...................... 2434. Amendment in Hazardous Waste

(Management & Transboundary Movement) Rules, 2016 ......................................... 2535. ‘Web Wonder Women’ .................................................................................................. 2536. Release of Dictionary of Martyrs of India’s Freedom Struggle (1857-1947) ............. 2637. Integrated Command and

Control Centres in Sikkim, Arunachal Pradesh and Tripura .................................... 2738. Cabinet approves Continuation of Atal Innovation Mission ...................................... 28

Page 4: JOIN THE DOTS! - Career Launcher · dots! A current affairs series for UPSC Examination’ series which will help you pick up relevant news items of the day from various national

39. National Council of Science Museums of Culture Ministry collaborates with Google Arts & Culture for largest interactive online exhibition on inventions and discoveries .................................................................... 28

40. Cabinet approves Kiru Hydro Electric (HE) Project (4 X 156 MW) in Jammu and Kashmir ............................................................................................... 29

41. Indore gets cleanest city tag for third year in a row .................................................. 2942. ISRO, French agency seal agreement on maritime security...................................... 3043. National Mission on Transformative Mobility and

Battery Storage ............................................................................................................ 3044. Advanced Braking Systems Made Mandatary for Vehicles ....................................... 3145. Government approves Strategic Disinvestment of 100% GOI

shares in Dredging Corporation .................................................................................. 3246. The Loan Agreement for Additional Financing of

$137 Million for the Dam Rehabilitation and Improvement Project (DRIP) ........... 3247. India Cooling Action Plan Launched ........................................................................... 3348. India signs $3 billion contract with Russia for lease of a nuclear submarine .......... 3449. SC appoints 3-member panel to mediate in Ayodhya dispute: .................................. 3550. India Bags First Prize at The International

‘Golden City Gate Tourism Awards 2019’ ................................................................... 3651. Only 3.32 lakh MSME jobs created in last four years, finds CII survey ................... 3752. The Seven Crore LPG connection under the

Pradhan Mantri Ujjwala Yojana (PMUY) ................................................................... 3753. Explicitly demarcate trees grown in forests from those grown outside .................... 3854. Twin Success for Guided PINAKA .............................................................................. 3955. Idukki’s MarayoorJaggery gets GI tag ........................................................................ 4056. 14thCII-EXIM Bank Conclave on India-Africa Project Partnerships ...................... 4157. Vice President concludes his visit to Paraguay and Costa Rica ............................... 4158. Pulse Polio Programme 2019 ....................................................................................... 4259. Arecanut gets ‘Sirsi supari’ geographical indication tag ............................................ 4360. Wood snake, last seen in 1878, rediscovered by scientists ......................................... 4461. SC may send plea challenging quota Bill to Constitution Bench .............................. 4462. India is world’s 2nd largest arms importer .................................................................. 4663. The DRDO Develops ‘combat drugs’ to reduce casualties in

Pulwama type attacks, warfare ................................................................................... 4764. PM Modi, Bangladesh PM Sheikh Hasina, jointly unveil

e-plaques for development projects in Bangladesh .................................................... 4865. INDO-OMAN Joint Ex Al Nagah 2019 ....................................................................... 4866. Cabinet approves proposal for accession of India to

The Nice Agreement The Vienna agreement and The Locarno Agreement ............ 4967. No names removed based on draft NRC: Election Commission ................................ 4968. Data for the Index of Industrial Production (IIP) ....................................................... 5069. Retail inflation rises to 4-month high of 2.57% in February 2019 ............................ 5170. World Gold Council Report .......................................................................................... 5171. Twin Success for Man Portable Anti Tank Guided Missile ....................................... 5272. Indo-Bangladesh joint Military Exercise Sampriti – 2019 ........................................ 5373. First Workshop on India Energy Modelling Forum Held .......................................... 5374. West Nile Virus ............................................................................................................ 5475. China places hold on listing Azhar as designated terrorist ....................................... 54

Page 5: JOIN THE DOTS! - Career Launcher · dots! A current affairs series for UPSC Examination’ series which will help you pick up relevant news items of the day from various national

76. Festival of Innovation & Entrepreneurship (FINE) ................................................... 5577. A climate vulnerability index for India on the anvil .................................................. 5678. India-Africa joint field training exercise to kick off on March 18:

AFINDEX -2019 ........................................................................................................... 5779. Global Environment Outlook (GEO) ........................................................................... 5780. Young Global Leaders 2019: WEF recognises Nara Lokesh as

Young Global Leader for 2019 ..................................................................................... 5881. ISRO set for next PSLV-C45 launch ........................................................................... 5982. World Consumer Rights Day ....................................................................................... 6083. Erode turmeric gets GI tag after an 8-year process ................................................... 6184. Fourth United Nations Environment Assembly ......................................................... 6185. Only 26% of rural toilets use twin-leach pits .............................................................. 6286. Contempt of Courts act 1971 ....................................................................................... 6387. New hydro policy to help meet renewables target ...................................................... 6488. A carbon-neutral project in Kerala ............................................................................. 6589. Pinaki Chandra Ghose set to be India’s first Lokpal ................................................. 6690. Domestic Systemically Important Banks (D-SIBs) .................................................... 6691. 3rd Indo-Japan Workshop on Disaster Risk Reduction held in New Delhi ............... 6792. Theatre Level Readiness and Operational Exercise (TROPEX) ................................ 6893. Tropical Cyclone Idai.................................................................................................... 6994. Pakistan, Bhutan, Sri Lanka and Bangladesh ranked higher than India. ............... 7095. India-Indonesia Coordinated Patrol (Ind-Indo Corpat) Commences ......................... 7196. Indo-Sri Lanka joint Exercise Mitrashakti-VI ........................................................... 7297. 34th meeting of the GST Council ................................................................................ 7298. Census of otters ............................................................................................................ 7399. Bomb cyclone ................................................................................................................ 74100. Indian Navy - first Responder to Cyclone ‘IDAI’ in Mozambique .............................. 74101. World Water Day ......................................................................................................... 75102. Indian Air Force to participate in Langkawi

International Maritime Aero Expo (LIMA) 2019 ........................................................ 75103. Navroz Festival ............................................................................................................ 76104. World Down Syndrome Day ......................................................................................... 76105. International Day of Forests 2019 ............................................................................... 77106. Evidence of water, particle plumes discovered on asteroid Bennu: NASA ............... 78107. NITI Aayog to organise FinTech Conclave 2019 ........................................................ 79108. Indian Forest Act, 1927 proposed amendments ......................................................... 80109. Trump says US will recognize Israel’s sovereignty over Golan Heights ................... 81110. The International Day for the Elimination of Racial Discrimination 2019 .............. 81111. Unlawful Activities Prevention Act (UAPA) ............................................................... 82112. Justice Pinaki Chandra Ghose takes oath as first Lokpal chief of India .................. 82113. Young Scientist Programme (Yuvika) ........................................................................ 83114. Induction of Chinook Helicopters in Indian Air Force ............................................... 84115. Japan gives UN World Food Programme $69m ......................................................... 85116. World Meteorological Day 2019 .................................................................................. 85117. World Tuberculosis Day 2019 ...................................................................................... 86118. PSLV-C45 project will mark several firsts for ISRO:

Emisat mission scheduled for launch on April 1 ........................................................ 87

Page 6: JOIN THE DOTS! - Career Launcher · dots! A current affairs series for UPSC Examination’ series which will help you pick up relevant news items of the day from various national

119. The Global Energy Transition index 2019 .................................................................. 88120. AUSINDEX: Australia, India’s Naval Exercise To Focus On

Anti-Submarine Warfare ............................................................................................. 90121. Pakistan Approves Plan to Open Sharda Temple Corridor ....................................... 90122. ‘Mission Shakti’, India’s homegrown anti-satellite missile. ....................................... 91123. India’s carbon dioxide emissions up 5 ......................................................................... 93124. Govt. notifies new rules for drugs, clinical trials ........................................................ 94125. Opening Ceremony Indo-Sri Lanka Joint Exercise Mitra Shakti-VI ........................ 94126. Cabinet approves five year’s extension of

Biomedical Research Career Programme ................................................................... 95127. Electoral bonds will affect transparency, EC tells SC ................................................ 95128. In 2018, Official Secrets Act invoked in five cases ..................................................... 97129. Assam launches initiative to educate voters ............................................................... 98130. India in pact to ease U.S. firms’ compliance ............................................................... 98131. National Company Law Appellate Tribunal (NCLAT) .............................................. 99132. Global Multidimensional Poverty Index 2018 report ................................................. 99133. In India, poverty reduction among children, the poorest states,

Scheduled Tribes, and Muslims was fastest ............................................................... 99134. Dhanush Howitzer guns ............................................................................................ 100135. President Kovind honoured with Croatia’s highest civilian order .......................... 101136. Coffee Board Activates Blockchain Based Marketplace in India ............................ 102137. GI Certification for five varieties of Indian coffee .................................................... 102138. Prelims Practice Questions ........................................................................................ 104 Answer Key ................................................................................................................. 109

Page 7: JOIN THE DOTS! - Career Launcher · dots! A current affairs series for UPSC Examination’ series which will help you pick up relevant news items of the day from various national

Page: 1 Join the dots! – March 2019

Join the dots! A current affairs series for UPSC Examination

20th Session of India-Italy JCEC

Relevance IN – Prelims (about the JCEC outcome) + Mains (GS II bilateral relations

What’s the NEWS

• The 20th Session of India-Italy Joint Commission for Economic Cooperation (JCEC) was held on 26-27 February,

2019 in New Delhi.

Know! all about the JCEC

• The JCEC is an institutional mechanism for bilateral trade engagement. the importance of Italy as a trade

partner for India

• Italy is India’s 5th largest trading partner in the European Union and 25th in the World during 2017-18.

• There is an immense potential for Italian and Indian industries to have partnership and cooperation.

• Bilateral trade during the recent years has been increasing despite global slowdown.

• Italy ranks 17th in FDI inflow in India during April, 2000 to December, 2018.

• Suresh Prabhu thanked the Italian side for submitting the evaluation report on Tricyclazole (TCA) in rice to

European Food Safety Authority (EFSA) and removing the alert on Indian Incense Sticks from RAPEX portal.

• The Indian Commerce Minister reiterated that India is committed to an early and balanced outcome of India-

EU BTIA (Broad-based Bilateral Trade and Investment Agreement) negotiations.

• The two Sides agreed to hold 21th Meeting of the Joint Commission in Italy in 2021.

Know! about RAPEX

• The European Union (EU) has introduced a rapid alert system (RAPEX) for products which pose a serious risk

to consumers.

• Data is rapidly exchanged and published on measures taken to prevent or restrict the marketing or use of

non-food, non-pharmaceutical or non-medical products or devices, which pose a serious risk to different public

interests.

Know! about India-EU Broad Based Trade and Investment Agreement (BTIA) negotiations

• On 28th June 2007, India and the EU began negotiations on a broad-based Bilateral Trade and Investment

Agreement (BTIA) in Brussels, Belgium.

• These negotiations are pursuant to the commitment made by political leaders at the 7th India-EU Summit

held in Helsinki on 13th October 2006 to move towards negotiations for a broad-based trade and investment

agreement on the basis of the report of India-EU High Level Technical Group.

• India and the EU expect to promote bilateral trade by removing barriers to trade in goods and services and

investment across all sectors of the economy.

• Both parties believe that a comprehensive and ambitious agreement that is consistent with WTO rules and

principles would open new markets and would expand opportunities for Indian and EU businesses.

Page 8: JOIN THE DOTS! - Career Launcher · dots! A current affairs series for UPSC Examination’ series which will help you pick up relevant news items of the day from various national

Join the dots! – March 2019 Page: 2

Union HRD Minister launches the Scheme for Higher Education Youth in Apprenticeship and Skills (SHREYAS) for providing industry apprenticeship opportunities: SHREYAS will help the youth to obtain gainful employment and contribute to country’s progress

Relevance IN – Prelims (about SHREYAS) + Mains (GS II issues relating to development and management of social sectors relating to education and Human resource

What’s the NEWS

• The Minister for Human Resources Development, Shri Prakash Javadekarhas launched the Scheme for Higher Education Youth in Apprenticeship and Skills (SHREYAS) for providing industry apprenticeship opportunities to the general graduates exiting in April 2019 through the National Apprenticeship Promotional Scheme (NAPS).

Know! more about the program

• The program aims to enhance the employability of Indian youth by providing ‘on the job work exposure’ and earning of stipend.

• The education with skills is the need of the hour and the SHREYAS will be a major effort in this direction to make our degree students more skilled, capable, employable and aligned to the needs of our economy so that they contribute to country’s progress and also obtain gainful employment.

• SHREYAS is a programme conceived for students in degree courses, primarily non-technical, with a view to introduce employable skills into their learning, promote apprenticeship as integral to education and also amalgamate employment facilitating efforts of the Government into the education system so that clear pathways towards employment opportunities are available to students during and after their graduation.

• SHREYAS is a programme basket comprising the initiatives of three Central Ministries, namely the Ministry of Human Resource Development, Ministry of Skill Development & Entrepreneurship and the Ministry of Labour& Employment viz the National Apprenticeship Promotion Scheme (NAPS), the National Career Service (NCS)and introduction of BA/BSc/BCom (Professional) courses in the higher educational institutions.

• SHREYAS portal will enable educational institutions and industry to log in and provide their respective demand and supply of apprenticeship.

• The matching of students with apprenticeship avenues will take place as per pre-specified eligibility criteria.

National Science Day: HRD Ministry launched Rs 250 crore scheme ‘STARS’ to fund science projects

Scheme for Translational & Advanced Research in Science #STARS

What’s the NEWS

• On the National Science day, the Union Human Resource Development (HRD) Minister, Prakash Javadekar launched STARS – Scheme for Translational and Advanced Research in Science to fund science projects.

• The ministry has approved funds worth Rs 250 crore for the scheme.

• These funds will be utilised to sponsor about 500 science projects. The selection of these projects would be done on the basis of competitions.

• The project will be coordinated by the Indian Institute of Science (IISc), Bangalore. Interested candidates will have to submit applications to take part in the competition. The applications will release on the first week of April 2019, informed HRD Minister in a tweet.

Page 9: JOIN THE DOTS! - Career Launcher · dots! A current affairs series for UPSC Examination’ series which will help you pick up relevant news items of the day from various national

Page: 3 Join the dots! – March 2019

Other initiatives by HRD

• The HRD Ministry in collaboration with the National Skill Ministry and Labour and employment Ministry collaboratively launched SHREYAS – Scheme for Higher Education Youth for Apprenticeship and Skills to empower non-technical courses.

• It announced to launch professional courses which would include 1000 hours of additional coursework out of which 250 will be dedicated to soft and ICT skilling (each) and rest 500 hours will be dedicated to specific skill courses.

• These courses will be open from this (2019-20) academic session onwards.

Know! about National Science Day

• It is celebrated every year on February 28 in the memory of Sir C.V. Raman. He was an Indian physicist and a Nobel laureate.

• He received the Nobel in 1930 for discovering of the ‘Raman effect’ which refers to the ‘scattering’ of light that occurs when it passes through molecules, say dust in the air.

ISRO Special Mission For DRDO: PSLV To Launch Electronic Intelligence Satellite ‘Emisat’ along with 28 Others

Relevance IN – Prelims (about Emisat) + Mains (GS III science and technology – awareness in the field of space)

What’s the NEWS

• The Indian Space Research Organisation will be conducting a ‘special mission’ in March this year (2019), sending into space an electronic intelligence satellite ‘Emisat’ for the Defence Research and Development Organisation (DRDO), along with 28 third party satellites.

• It will also be attempting to release its payload in the orbit at three different altitudes.

Know! more about the upcoming mission

• ISRO will be using a PSLV rocket with four strap-on motors and for the first time ISRO will be trying to orbit the rocket at three different altitudes

• The main passenger for the PSLV rocket will be the defence intelligence satellite Emisat belonging to Defence Research and Development Organisation (DRDO). The DRDO’s Emisat is an electronic intelligence satellite

• The ISRO will also be launching two more defence satellites sometime in July/August with its new rocket Small Satellite Launch Vehicle (SSLV).

• In January, the space agency launched a defence imaging satellite Microsat R for the DRDO.

• After launching Emisat at an altitude of 763 km, the rocket will be brought down to put into orbit the 28 satellites at an altitude of 504 km.

• Following that the rocket will be brought down further to 485 km where the fourth stage will turn into a payload platform carrying three experimental payloads -- one developed by the students of Indian Institute of Space Science and Technology, besides ISRO’s own technology demonstrator and a Hamsat.”

• The PSLV is a four-stage engine expendable rocket with alternating solid and liquid fuel. In its normal configuration, the rocket will have six strap-on motors hugging the rocket’s first stage.

• On January 24, the ISRO flew a PSLV with two strap-on motors while in March it will have four strap-on motors.

• The Indian space agency also has two more PSLV variants viz Core Alone (without any strap-on motors) and PSLV-XL a larger rocket.

• The ISRO selects the kind of rocket to be used based on the weight of satellites it carries.

Page 10: JOIN THE DOTS! - Career Launcher · dots! A current affairs series for UPSC Examination’ series which will help you pick up relevant news items of the day from various national

Join the dots! – March 2019 Page: 4

Cabinet approves Scheme for FAME India Phase II

Relevance IN – Prelims (about FAME) + Mains (GS III infrastructure development + energy conservation)

What’s the NEWS• The Union cabinet has approved the proposal for implementation of scheme titled ‘Faster Adoption and

Manufacturing of Electric Vehicles in India Phase II (FAME India Phase II)’ for promotion of Electric Mobility in the country.

• The scheme with total outlay of Rs 10000 Crores over the period of three years will be implemented with effect from 1st April 2019.

• This scheme is the expanded version of the present scheme titled ‘FAME India1 which was launched on 1st April 2015, with total outlay of Rs. 895 crores.

Know! the objective of FAME• The main objective of the scheme is to encourage Faster adoption of Electric and hybrid vehicle by way of

offering upfront Incentive on purchase of Electric vehicles and also by way of establishing a necessary charging Infrastructure for electric vehicles.

• The scheme will help in addressing the issue of environmental pollution and fuel security.

Know! more about the scheme• To encourage advance technologies, the benefits of incentives, will be extended to only those vehicles which are

fitted with advance battery like a Lithium Ion battery and other new technology batteries.

• The scheme proposes for establishment of charging infrastructure, whereby about 2700 charging stations will be established in metros, other million plus cities, smart cities and cities of Hilly states across the country so that there will be availability of at least one charging station in a grid of 3 km x 3 km.

• Establishment of Charging stations are also proposed on major highways connecting major city clusters. On such highways, charging stations will be established on both sides of the road at an interval of about 25 km each.

TECH-SOP 2019 for MSMEs

Page 11: JOIN THE DOTS! - Career Launcher · dots! A current affairs series for UPSC Examination’ series which will help you pick up relevant news items of the day from various national

Page: 5 Join the dots! – March 2019

Relevance IN – Prelims (about TECH -SOP)

What’s the NEWS

• With a view to enhance awareness among MSMEs about latest technological innovations available, and sensitize them on the role of technology in creating competitiveness and opportunities, the Ministry of Micro, Small and Medium Enterprises (MSME)organized a programme on Technology Support and Outreach (TECH-SOP 2019)

Know! about the event

• Various research and development institutions in the country have developed technologies which are relevant for MSMEs for sustainable growth and is available to MSMEs in a cost effective manner.

• TECH – SOP 2019 is an initiative to bridge the gap between the technological innovations and MSMEs so that they can harness the technologies and grow in the global value chain.

• ‘India Green Tech Open Challenge was also launched during this occasion which aims to encourage MSMEs to adopt sustainable and green technologies.

Mainamati Maitree Exercise 2019

India, Bangladesh hold joint exercise to boost confidence on Tripura border Relevance IN – Prelims (about the exercise)

What’s the NEWS

• Border Security Force (BSF) and Border Guards Bangladesh (BGB) concluded a three-day Mainamati Maitree Exercise 2019 as a part of confidence-building measures between the two border guards.

Know! all about Mainamati Maitree Exercise 2019

• The exercise was named after Mainamati hill range situated 8 km west of Comilla Township in Bangladesh, which is home to an ancient Buddhist archaeological site in the region.

• The main objective of this exercise was to plan and conduct anti-smuggling and anti-criminal activity related operations with ultimate aim of achieving better joint operational efficiency and border management in the area of responsibility of respective BSF and BGB Battalions participating in the joint exercise

• A series of joint exercises and exchanges are being held in the Tripura-Bangla frontier since a four-day standoff between the two forces centring 31 Rohingya people who were found crossing the international border into Bangladesh in January this year.

Page 12: JOIN THE DOTS! - Career Launcher · dots! A current affairs series for UPSC Examination’ series which will help you pick up relevant news items of the day from various national

Join the dots! – March 2019 Page: 6

7th RCEP Inter-Sessional Ministerial Meeting

Relevance IN – Prelims (about RCEP) + Mains (GS II international organisations)

What’s the NEWS

• Union Minister of Commerce & Industry and Civil Aviation, Suresh Prabhu, has reached Siem Reap in Cambodia for the Seventh Regional Comprehensive Economic Partnership Inter-sessional Ministerial Meeting (7th RCEP ISSL MM)

• In this meeting, developments since the second RCEP summit held on 14th November 2018 in Singapore will be reviewed

• During the term of the Singapore chair, several ministerial and experts’ meetings resulted in successful conclusion of five chapters taking total number of chapters concluded so far to 7 out of 16.

• At the RCEP Trade Ministers’ Meeting on 12-13 November 2018 Singapore deftly steered the consensus on ‘substantial progress’ instead of ‘substantial conclusion’.

Know! about RCEP

• The Regional Comprehensive Partnership (RCEP) is a proposed Free Trade Agreement (FTA) between sixteen countries namely 10 countries of ASEAN (Brunei, Cambodia, Indonesia, Lao, Malaysia, Myanmar, Philippines, Singapore, Thailand, and Vietnam) and their six FTA partners (also known as AFP’s or ASEAN FTA Partners) namely Australia, China, India, Japan, Korea and New Zealand.

• The RCEP has entered the sixth year of negotiations. During the last five years 24 Rounds of negotiations at the expert level were held – the last Round was held from 18-27 October, 2018 in Auckland; six Ministerial meetings and seven inter-sessional Ministerial meetings were held so far - the 6th Ministerial meeting held on 30-31 August, 2018 in Singapore and the 7thinter-sessional Ministerial meeting held on 12-13 November, 2018 in Singapore; and two RCEP Leaders’ Summit swere held.

• During the 2nd RCEP Summit on 14 November 2018 in Singapore, Leaders acknowledged the ‘substantial progress’ in the negotiations and reiterated their determination to conclude a modern, comprehensive, high quality, and mutually beneficial RCEP in 2019.

Construction Technology India -2019

Relevance IN – Prelims(about the conference) + Mains (GS III infrastructure development + Sustainable development + GS II welfare scheme for the vulnerable section of population)

What’s the NEWS

• Prime Minister will inaugurate the Construction Technology India-2019 Expo-cum-Conference in New Delhi on 2nd March, 2019.

Global Housing Technology Challenge - India

• The Ministry of Housing and Urban Affairs, Government of India has conceptualized a Global Housing Technology Challenge - India (GHTC- India) which aims to identify and mainstream a basket of innovative technologies from across the globe that are sustainable and disaster-resilient.Pradhan Mantri Awas Yojana - Urban (PMAY-U) targets the construction of 1 Crore housing units by 2022, out of which 79 lakh houses have been sanctioned by MoHUA.

• More than 41 lakh houses have already been grounded and 16 lakh houses are completed and handed over to beneficiaries.

• The GHTC-India platform aspires to provide an eco-system for the adoption of innovative technologies in the housing construction sector in a holistic manner.

Construction Technology India – 2019

• Construction Technology India – 2019” an Expo –cum –Conference is being organised by the Ministry of Housing & Urban Affairs - to identify proven, innovative and globally established technologies for use in the Indian context.

Page 13: JOIN THE DOTS! - Career Launcher · dots! A current affairs series for UPSC Examination’ series which will help you pick up relevant news items of the day from various national

Page: 7 Join the dots! – March 2019

• Technology providers, researchers, start-ups, developers, academia, public sector agencies and other domain experts are participating in the 2-day event.

• CTI-2019, an expo-cum-Conference will provide avenues for interaction among stakeholders that can open up new areas for collaboration

• Potential technology providers can benefit from the support provided by Affordable Sustainable Housing Accelerators- India (ASHA-India) initiative.

• GHTC-India will also contribute substantially towards achieving the Sustainable Development Goals (SDGs) as laid out by the United Nations (UN), the New Urban Agenda and the Paris Climate Accord to which India is a signatory

• GHTC-India will bring about a paradigm shift in the manner in which construction is not only done but also received and perceived in the country.

Setting up of Centre for Disability Sports at Gwalior in MadhyaWhat’s the NEWS• The Government has approved the proposal regarding setting up of a Centre for Disability Sports at Gwalior in

Madhya Pradesh, to be registered under the Societies Registration Act, 1860, which is to function under the name of Centre for Disability Sports, Gwalior.

Know! more about the centre• The facilities so developed would be multi-functional centres with provision for training, selection, sports

academics and research, medical support, spectator galleries and suitable for holding national/international events.

• Taking 1st April, 2019 as the zero date, the project is expected to be completed in two years time, i.e, by 31st March, 2021. Thereafter, it will take another 6 months for the Centre to become operational.

• Once operational, this Centre will result in employment of 140 personnel. In addition, construction and allied activities will also generate considerable employment opportunities.

• Improved sports infrastructure created by this Centre will ensure effective participation of persons with disabilities in sports activities and also enable them to compete at national and international levels. Setting up of the Centre will develop a sense of belonging in Divyangjan to facilitate their integration in society.

Government to celebrate ‘Poshan Pakhwada’ across the Country

Relevance IN – Prelims (about POSHAN scheme) + Mains (GS II Social issues, awareness in the field of social services relating to health sector + government policies interventions)

Page 14: JOIN THE DOTS! - Career Launcher · dots! A current affairs series for UPSC Examination’ series which will help you pick up relevant news items of the day from various national

Join the dots! – March 2019 Page: 8

What’s the NEWS• The Minister for Women and Child Development, Smt. Maneka Sanjay Gandhi today announced that the

Government will celebrate the first anniversary of POSHAN Abhiyaan on 8th March, 2019 by organising Poshan Pakhwada across the Country.

• The Poshan Pakhwada will be celebrated on lines of ‘Poshan Maah’ held in the month of September, 2018.

• The Pakhwada will be launched on International Women’s Day, 8th March, 2019.

Know! all about POSHAN Abhiyaan• POSHAN Abhiyaan is a multi-ministerial convergence mission with the vision to ensure attainment of

malnutrition free India by 2022.

• The objective of POSHAN Abhiyaan to reduce stunting in identified Districts of India with the highest malnutrition burden by improving utilization of key Anganwadi Services and improving the quality of Anganwadi Services delivery.

• Its aim to ensure holistic development and adequate nutrition for pregnant women, mothers and children.

• The Ministry of Women and Child Development (MWCD) is implementing POSHAN Abhiyaan in 315 Districts in first year, 235 Districts in second year and remaining districts will be covered in the third year.

• There are a number of schemes directly/indirectly affecting the nutritional status of children (0-6 year’s age) and pregnant women and lactating mothers.

• In spite of these, level of malnutrition and related problems in the country is high. There is no dearth of schemes but lack of creating synergy and linking the schemes with each other to achieve common goal.

• POSHAN Abhiyaan through robust convergence mechanism and other components would strive to create the synergy.

The POSHAN Abhiyaan, as an apex body, will monitor, supervise, fix targets and guide the nutrition related interventions across the Ministries.

The proposal consists of• mapping of various Schemes contributing towards addressing malnutrition

• introducing a very robust convergence mechanism

• ICT based Real Time Monitoring system

• incentivizing States/UTs for meeting the targets

• incentivizing Anganwadi Workers (AWWs) for using IT based tools

• eliminating registers used by AWWs

• introducing measurement of height of children at the Anganwadi Centres (AWCs)

• Social Audits

• Setting-up Nutrition Resource Centres, involving masses through Jan Andolan for their participation on nutrition through various activities, among others.

• Implementation strategy would be based on intense monitoring and Convergence Action Plan right up to the grass root level.

• POSHAN Abhiyaan will be rolled out in three phases from 2017-18 to 2019-20.

• POSHAN Abhiyaan targets to reduce stunting, under-nutrition, anaemia (among young children, women and adolescent girls) and reduce low birth weight by 2%, 2%, 3% and 2% per annum respectively.

• Although the target to reduce Stunting is at least 2% p.a., Mission would strive to achieve reduction in Stunting from 38.4% (NFHS-4) to 25% by 2022 (Mission 25 by 2022).

Cabinet approves setting up of a Railway Zone at Vishakhapatnam

Relevance IN – Prelims 9 about new zone) + Mains (GS III infrastructure development)

What’s the NEWS• The Union Cabinet, has approved setting up of a Railway Zone at Vishakhapatnam and a new division with

headquarter at Rayagada by reorganizing the existing South Central Railway and East Coast Railway.

Page 15: JOIN THE DOTS! - Career Launcher · dots! A current affairs series for UPSC Examination’ series which will help you pick up relevant news items of the day from various national

Page: 9 Join the dots! – March 2019

Southern Coast Railway zone• The Southern Coast Railway zone would comprise of existing Guntakal, Guntur and Vijayawada divisions

that currently fall under the South Central Railway and the South Central Railway will then consist of the Hyderabad, Secunderabad and Nanded divisions.

• Waltair division under the East Coast Railway Zone, covering the northern districts of Andhra Pradesh, parts of Chhattisgarh and Odisha will be split into two.

• One part will be incorporated in the new zone and merged with the neighbouring Vijayawada division, the remaining portion will be converted into a new division, headquartered at Rayagada, under the East Coast Railway (ECoR)

• Setting up of the new Railway Zone will bring optimality with regard to size and scale of railway operations and also to meet the persistent demand and aspirations of the people of the area.

Cabinet approves”Pradhan Mantri Jl-VAN yojana”

Relevance In – Prelims 9 about PM JI- VAN) + Mains (GS III energy conservation, awareness in the field of bio technology)

What’s the NEWS• The Cabinet Committee on Economic Affairs has approved the “Pradhan Mantri JI-VAN (Jaiv Indhan- Vatavaran

Anukool fasal awashesh Nivaran) Yojana” for providing financial support to Integrated Bioethanol Projects using lignocellulosic biomass and other renewable feedstock.

Know! all about PM JI -VAN• Under this Yojana, 12 Commercial Scale and 10 demonstration scale Second Generation (2G) ethanol Projects

will be provided a Viability Gap Funding (VGF) support in two phases:

- Phase-I (2018-19 to 2022-23): wherein six commercial projects and five demonstration projects will be support

- Phase-II (2020-21 to 2023-24): wherein remaining six commercial projects and five demonstration projects will be supported.

• The scheme focuses to incentivise 2G Ethanol sector and support this nascent industry by creating a suitable ecosystem for setting up commercial projects and increasing Research & Development in this area.

• The ethanol produced by the scheme beneficiaries will be mandatorily supplied to Oil Marketing Companies (OMCs) to further enhance the blending percentage under EBP Programme.

• Ministry of Petroleum & Natural Gas has targeted to achieve 10% blending percentage of Ethanol in petrol by 2022.

• Despite efforts of the Government such as higher ethanol prices and simplification of ethanol purchase system, the highest ever ethanol procurement stands around 150 crore litres during Ethanol supply year 2017-18 which is sufficient for around 4.22% blending on Pan India basis.

• Therefore, an alternate route viz. Second Generation (2G) Ethanol from biomass and other wastes is being explored by MoP&NG to bridge the supply gap for EBP programme. In this direction, “Pradhan Mantri JI-VAN Yojana” is being launched as a tool to create 2G Ethanol capacity in the country and attract investments in this new sector.

• Centre for High Technology (CHT), a technical body under the aegis of MoP&NG, will be the implementation Agency for the scheme.

Know! about the EBP Programme• Government of India launched Ethanol Blended Petrol (EBP) programme in 2003 for undertaking blending

of ethanol in Petrol to address environmental concerns due to fossil fuel burning, provide remuneration to farmers, subsidize crude imports and achieve forex savings.

• Presently, EBP is being run in 21 States and 4 UTs of the country.

• Under EBP programme, OMCs are to blend upto 10% of ethanol in Petrol. The present policy allows procurement of ethanol produced from molasses and non-food feed stock like celluloses and lignocelluloses material including petrochemical route.

Page 16: JOIN THE DOTS! - Career Launcher · dots! A current affairs series for UPSC Examination’ series which will help you pick up relevant news items of the day from various national

Join the dots! – March 2019 Page: 10

Arun-3 Hydro Electric Project (Nepal portion)

Relevance IN – Prelims (about the hydro project and Arun river) + Mains (GS III infrastructure development + GS II bilateral relations)

What’s the NEWS• The Cabinet Committee on Economic Affairs, has approved investment for Transmission Component of Arun-3

Hydro Electric Project(Nepal portion)

• The current approval is for Investment approval of 400 kV D/C Diding (In Nepal) - Bathnaha (International Border) via Dhalkebar (In Nepal) Transmission Line of approximately 217 KM falling within the territory of Nepal to evacuate power from Arun-3 HEP in Nepal.

Benefits • Employment generation of around 400 persons is envisaged in construction of transmission component of the

project.

• The project will provide surplus power to India strengthening economic linkages with Nepal. The power from the project shall be exported from Dhalkebar in Nepal to Muzaffarpur in India.

Know! more about the project• The Arun-3 Hydro Electric project (900 MW) is located on Arun River in Sankhuwasabha District of

Eastern Nepal.

• The Arun River is a trans-boundary river and is part of the Kosi or Sapt Koshi river system in Nepal. It originates in Tibet Autonomous Region of the People’s Republic of China where it is called the Phung Chu or Bum-chu.

• The Run-of-River scheme envisages about 70 mtr. High concrete gravity dam and Head Race Tunnel (HRT) of 11.74 Km. with underground Power House containing four generating units of 225 MW each on Left Bank.

• Satluj Jal Vidyut Nigam (SJVN) limited bagged the project through International Competitive Bidding. An MoU was signed between Government of Nepal and SJVN Limited for the project in March, 2008 for execution on Build Own Operate and Transfer (BOOT) basis for a period of 30 years including five years of construction period.

Know! about SJVN• SJVN Limited, a Mini Ratna, Category-I and Schedule –‘A’ CPSE under administrative control of Ministry of

Power, Govt. of India, was incorporated on May 24, 1988 as a joint venture of the Government of India (GOI) and the Government of Himachal Pradesh (GOHP).

• SJVN is now a listed Company having shareholders pattern of 62.44 % with Govt. of India,26.85% with Govt. of Himachal Pradesh and rest of 10.71 % with Public.

Page 17: JOIN THE DOTS! - Career Launcher · dots! A current affairs series for UPSC Examination’ series which will help you pick up relevant news items of the day from various national

Page: 11 Join the dots! – March 2019

• Beginning with a single Project and single State operation (i.e. India’s largest 1500 MW Nathpa Jhakri Hydro Power Station in Himachal Pradesh) the Company has commissioned four projects totalling 2003.2 MW of installed capacity.

• SJVN is presently implementing Power Projects in Himachal Pradesh, Uttarakhand, Bihar, Maharashtra and Gujarat in India besides neighbouring countries viz. Nepal and Bhutan totalling 4018 MW.

Indo-Pacific Regional Dialogue

Relevance IN – Prelims (about the IPR Dialogue) + Mains (GS II international organisations)

What’s the NEWS

• The second edition of Indo-Pacific Regional Dialogue (IPRD) - 2019 will be held on 05 and 06 March 2019 at the Manekshaw Centre, New Delhi.

Know! more about the dialogue

• This dialogue will build upon the foundation laid by the inaugural edition and will examine five fresh themes:

(i) Practical solutions for achieving cohesion in the region through maritime connectivity;

(ii) Measures to attain and maintain a free-and-open Indo-Pacific;

(iii) A regional approach to the region’s transition from a ‘Brown’ to a ‘Blue’ economy;

(iv) Opportunities and challenges arising from the maritime impact of ‘Industry 4.0’; and

(v) How the twin conceptualisations of ‘SAGAR’ and ‘SAGARMALA’ might best be made mutually-reinforcing on a regional level.

• These themes would be addressed in five sessions spread over two days, with three of the sessions being steered as panel-discussions, which would encourage a freer flow of ideas and views and ensure greater audience-interaction.

• IPRD-2019 is likely to witness active participation of globally-renowned domain experts and policy-makers from thirteen countries of the Indo-Pacific — Australia, Bangladesh, Canada, China, Indonesia, Israel, Japan, Seychelles, Singapore, South Korea, Sri Lanka, the United Kingdom, and the United States of America.

• Through this annual dialogue, the Indian Navy and the National Maritime Foundation, aim to provide a platform for substantive and insightful discussions pertaining to the geopolitical developments affecting the maritime domain of the Indo-Pacific, and provide policy-relevant inputs to the policy-makers and the public at large.

Know! more about Indo-Pacific Regional Dialogue (IPRD)

• The idea of an Indo-Pacific Regional Dialogue (IPRD) was first conceptualised and conducted in 2018, as the apex level conference of the Indian Navy, organised by the National Maritime Foundation as the Navy’s Knowledge Partner.

• The permanent theme of this annual dialogue is a review of India’s opportunities and challenges in the Indo-Pacific region.

• The aim is to focus attention on the Indo-Pacific, as a maritime geographical-entity, while deliberating aspects of great relevance to regional geopolitics.

• The 2018 edition of the IPRD sought to highlight the opportunities that lay before India’s maritime policy-shapers, policy-makers, and, the practitioners of the country’s maritime policies.

EASE Reforms for Public Sector Banks

Relevance IN – Prelims (about EASE reforms outcome) + Mains (GS III economic reforms)

What’s the NEWS

• The BCG-IBA report EASE Reforms for Public Sector Banks measures the performance of each PSB on 140 objective metrics across 6 themes.

Page 18: JOIN THE DOTS! - Career Launcher · dots! A current affairs series for UPSC Examination’ series which will help you pick up relevant news items of the day from various national

Join the dots! – March 2019 Page: 12

Know! the findings of the report• Punjab National Bank has been ranked first among public sector banks in the implementation of ‘reforms

agenda’, followed by Bank of Baroda and State Bank of India.

• The six Public sector banks which are under PCA framework of the RBI have also been ranked under the Index.

• The performance of PSBs is as shown Indian Overseas Bank (66.7), UCO Bank (64.1), United Bank of India (60.8), IDBI Bank (60.2), Central Bank of India (55.7) and Dena Bank (53.8).

Know! the genesis of EASE -Index• The government had announced in January 2018 that the government would come out with EASE -Index for

ranking of banks aimed at increasing the public accountability of PSBs as independent agencies evaluate and rank PSBs annually on reforms.

• The report has been commissioned through Indian Banks’ Association and authored by BCG with Forrester Inc., Kantar IMRB and TransUnion CIBIL as knowledge partners

Construction Technology India-2019 Expo-cum-conference

Relevance IN – Prelims (about the conference) + Mains (GS III infrastructure development)

What’s the NEWS• Prime Minister Narendra Modi has declared ‘April 2019-March 2020’ as Construction-Technology year at the

Construction Technology India-2019 Expo-cum-conference in New Delhi.

Construction-Technology year• Construction-Technology year emphasises the role of advanced technology to meet the increasing demand for

housing in the country

• The declaration of Construction year aims to give the housing sector a new pace by using the latest technologies available in the world.

• The integration of modern technology to the housing sector would make the sector more dynamic and vibrant.

• India has even launched the Global Housing Technology Challenge to fast-track the construction of affordable housing and meet the target of constructing 1.2 crore houses by 2022.

• Government is also focussing on bringing systematic reforms to provide technical skills to the youth and making changes in engineering and technology curriculum.

• At the Construction Technology India-2019 Expo-cum-conference, Prime Minister also launched the GHTC-India mobile application, an interactive platform for all stakeholders for the exchange of knowledge on innovation and alternative housing technologies.

National Policy on Software Products – 2019

Relevance IN – Prelims (about the objectives of the policy) + Mains (GS III awareness in the field of IT and issues related to intellectual property rights)

What’s the NEWS• The National Policy on Software Products – 2019 has been approved by the Union Cabinet.

Know! more about the policy• The policy is aimed at developing India as a Software Product Nation. An outlay of Rs 1500 crore divided

into Software Product Development Fund and Research & Innovation fund is envisaged to implement the programmes/ schemes envisaged under this policy over a period of 7 years.

• The Policy is aimed at giving a direction for the formulation of several schemes, initiatives, projects and measures for the development of Software products sector in the country as per the roadmap envisaged therein.

Five Missions of the National Policy on Software Products – 2019• To promote the creation of a sustainable Indian software product industry, driven by intellectual property (IP),

leading to a ten-fold increase in India’s share of the Global Software product market by 2025.

• To nurture 10,000 technology startups in the software product industry, including 1000 such technology startups in Tier-II and Tier-III towns & cities and generating direct and indirect employment for 3.5 million people by

Page 19: JOIN THE DOTS! - Career Launcher · dots! A current affairs series for UPSC Examination’ series which will help you pick up relevant news items of the day from various national

Page: 13 Join the dots! – March 2019

2025.

• To create a talent pool for software product industry through (i) up-skilling of 1,000,000 IT professionals, (ii) motivating 100,000 school and college students and (iii) generating 10,000 specialized professionals that can provide leadership.

• To build a cluster-based innovation-driven ecosystem by developing 20 sectoral and strategically located software product development clusters having integrated ICT infrastructure, marketing, incubation, R&D/testbeds and mentoring support.

• In order to evolve and monitor scheme & programmes for the implementation of this policy, the National Software Products Mission will be set up with participation from Government, Academia and Industry.

• To create a robust software product ecosystem the Government has approved the National Policy on Software Products – 2019 to develop India as the global software product hub, driven by innovation, improved commercialisation, sustainable Intellectual Property (IP).

Crop burning raises risk of respiratory illness threefold, says IFPRI study

Relevance IN – Prelims (about the findings of the survey and IFPRI) + Mains (GS environment conservation)

What’s the NEWS• The burning of agricultural residue — a contributor to north India’s winter pollution — increases the risk of

respiratory illnesses threefold for those who experience it.

• It may also be responsible for an annual $30 billion (approximately ₹2 trillion) loss in terms of days of work lost in States affected by crop burning, according to a study by the International Food Policy Research Institute (IFPRI).

Know! about the findings• The findings were based on a study of the health records of 250,000 people in Haryana (which sees a spike

in crop burning episodes in winter), and Andhra Pradesh and Tamil Nadu, which don’t see similar burning episodes.

• The researchers used health records and satellite data from September 2013-February 2014.

• The satellite data was for crop-burning fires detected by the Moderate-Resolution Imaging Spectroradiometer

Page 20: JOIN THE DOTS! - Career Launcher · dots! A current affairs series for UPSC Examination’ series which will help you pick up relevant news items of the day from various national

Join the dots! – March 2019 Page: 14

(MODIS) Terra satellite, managed by the National Aeronautics Space Administration (NASA).

• In Haryana, 5.4% of surveyed individuals reported suffering from ARI (Acute Respiratory Infection) whereas the reported ARI symptoms in southern States was only 0.1%.

• In 2013, the National Green Tribunal (NGT) issued a directive to Punjab, Haryana and Uttar Pradesh, asking them to ban stubble burning.

Know! about The International Food Policy Research Institute (IFPRI)• IFPRI is an international agricultural research center founded in the early 1970s to improve the understanding

of national agricultural and food policies to promote the adoption of innovations in agricultural technology.

• Additionally, IFPRI was meant to shed more light on the role of agricultural and rural development in the broader development pathway of a country.

• The mission of IFPRI is to provide research-based policy solutions that sustainably reduce poverty and end hunger and malnutrition.

• IFPRI carries out food policy research and disseminates it through hundreds of publications, bulletins, conferences, and other initiatives.

• IFPRI was organized as a District of Columbia non-profit, non-stock corporation on March 5, 1975, and its first research bulletin was produced in February 1976

• IFPRI has offices in several developing countries, including China, Ethiopia, and India, and has research staff working in many more countries around the world.

India to tie-up with 4 nations to save rhinos

Relevance IN – Prelims (about rhinos) + Mains (GS environment conservation)

What’s the NEWS• India will collaborate with Bhutan, Nepal, Indonesia and Malaysia to increase the population of three species

of Asian rhinos, including the Greater one-horned rhinoceros found in the Indian sub-continent.

Know! more about the declaration• The five rhino range nations

signed a declaration ‘The Declaration on Asian Rhinos 2019’ for the conservation and protection of the species at the recently held Second Asian Rhino Range Countries meeting here.

• The declaration was signed to conserve and review the population of the Greater one-horned, Javan and Sumatran rhinos every four years to reassess the need for joint actions to secure their future.

• The national strategy will pave the path for long-term conservation of the Greater one-horned rhinos in India

• The declaration includes undertaking studies on health issues of the rhinos, their potential diseases and taking necessary steps; collaborating and strengthening wildlife forensics and strengthening of transboundary collaboration among India, Nepal and Bhutan for conservation of the Greater one-horned rhino.

Page 21: JOIN THE DOTS! - Career Launcher · dots! A current affairs series for UPSC Examination’ series which will help you pick up relevant news items of the day from various national

Page: 15 Join the dots! – March 2019

World Wildlife Day 2019

Relevance IN – Prelims (about world wildlife day)

What’s the NEWS• The World Wildlife day was observed on March 3rd with the theme ‘Life below water: for people and planet.’

Objectives• To provide an opportunity to celebrate the beautiful and varied forms of wild fauna and flora.

• To raise awareness of the multitude of benefits that conservation provides to people.

• To remind the world of the urgent need to step up the fight against wildlife crime and human-induced reduction of species, which have a wide-ranging economic, environmental and social impact.

• The theme of World Wildlife Day 2019 ‘Life below water: for people and planet’ complements the Goal 14 of the Sustainable Development Goals (SDG14) aims to “conserve and sustainably use the oceans, seas and marine resources for sustainable development.”

• Sustainable Development Goal #15 - Halting biodiversity loss.

Know! about the inception• The United Nations General Assembly (UNGA) on its 68th session on 20 December 2013 decided to proclaim

3rd March as World Wildlife Day.

• March 3rd is the day of the adoption of the Convention on International Trade in Endangered Species of Wild Fauna and Flora (CITES)

Factoids

What’s the NEWS• The Central Industrial Security Force (CISF) created a new Guinness World Record for the longest single

moving line bicycle parade at the Yamuna Expressway.

• The world record was attempted to commemorate the 50 years of the paramilitary force CISF.

Know! about Central Industrial Security Force• The CISF came into existence in 1969 with a modest three battalions, to provide integrated security cover to the

Public Sector Undertakings (PSUs) which had occupied the commanding heights of the economy in those days.

• In a span of five decades, the Force has grown several folds to reach one lakh forty-eight thousand and three hundred seventy-one personnel.

• CISF is no longer a PSU-centric organization. Instead, it now a premier multi-skilled security agency of the country, mandated to provide security to major critical infrastructure installations of the country in diverse areas.

• CISF is currently providing security cover to nuclear installations, space establishments, airports, seaports, power plants, sensitive Government buildings heritage monuments and even Private entities.

PSBloansin59minutes.com

Relevance IN – Prelims (about MSME reforms) + Mains(GS III economic development)

What’s the NEWS• PSBloansin59minutes.com which was launched in 2018 has emerged as the country’s largest online lending

platform, with loan sanctions exceeding over Rs 35,000 crore.

• The portal provides credit up to Rs 1 crore to micro, small and medium enterprises (MSMEs) in just 59 minutes or less than an hour.

Know! more about the benefits of the portal• The online portal has reduced the turnaround time from 20-25 days to 59 minutes and upon approval, the loan

is disbursed in 7-8 working days.

• The automated loan processing system together with ease has helped in fostering transparency.

Page 22: JOIN THE DOTS! - Career Launcher · dots! A current affairs series for UPSC Examination’ series which will help you pick up relevant news items of the day from various national

Join the dots! – March 2019 Page: 16

• The online portal has aided in eliminating the discretion at the bankers’ end, as in-principle approval of loans does not require human intervention.

• Through the Portal, the MSMEs apply for a loan using their GST registration as the portal is integrated with GST server at the back-end as well as IT, credit bureaus and banks.

One Nation One Card

Relevance IN – Prelims (about the features of the card)

What’s the NEWS

• PM Modi launched “One Nation One Card” in Ahmedabad, which can be used for paying for travel on any mode of transportation.

Know! more about the card

• Roll-out of the card and a new feature in all credit and debit cards issued by banks will enable people to use them either in PoS machines deployed at ticket counters or just use them like Metro Rail smart cards to enter a platform.

• All new credit and debit cards issued by most banks have the National Common Mobility Card feature to make payments for their travel. This is like any other wallet

• Delhi Metro has started a pilot run of indigenously designed automatic fare collection counters (AFCs) that can read these cards for seamless entry and exit at stations.

• All new Metro rail networks would get these new AFCs, existing networks will gradually replace the operational ones.

• India was one of the few countries with indigenous technology for AFCs and the mass rollout would reduce their prices significantly.

Page 23: JOIN THE DOTS! - Career Launcher · dots! A current affairs series for UPSC Examination’ series which will help you pick up relevant news items of the day from various national

Page: 17 Join the dots! – March 2019

• Till now, India was dependent on foreign technology players for this and they were charging high royalty, which

was the reason for the products being expensive.

US to withdraw duty benefits on $5.6 billion exports from India: Generalized System of Preferences (GSP)

Relevance IN – Prelims (about GSP) + Mains (GS III economic development + GS II effect of policies of developed and developing countries on India’s interest)

Highlights

• The GSP programme allows duty-free entry of 1,784 products from India into the US

• Exporters of textiles, engineering, gems and jewellery and chemical products benefit the most from this programme

What’s the NEWS

• Arguing that New Delhi had failed to assure America that it would provide equitable and reasonable access to

its markets in numerous sectors, US President Donald Trump informed the US Congress about his intent to

terminate the designation of India and Turkey as a beneficiary developing country under the Generalised

System of Preferences (GSP) programme.

• The US has decided to withdraw duty benefits on $5.6 billion worth of exports from India by May this year,

blaming the trade barriers created by the Indian government, after negotiations for a trade package fell through.

Page 24: JOIN THE DOTS! - Career Launcher · dots! A current affairs series for UPSC Examination’ series which will help you pick up relevant news items of the day from various national

Join the dots! – March 2019 Page: 18

Know! about the GSP Programme • The GSP programme allows duty-free entry of 1,784 products from India into the US, benefitting exporters of

textiles, engineering, gems and jewellery and chemical products.

• Total US imports under GSP in 2017 was $21.2 billion, of which India was the biggest beneficiary with $5.6 billion, followed by Thailand ($4.2 billion) and Brazil ($2.5 billion).

• India is the world’s largest beneficiary of the GSP program and ending its participation would be the strongest punitive action against India since Trump took office in 2017

• Under the United States GSP programme, certain products can enter the US duty-free if the beneficiary developing countries meet the eligibility criteria established by Congress.

• The GSP criteria include, among others, respecting arbitral awards in favour of US citizens or corporations, combatting child labour, respecting internationally recognised worker rights, providing adequate and effective intellectual property protection and providing the US with equitable and reasonable market access.

• Countries can also be graduated from the GSP programme, depending on factors related to economic development.

QS World Ranking: Five Indian institutes in top 300, IIT Bombay leads India chart

Relevance IN – Prelims (about QS ranking outcome) + Mains (GS II issues relating to development and management of social sector/services relating to education + international organisations)

What’s the NEWS• QS World Ranking: IIT Bombay, IIT Delhi and IISc remain the top three Indian institutes. Additionally, top five

Indian institutes were ranked in the top 300. Indian institutes rank ‘very high’ in research output.

Know! more about the ranking• According to the recently released QS world ranking, five Indian institutes have secured a spot in the list of top

300 world universities. The Indian Institute of Technology (IIT), Bombay has gained the top rank among Indian institutes followed by Indian Institute of Science (IISc), Bangalore.

• IIT Bombay surpassed last year’s top institute IIT Delhi by moving 17 ranks up and securing 162nd rank worldwide.

• IISc has shown the maximum improvement with a jump of 20 positions from last year. It moved up from 190th rank last year to 170.

• IIT Delhi remained at 172nd spot globally, however, in Indian ranking it went down to the third spot as IIT Bombay and IISc improved their score.

Institute 2018 2019 IIT Delhi 172 172 IT Bombay 179 162 IISc 190 170 IIT Madras 264 264 IIT Kanpur 293 283

• IIT Kanpur, which secured the fifth rank in the country and 283rd in the world also moved up by 10 ranks from last year showing a significant improvement.

• All the Indian institutes have got ‘very high’ ranking in research output, according to the QS World Ranking.

• Internationally, the top three institutes remain unshakable with the Massachusetts Institute of Technology (MIT), Stanford University and Harvard University securing first, second and third rank respectively, according to the QS World Ranking.

• In the top five, Oxford University replaced the University of Cambridge for the fifth rank.

Top five world institutes this year were –• Massachusetts Institute of Technology

• Stanford University

• Harvard University

Page 25: JOIN THE DOTS! - Career Launcher · dots! A current affairs series for UPSC Examination’ series which will help you pick up relevant news items of the day from various national

Page: 19 Join the dots! – March 2019

• California Institute of Technology

• University of Oxford

Know! about QS World University Rankings • It is an annual publication of university rankings by Quacquarelli Symonds (QS). Previously known as Times

Higher Education–QS World University Rankings, the publisher had collaborated with Times Higher Education (THE) magazine to publish its international league tables from 2004 to 2009 before both started to announce their own versions.

• QS then chose to continue using the pre-existing methodology while Times Higher Education adopted a new methodology to create their rankings.

• The QS system now comprises the global overall and subject rankings (which name the world’s top universities for the study of 48 different subjects and five composite faculty areas), alongside five independent regional tables (Asia, Latin America, Emerging Europe and Central Asia, the Arab Region, and BRICS).

Government of India and World Bank Sign $250 Million Agreement for the National Rural Economic Transformation Project (NRETP) to boost Rural Incomes across 13 States in India

Relevance IN – Prelims (about the agreement and NRETP) + Mains (GS I role of women and women organisation + GS II social Justice, role of SHG, welfare schemes for the vulnerable section

What’s the NEWS• The World Bank and the Government of India signed a $250 Million Agreement for the National Rural Economic

Transformation Project (NRETP) which will help women in rural households shift to a new generation of economic initiatives by developing viable enterprise for farm and non-farm products.

Know! more about the project and agreement• A Key Focus of the Project will be to promote women-owned and women-led farm and non-farm enterprises

across value chains; enable them to build businesses that help them access finance, markets and networks; and generate employment.

• The National Rural Economic Transformation Project (NRETP) is an additional financing to the $500 million National Rural Livelihoods Project (NRLP) approved by the World Bank in July 2011.

• The NRLP which is currently being implemented across 13 states, 162 districts and 575 blocks, has so far mobilized more than 8.8 million women from poor rural households into 750,000 self-help groups (SHGs).

• These SHGs have been further federated into 48,700 Village Organizations and 2900 Cluster/Gram Panchayat-level Federations.

• While these 13 states will continue to be supported under the new project signed today, 125 new districts will be added from within these states.

Know! about NRLM• The National Rural Livelihoods Mission (NRLM) aims to alleviate rural poverty and create sustainable livelihood

opportunities in rural communities by promoting sustainable community-based institutions which will facilitate economic and financial services for the rural poor.

• The National Rural Livelihoods Project has mobilized close to 9 million rural women into strong self-managed institutional platforms, helping them access services, participate in Gram Sabhas and start engaging in income generating activities.

• Many of these women are now geared towards moving into higher order economic initiatives in farm and non-farm sectors.

• Since its launch in 2011, the National Rural Livelihoods Mission (NRLM) has mobilized 50 million poor rural women into self-help groups and their higher-level federations. These groups have leveraged nearly $30 billion from commercial banks.

• The NERTP will support enterprise development programs for rural poor women and youth by creating a platform to access finance including start-up financing options to build their individual and/or collectively owned and managed enterprises.

Page 26: JOIN THE DOTS! - Career Launcher · dots! A current affairs series for UPSC Examination’ series which will help you pick up relevant news items of the day from various national

Join the dots! – March 2019 Page: 20

• The other key component of the project includes developing financial products using digital financial services to help small producer collectives scale-up and engage with the market. It will also support youth skills development, in coordination with the Deen Dayal Upadyaya Grameen Kaushalya Yojana.

AL NAGAH 2019

Relevance IN – Prelims (about AL NAGAH)

What’s the NEWS• Exercise Al Nagah III, third in the series

of bilateral joint exercise between India and Oman is scheduled to be held from 12 to 25 March 2019 at Jabel Al Akhdar Mountains in Oman.

Know! more about the exercise• The exercise will see both the armies

exchanging expertise and experience in tactics, weapon handling and firing, with an aim to enhance interoperability in counterterrorist operations in semi urban mountainous terrain.

• India-Oman bilateral security ties have continued to develop since the beginning of India-Oman Joint Military Cooperation meetings in 2006.

• Exercise Al Nagah III follows the first two joint exercises that were held in Oman in January 2015 and India in March 2017 respectively.

• Similar exercises are also in vogue between the navies and air forces of both the nations thus underscoring the growing bilateral military and strategic partnership between the two important nations of Indian Ocean Region (IOR).

Pradhan Mantri Shram Yogi Maan-Dhan (PM-SYM): Prime Minister Launches PM-SYM in Ahmedabad

Relevance IN – Prelims (about PM -SYM) + Mains (GS II social Justice, welfare schemes for the vulnerable section of the population)

What’s the NEWS• Prime Minister Shri Narendra Modi formally

launched Pradhan Mantri Shram Yogi Maan-Dhan (PM-SYM) Scheme in Ahmedabad.

Role of Common Service Centres • CSCs spread across the country are enrolling

PM-SYM beneficiaries. There are over three lakh CSCs in the country.

• Nearly 12 lakh youth are working in CSCs and they are instrumental in smooth implementation of not only PMSYM but other welfare schemes like Aayushman Bharat Yojana.

Know! about PM-SYM• Pradhan Mantri Shram Yogi Maan-Dhan (PM-SYM), a pension scheme for Unorganized Workers is being

implemented for an estimated 42 crore workers in the unorganized sector constituting around 85% of the total Labour force of the country.

• PM-SYM has been made effective since 15thFeb, 2019 in all states.

Page 27: JOIN THE DOTS! - Career Launcher · dots! A current affairs series for UPSC Examination’ series which will help you pick up relevant news items of the day from various national

Page: 21 Join the dots! – March 2019

• It is a central sector scheme open to unorganised workers, whose monthly income is Rs 15000/- or below and who have an Aadhar number as well as savings bank account. The minimum age for joining the scheme is 18 years and the maximum is 40 years.

• It is a voluntary and contribution based scheme under which the subscriber gets an assured minimum monthly pension of Rs 3000/- from the age of 60 years onwards.

• The contribution of a subscriber ranges from Rs 55/- to Rs 200/- depending on his entry age which is 18 to 40.

• Under the scheme, the Central Government will also give matching contribution towards beneficiary’s pension account.

• Further, it has unique feature that in case of exit, subscriber would be returned his entire contribution.

• The unorganized workers are mostly engaged as home based workers, street vendors, mid-day meal workers, head loaders, brick kiln workers, cobblers, rag pickers, domestic workers, washer men, rickshaw pullers, rural landless labourers, own account workers, agricultural workers, construction workers, beedi workers, handloom workers, leather workers, Audio- visual workers, and in similar other occupations.

Union Home Minister launches Smart Fencing on Indo-Bangladesh border

Relevance IN – Prelims (about the smart fencing technology) + Mains (GS III security challenges and their management in border areas

What’s the NEWS• Two pilot projects covering about 71 Kms on Indo-Pakistan Border (10 Kms) and Indo-Bangladesh Border

(61 Kms) of Comprehensive Integrated Border Management System (CIBMS) have been completed.

• The Stage has been set now for taking up Stage-II and Stage-III covering about 1955 Kms of the border which cannot be physically fenced.

Know! about BOLD -QIT project under CIBMS• The CIBMS project will vastly improve the capability of BSF in detecting and controlling the cross border

crimes like illegal infiltration, smuggling of contraband goods, human trafficking and cross border terrorism etc.

• The project BOLD-QIT (Border Electronically Dominated QRT Interception Technique) under Comprehensive Integrated Border Management System (CIBMS) on Indo- Bangladesh border in Dhubri district of Assam.

• Earlier in the month of September, 2018, two pilot projects of “smart” border fencing built under the CIBMS programme in Jammu were operationalised.

• At Dhubri, where BOLD-QIT project has been implemented along riverine border, it was not feasible to construct border fencing.

Page 28: JOIN THE DOTS! - Career Launcher · dots! A current affairs series for UPSC Examination’ series which will help you pick up relevant news items of the day from various national

Join the dots! – March 2019 Page: 22

• The 61 kms of border area in Dhubri where River Brahmaputra enters into Bangladesh consists of vast char lands and innumerable river channels thus making border guarding in this area a challenging task especially during rainy season.

• To address these problems Ministry of Home Affairs has decided to use technological solutions to enhance capabilities of manpower of Border Security Force (BSF) deployed on the ground.

Features of CIBMS• CIBMS involves deployment of a range of state-of-the-art surveillance technologies — thermal imagers, infra-

red and laser-based intruder alarms, aerostats for aerial surveillance, unattended ground sensors that can help detect intrusion bids, radars, sonar systems to secure riverine borders, fibre-optic sensors and a command and control system that shall receive data from all surveillance devices in real time.

• Implementation of CIBMS projects on Indo - Pakistan and Indo - Bangladesh border will enhance the capabilities of Border security Force (BSF).

Textiles Minister inaugurates Refurbished Handloom Haat

Relevance IN – Prelims(about the new projects) + Mains (GS II government policies and intervention for the development of various sectors)

What’s the NEWS• Union Textiles Minister, Smriti Zubin Irani, inaugurated the renovated Handloom Haat in New Delhi

• She also launched three projects of NIFT - VisionNXT – Trend Forecasting Initiative, Indian Textiles and Craft Repository and Design Innovation and Incubation.

Know! more about these new projects • The Haat at Janpath in New Delhi has been set up by Ministry of Textiles to provide marketing opportunities

to authentic handloom products from various States, PSUs and cooperative societies.

• Its main objective is to provide infrastructure support to handloom agencies to augment their sales of handloom products and to showcase the exquisite variety of handloom products produced all over the country.

Know! about NIFT - VisionNXT – Trend Forecasting Initiative, Indian Textiles and Craft Repository and Design Innovation and Incubation. • The NIFT will accomplish their job through artificial intelligence.

• The virtual museum will not only help industry and research scholars but also carry forward the knowledge to the next generation.

• Project related to trend innovation lab ‘VisionNxt’ initiative being set up by NIFT in the building will create an indigenous fashion forecasting service that endeavours to design seasonal directions for our country.

Page 29: JOIN THE DOTS! - Career Launcher · dots! A current affairs series for UPSC Examination’ series which will help you pick up relevant news items of the day from various national

Page: 23 Join the dots! – March 2019

• The project of Indian Textiles and Craft Repository Initiative of NIFT is supported by the DC Handlooms and the DC Handicrafts, Ministry of Textiles.

• The body of textile and craft knowledge generated through the Craft Cluster Initiative will be channelled into a national knowledge portal titled Indian Textile & Craft Repository.

Independent Verification of Swachh Bharat Grameen confirms over 96% usage of toilets

Relevance IN – Prelims – (about the survey outcomes and about SBM) + Mains (GS II government policies and interventions for the development of various sectors)

What’s the NEWS

• The National Annual Rural Sanitation Survey (NARSS) 2018-19, conducted by an Independent Verification Agency (IVA) under the World Bank support project to the Swachh Bharat Mission Grameen (SBM-G), has found that 96.5% of the households in rural India who have access to a toilet use it.

• The NARSS also re-confirmed the Open Defecation Free (ODF) status of 90.7% of villages which were previously declared and verified as ODF by various districts/States.

Know! more about the survey outcomes

• The survey was conducted between November 2018 and February 2019 and covered 92040 households in 6136 villages across States and UTs of India.

The key findings of NARSS 2018-19 are as follows:

• 93.1% of households were found to have access to toilets during the survey period (the corresponding figure as per the SBMG MIS in November 2018 was 96%)

• 96.5% of the people who had access to toilets used them

• 90.7% of villages which were previously declared and verified as ODF were confirmed to be ODF. The remaining villages also had sanitation coverage of about 93%

• 95.4% of the villages surveyed found to have minimal litter and minimal stagnant water

Know! more about the survey methodology

• The IVA presented their findings to the Expert Working Group (EWG) constituted for oversight of NARSS, comprising representatives from organizations including the World Bank, UNICEF, Water Aid, Bill & Melinda Gates Foundation, India Sanitation Coalition, NITI Aayog, and Ministry of Statistics and Program Implementation.

• The survey used the PPS (Probability Proportion to Size) sampling methodology, which yields results within a confidence interval of 95%.

• Data was collected using the Computer Assisted Personal Interviewing (CAPI) platform. The survey also covered schools, anganwadis and public/community toilets in these villages.

Know! about Swatch Bharat Mission

• Since its launch in October 2014, the SBM, the world’s largest sanitation program, has changed the behaviour of hundreds of millions of people with respect to toilet access and usage.

• 500 million people have stopped defecating in the open since the SBM began, down from 550 million at the beginning of the programme to less than 50 million today.

• Over 9 crore toilets have been built across rural India under the Mission. Over 5.5 lakh villages and 615 districts have been declared ODF, along with 30 ODF States and Union Territories.

Page 30: JOIN THE DOTS! - Career Launcher · dots! A current affairs series for UPSC Examination’ series which will help you pick up relevant news items of the day from various national

Join the dots! – March 2019 Page: 24

Ministry of Youth Affairs and Sports recognises Kudo International Federation India as National Sport Federation

Relevance IN – Prelims (about the benefits of getting the recognition)

What’s the NEWS

• Ministry of Youth Affairs and Sports has granted provisional recognition to Kudo International Federation India(KIFI) as National Sport Federation with immediate effect.

• The grant of recognition is subject to compliance of the provisions of National Sports Development Code of India, 2011 as modified from time to time, by the federation including displaying information on their website as per the instructions issued by the Ministry.

• The recognition means granting a major role to KIFI Association for promotion and Development of Kudo sport in India.

• The Kudo sport is placed in the ‘Others’ category.

7th March 2019 to be celebrated as ‘JanaushadhiDiwas’ across India

Relevance IN – Prelims(about PMBJP) + Mains (GS II issues related to development and management of social sector/services relating to health + welfare schemes for vulnerable sections

What’s the NEWS

• In the direction of making quality healthcare affordable for all, the Government has taken important steps to make affordable and quality generic medicines popular among the people through Pradhan MantriBhartiyaJanaushadhiPariyojana (PMBJP)

• For providing further impetus & creating awareness about use of generic medicines, it has been decided to celebrate 7th March 2019 as ‘JanaushadhiDiwas’ across India.

• PMBJP Kendras to increase the awareness about the scheme. These programmes would witness participation of doctors, health experts, NGOs and beneficiaries, which would help in spreading awareness about the scheme to common masses.

Know! more about PMJAY

• In September 2015, the ‘Jan Aushadhi Scheme’ was revamped as ‘Pradhan Mantri Jan Aushadhi Yojana’ (PMJAY) and in november 2016, to give further impetus to the scheme, it was again renamed as “Pradhan MantriBhartiyaJ anaushadhi Pariyojana” (PMBJP) under the Department of Pharmaceuticals, Ministry of Chemicals & Fertilizers with an objective of making available quality generic medicines at affordable prices to all.

• This has been implementing by the Bureau of Pharma PSUs of India (BPPI), under the administrative control of Department of Pharmaceuticals, Ministry of Chemicals & Fertilizers.

Salient features of the scheme:

• Ensure access to quality medicines

• Extend coverage of quality generic medicines so as to reduce the out of pocket expenditure on medicines and thereby redefine the unit cost of treatment per person

• Create awareness about generic medicines through education and publicity so that quality is not synonymous with only high price

• A public program involving Government, PSUs, Private Sector, NGO, Societies, Co-operative Bodies and other Institutions

• Create demand for generic medicines by improving access to better healthcare through low treatment cost and easy availability wherever needed in all therapeutic categories.

Page 31: JOIN THE DOTS! - Career Launcher · dots! A current affairs series for UPSC Examination’ series which will help you pick up relevant news items of the day from various national

Page: 25 Join the dots! – March 2019

Amendment in Hazardous Waste (Management& Transboundary Movement) Rules, 2016

Relevance IN – Prelims (about the amendments) + Mains (GS III environment conservation)

What’s the NEWS• In order to strengthen the implementation of environmentally sound management of hazardous waste in the

country, the Ministry of Environment, Forest and Climate Change has amended the Hazardous and Other Wastes (Management & Transboundary Movement) Rules, 2016

Some of the salient features of the Hazardous and Other Wastes (Management& Transboundary Movement) Amendment Rules, 2019 are as follows:

1. Solid plastic waste has been prohibited from import into the country including in Special Economic Zones (SEZ) and by Export Oriented Units (EOU).

2. Exporters of silk waste have now been given exemption from requiring permission from the Ministry of Environment, Forest and Climate Change.

3. Electrical and electronic assemblies and components manufactured in and exported from India, if found defective can now be imported back into the country, within a year of export, without obtaining permission from the Ministry of Environment, Forest and Climate Change.

4. Industries which do not require consent under Water (Prevention and Control of Pollution) Act 1974 and Air (Prevention and Control of Pollution) Act 1981, are now exempted from requiring authorization also under the Hazardous and Other Wastes (Management & Transboundary Movement) Rules, 2016, provided that hazardous and other wastes generated by such industries are handed over to the authorized actual users, waste collectors or disposal facilities.

Smt. Maneka Gandhi felicitates ‘Web Wonder Women’

Relevance IN – Prelims (about various initiatives by the ministry)

What’s the NEWS

Page 32: JOIN THE DOTS! - Career Launcher · dots! A current affairs series for UPSC Examination’ series which will help you pick up relevant news items of the day from various national

Join the dots! – March 2019 Page: 26

• The Ministry of Women and Child Development hosted a felicitation event for Web Wonder Women, a campaign organised to celebrate the exceptional achievements of women, who have been driving social reforms via social media.

Know! more about the event• Organised in collaboration with Twitter India and Breakthrough India, the event aimed to recognise the

fortitude of Indian women stalwarts from across the globe who have used the power of social media to run positive campaigns to steer a change in society.

• #WebWonderWomen was a campaign to recognise, honour and encourage such voices that have in their own capacity driven a positive impact on social media platforms

• The WCD Minister, along with a panel of 10 judges, finalised the names of 30 women who have impacted and influenced society through social media.

Know! some more campaigns of the ministry• On 8th March, the Ministry honours women who have contributed to society but have remained unsung with

the Nari Shakti Puruskar, which is conferred by the President of India at Rashtrapati Bhawan.

• Along with these Puruskars, the Ministry started a new system of recognising women who have broken the glass ceiling and ventured into unusual fields.

• In 2018, the Ministry hosted ‘First Ladies’, a first-of-its-kind Government initiative to felicitate exceptional women who were the first to set a milestone in their respective fields.

• In 2015, the Ministry had collaborated with Facebook to recognise ‘100 Women Achievers’ who have excelled in diverse sectors of public work.

• Web Wonder Women’ is the third leg of the ‘Women Achievers’ campaign of the Ministry.

Release of Dictionary of Martyrs of India’s Freedom Struggle (1857-1947)

Relevance IN – Prelims(about the dictionary) + Mains – (GS I – Indian art ans culture + freedom struggle)

What’s the NEWS• The Prime Minister, released the Dictionary of

Martyrs of India’s Freedom Struggle.

Know! about the dictionary • This five-volume dictionary contains an account of

the martyrs from India’s First War of Independence in 1857, to India’s Independence in 1947.

• This includes the martyrs of the Jallianwala Bagh massacre, the non-cooperation movement, the Quit India Movement, and those soldiers of the Azad Hind Fauj, who attained martyrdom, among many others.

• The effort of the Union Government is to nurture and recall the brave deeds of the heroes of our freedom struggle.

Know! more about the project• The project for compilation of “Dictionary of Martyrs”

of India’s Freedom Struggle was commissioned by the Ministry of Culture, to the Indian Council of Historical Research (ICHR) to commemorate the 150th anniversary of uprising of 1857.

Page 33: JOIN THE DOTS! - Career Launcher · dots! A current affairs series for UPSC Examination’ series which will help you pick up relevant news items of the day from various national

Page: 27 Join the dots! – March 2019

• In this dictionary a martyr has been defined as a person who died or who was killed in action or in detention, or was awarded capital punishment while participating in the national movement for emancipation of India.

• It includes ex-INA or ex-military personnel who died fighting the British.

• It includes the martyrs of 1857 Uprising, Jallianwala Bagh Massacre (1919), Non-Cooperation Movement (1920-22), Civil Disobedience Movement (1930-34), Quit India Movement (1942-44), Revolutionary Movements (1915-34), Kissan Movements, Tribal Movements, Agitation for Responsible Government in the Princely States (Prajamandal), Indian National Army (INA, 1943-45), Royal Indian Navy Upsurge (RIN, 1946), etc. Information of about 13,500 martyrs has been recorded in these volumes.

PM lays foundation stone of Integrated Command and Control Centres in Sikkim, Arunachal Pradesh and Tripura

Relevance IN – Prelims(about the command centre and smart city) + Mains (GS III infrastructure development)

What’s the NEWS

• The Prime Minister,laid the foundation stone of Integrated Command and Control Centres at five North Eastern Smart Cities

• The Command and Control Centres at Namchi and Gangtok in Sikkim, Itanagar and Pasighat in Arunachal Pradesh and Agartala in Tripura.

Know! about Integrated Command and Control Centres

• One of the key components of the system is the CCTV Surveillance system for Citizen Safety.

• This system is designed to help tackle crime. The Intelligent Traffic Management System will help ease traffic flow.

• The Solid Waste Management component in the Command and Control Centre will boost cleanliness in these cities.

• Smart Street Lighting will make our streets safer and citizen friendly. It will also improve energy efficiency through the conversion to LED systems.

• Smart cities are also installing Public-Address Systems and Variable Message Signs to communicate important civic information.

• Digital access is an important component of Digital India Mission. Public Wi-Fi system will provide free internet access to Citizens.

• The North-East is an environmentally sensitive region. The Environmental Monitoring System and Disaster Management modules will give real time information to Citizens and Government.

• The North-East will have its first completed Smart Command and Control Centre as early as October 2019.

Page 34: JOIN THE DOTS! - Career Launcher · dots! A current affairs series for UPSC Examination’ series which will help you pick up relevant news items of the day from various national

Join the dots! – March 2019 Page: 28

Cabinet approves Continuation of Atal Innovation Mission

Relevance IN – Prelims (about AIM) + Mains (GS III awareness in the field of science and technology)

What’s the NEWS• The Union Cabinet has approved Continuation of Atal Innovation Mission (AIM), and for incurring expenditure

up to Rs. 1000 crore till 2019-20 by Atal Innovation Mission for expanding Atal Tinkering Labs to 10,000 schools in view of their huge success at school level.

Atal Innovation Mission• AIM has multiple programs to encourage and support innovation in the country.

• State of the art Atal Tinkering Labs (ATLs) are being established in thousands of schools, world class Atal Incubation Centres (AIC) and Atal Community Innovation Centers (ACIC) are being established for universities and industry,

• Promotion of product development in areas of national relevance and social importance is being supported through Atal New India Challenges (ANICs).

Impact• The Mission has undertaken many bold and forward-looking initiatives such as Alal Tinkering Labs (ATL) and

Atal Incubation Centres (AIC), which have received great traction;

• Many Ministries/Departments of Government of India have initiated innovation related activities with the help and technical support of AIM.

• Under the ATL program, more than 10,000 schools are expected to establish these labs by 2020.

• More than 100 Atal Incubation Centres (AICs) are likely to established around the country, supporting at least 50-60 startups each over the first five years.

• More than 100 innovators/startups are expected to receive some support for productizing their innovations.

• Each incubator is expected to foster 50-60 technology driven innovative Startups every four years.

Implementation Strategy and targets• Corporate and international partners are connected to AIM beneficiaries, to ensure flow of information and

technology between academia, innovation, and the start-up ecosystem as well as provide mentoring support to ensure success of AIM’s initiatives.

• AIM’s Mentor of Change Programme is mandated to ensure the success of the ATLs. • Through this programme, well-qualified mentors from different professional backgrounds are selected to provide

pro-bono mentoring to ATL students over a range of skills.• Mentors also collaborate with the School administrators to ensure the ATLs operate in the manner

Objective of AIM• AlM’s objectives are to create and promote an ecosystem of innovation and entrepreneurship across the country

at school, university, research institutions, MSME and industry levels.

• The Mission has been set up under NITI Aayog, in accordance with the Hon’ble Finance Minister’s declaration in the 2015 Budget Speech.

National Council of Science Museums of Culture Ministry collaborates with Google Arts & Culture for largest interactive online exhibition on inventions and discoveries

Relevance IN – Prelims(about NCSM and its collaboration) + Mains (GS I Indian heritage and culture)

What’s the NEWS• The National Council of Science Museums (NCSM), an organization under the Ministry of Culture, Government

of India has collaborated with Google Arts & Culture for ‘Once Upon a Try’: Epic journeys of invention and discovery - the largest online exhibition about inventions and discoveries ever curated, as an attempt to explore humanity’s greatest inventions and discoveries in an interactive online exhibition.

Page 35: JOIN THE DOTS! - Career Launcher · dots! A current affairs series for UPSC Examination’ series which will help you pick up relevant news items of the day from various national

Page: 29 Join the dots! – March 2019

Know! more about the online exhibition• The online exhibition was launched yesterday and contains collections, stories and knowledge from over 110

renowned institutions from across 23 countries, highlighting millennia of major breakthroughs and the great minds behind them.

• Everybody can now explore more than 400 interactive exhibitions that pay tribute to humanity’s greatest leaps in science and technology, and the visionaries that shaped our world, as well as tales of epic fails and happy accidents.

• National Council of Science Museums contributes six interactive stories that share some key insights into the long and glorious Science and Technology Heritage of India.

• The treasure which was lesser known to the world is now open to all. Explore National Council of Science Museums on Google Arts & Culture at: https://artsandculture.google.com/partner/national-council-of-science-museums

• Google Arts & Culture puts the collections of more than 1,800 museums at your fingertips.

National Council of Science Museums (NCSM) • National Council of Science Museums (NCSM), a premiere institution in the field of science communication, is

an autonomous organization under the Ministry of Culture, Government of India.

• Primarily engaged in popularizing Science and Technology through a network of science centres, Mobile Science Exhibitions (MSE) units that visit rural schools and plethora of activities for public and students in particular, NCSM has now become a trend setter in the field of science communication both at national and international level.

• Presently NCSM, with its Headquarters in Kolkata, administers and manages 25 science museums/centres spread across the country and is the world’s largest network of science centres and museums that functions under a single administrative umbrella with an annual reach to about 15 million people.

Cabinet approves Kiru Hydro Electric (HE) Project (4 X 156 MW) in Jammu and Kashmir

Relevance IN – Prelims(about the Kiru dam location)

What’s the NEWS• The Cabinet Committee on Economic Affairs has approved the investment sanction for construction of Kiru

Hydro Electric(HE) Project in Jammu & Kashmir.

Know! more about the project• The project is located on River Chenab in Kishtwar district of Jammu & Kashmir.

• It envisages construction of a 135 m high concrete gravity Dam above deepest foundation level

• The project shall provide much needed power in northern grid and shall accelerate process of development of remote areas of Jammu and Kashmir. The Project is scheduled to be completed in a period of 4 1/2 years.

• Kiru HE Project is envisaged as a Run of River (RoR) Scheme, designed complying with the requirements of Indus Water Treaty 1960

Indore gets cleanest city tag for third year in a row

Relevance IN – Prelims (about Swachh Survekshan awards)

What’s the NEWS• Indore was adjudged India’s cleanest city for the third year in a row in the Centre’s ‘Cleanliness Survey’.

• The second and third positions were bagged by Ambikapur in Chhattisgarh and Mysuru in Karnataka.

Know! more about the award• The Swachh Survekshan awards 2019 were conferred by President Ram Nath Kovind in New Delhi.

• While the New Delhi Municipal Council was given award for the ‘Cleanest Small City’ award, Uttarakhand’s Gauchar bagged the ‘Best Ganga Town’.

Page 36: JOIN THE DOTS! - Career Launcher · dots! A current affairs series for UPSC Examination’ series which will help you pick up relevant news items of the day from various national

Join the dots! – March 2019 Page: 30

• The ‘Cleanest Big City’ award has been bagged by Ahmedabad, while Raipur is the ‘Fastest Moving Big City’.

• Ujjain has been the adjudged the ‘Cleanest Medium City’ and Mathura-Vrindavan bagged the tag of the ‘Fastest Moving Medium Cities’.Bhopal has been named the cleanest capital.

• Raipur won the ‘Fastest Moving Big City’. Mathura-Vrindavan won the tag of Fastest Moving Medium City.

• Chhattisgarh has bagged the top spot in the category of best performing states. Jharkhand is at the second spot while Maharashtra is at the third.

• Swachh Survekshan covered all urban local bodies in the country, making it the largest such cleanliness survey in the world.

ISRO, French agency seal agreement on maritime security

Relevance IN – Prelims(about the agreement) + Mains (GS III awareness in the field of space)

What’s the NEWS• National space agency ISRO and its French counterpart CNES on Wednesday sealed an agreement to set up a

joint maritime surveillance system in the country in May.

Know! more about the agreement• The two nations will explore putting up a constellation of low-Earth orbiting satellites that will identify and

track movement of ships globally – and in particular those moving in the Indian Ocean region where France has its Reunion Islands.

• The agreement intends to supply an operational system for detecting, identifying and tracking ships in the Indian Ocean, provides for a maritime surveillance centre to be set up in India

• The CNES-ISRO agreement [intends] to supply an operational system for detecting, identifying and tracking ships in the Indian Ocean.

• It provides for a maritime surveillance centre to be set up in India in May this year; sharing of capacity to process existing satellite data and joint development of associated algorithms

• The two agencies have put up two climate and ocean weather monitoring satellites Megha-Tropiques (of 2011) and SARAL-AltiKa (2013) that are considered a model.

• This fleet will be augmented with the launch of Oceansat-3-Argos mission in 2020 along with a joint infrared Earth-observation satellite

• ISRO and CNES will build a constellation of satellites for maritime surveillance intended to identify and track ships in the Indian Ocean.

• ISRO and CNES have signed an agreement to train Indian Scientists for the ‘Gaganyaan’ project at the Toulouse Space Centre in France.

• Indian Scientists will also be trained at CADMOS, the centre for development of microgravity applications and space operations, and the MEDES Space Clinic in France.

• India has also signed an agreement with Russia for the Gaganyaan Project.

National Mission on Transformative Mobility and Battery Storage approved by Cabinet

Relevance IN – Prelims (about the mobility mission) + Mains (GS III infrastructure development – energy + environment conservation)The Union Cabinet chaired by Prime Minister Narendra Modi has approved:

i. Setting up of a National Mission on Transformative Mobility and Battery Storage, to drive clean, connected, shared, sustainable and holistic mobility initiatives;

Page 37: JOIN THE DOTS! - Career Launcher · dots! A current affairs series for UPSC Examination’ series which will help you pick up relevant news items of the day from various national

Page: 31 Join the dots! – March 2019

ii. Phased Manufacturing Programme (PMP) valid for 5 years till 2024 to support setting up of a few large-scale, export-competitive integrated batteries and cell-manufacturing Giga plants in India.

iii. Creation of a PMP valid for 5 years till 2024 to localize production across the entire Electric Vehicles value chain.

Know! all about National Mission on Transformative Mobility and Storage:

Composition:

• The multi-disciplinary “National Mission on Transformative Mobility and Battery Storage” with an Inter-Ministerial Steering Committee will be chaired by CEO NITI Aayog.

Role:

• The Mission will recommend and drive the strategies for transformative mobility and Phased Manufacturing Programmes for EVs, EV Components and Batteries.

• A Phased Manufacturing Program (PMP) will be launched to localize production across the entire EV value chain.

• The National Mission on Transformative Mobility and Battery Storage will determine the contours of PMP, and will finalise the details of such a program.

Top of Form

Roadmaps:

• A phased roadmap to implement battery manufacturing at Giga-scale will be considered with initial focus on large-scale module and pack assembly plants by 2019-20, followed by integrated cell manufacturing by 2021-22.

• Details of the PMP for Batteries shall be formulated by the Mission. The Mission will ensure holistic and comprehensive growth of the battery manufacturing industry in India.

Impact:

• The Mission will drive mobility solutions that will bring in significant benefits to the industry, economy and country.

• These solutions will help improve air quality in cities along with reducing India’s oil import dependence and enhance the uptake of renewable energy and storage solutions.

• The Mission will lay down the strategy and roadmap which will enable India to leverage upon its size and scale to develop a competitive domestic manufacturing ecosystem for electric mobility.

Know! the genesis of it

• During the Global Mobility Summit held in September 2018, Prime Minister had outlined the vision for the future of mobility in India based on 7 C’s which are Common, Connected, Convenient, Congestion-free, Charged, Clean and Cutting-edge mobility.

• Mobility has the potential to drive the economy forward and positively impact the lives of citizens both in urban and rural areas.

Advanced Braking Systems Made Mandatary for Vehicles

Relevance IN – Prelims (facts regarding the advance braking system)

What’s the NEWS

• The Ministry of Road Transport & Highways has decided to mandate advanced braking systems, technologies and performance requirements for improved road safety and reducing accidents.

• The provision will be binding on all vehicles with 9 seats and above.

Know! more about it

• A notification to this effect was issued. The existing vehicles will be required to adopt new provisions with effect from April 2021, while all new vehicles rolling out from April 2022 will have these pre-fitted.

Page 38: JOIN THE DOTS! - Career Launcher · dots! A current affairs series for UPSC Examination’ series which will help you pick up relevant news items of the day from various national

Join the dots! – March 2019 Page: 32

• This includes mandatory fitment of Anti lock braking system, introduction of stringent braking performance, endurance braking requirements, intelligent braking system to assist drivers in managing braking force, and electronic stability for better stability and to reduce roll over.

• With this, Indian braking regulations will be at par with European standards.

Government approves Strategic Disinvestment of 100% GOI shares in Dredging Corporation in favour of Visakhapatnam Port, Paradip Port,Deendayal Port and JNPT

Relevance IN – (Prelims – about disinvestment) + Mains (GS III economic developments

What’s the NEWS• The Government had accorded approval for Strategic Disinvestment of 100% GOI shares in DCIL in favour of

consortium of 4 ports, namely, Visakhapatnam Port Trust, Paradip Port Trust, Jawaharlal Nehru Port Trust and Deendayal Port Trust (formely known as Knadla Port Trust).

• The share purchase agreement was executed between GOI and the four ports on 8.3.2019. With this the 73.47% holding of GOI in DCI has been transferred to the four ports

• The Government has mobilized a total amount of Rs. 1049 Cr. By this strategic sale

• With this transfer of shares, the management and control of the company has been also transferred to the four Ports.

• With this transfer of the shares and management and control, of the Company, it is expected that the Company will have more operational and financial freedom in decision making which would enable the Company to take up and execute more works in an efficient way.

Know! about DCI• Dredging Corporation of India Limited (DCI) on 29th March, 1976, was incorporated as wholly owned Government

of India Undertaking with the primary objective of catering to the dredging requirements of Indian ports.

• DCI was initially incorporated as a 100% Government owned Company.

• The Company was declared a Mini Ratna – Category I Public sector enterprise in the year 1999. The Company is a Schedule B Public Sector Enterprises.

• The Registered Office of the Company is situated at New Delhi. The head office of the company is located on the east coast of India at Visakhapatnam.

The Government of India, the World Bank and the representatives of Five States sign the Loan Agreement for Additional Financing of $137 Million for the Dam Rehabilitation and Improvement Project (DRIP)

Relevance IN – Prelims (about DRIP) + Mains (GS III economic developments + environment conservation)

What’s the NEWS

• The World Bank, the Government of India and representatives of 5 States of Government of India and Implementing Agencies signed the Loan Agreement for Additional Financing of $137 Million for the Dam Rehabilitation and Improvement Project (DRIP) that will help rehabilitate and modernize over 220 selected large dams in the States of Karnataka, Kerala, Madhya Pradesh, Odisha, Tamil Nadu and Uttarakhand.

Page 39: JOIN THE DOTS! - Career Launcher · dots! A current affairs series for UPSC Examination’ series which will help you pick up relevant news items of the day from various national

Page: 33 Join the dots! – March 2019

Know! more about the agreement• In 2010, the Bank’s Board approved $350 million to finance the Dam Rehabilitation and Improvement Project

to improve the safety and sustainable performance of over 220 selected dams in India.

• This Ongoing Project has so far benefitted 25 Million Primary Beneficiaries from urban and rural communities providing them water and livelihood opportunities.

• India is home to more than 5200 large dams and another 400 that are under construction having a total storage capacity of more than 300 billion cubic meters.

• The dams play a key role in fostering rapid and sustained agricultural and rural growth and development – a key priority for the Government of India since independence.

• The World Bank has so far invested $280 million in the Project that acts as a “lighthouse”, showcasing how best to make dams fully operational and safe in a technically sound and sustainable manner.

Know! more about DRIP• Dam Rehabilitation & Improvement Project (DRIP) DRIP is a state sector scheme with central component

to improve safety and operational performance of selected dams, along with institutional strengthening with system wide management approach.

• The project was launched in 2012 by Central Water Commission (CWC) under Ministry of Water Resources, River Development & Ganga Rejuvenation with assistance from World Bank.

• Originally the scheme was scheduled for six years with closure in June 2018 with total original cost of Rs. 2100 crore with state component of Rs. 1968 crore and central component of Rs. 132 Crore

• Rehabilitation of old dams in country experiencing distress and are in need of attention for ensuring their structural safety and operational efficiency

• Strengthening institutional capacity and project management in this area.

• Bring greater awareness on dam safety issues and finding novel solutions to address them by pooling best knowledge, technologies and experience available around world

India Cooling Action Plan Launched

Relevance IN – Prelims (about the action plan) + Mains(GS III environment conservation)

What’s the NEWS• India is one of the first countries in the world to develop a comprehensive Cooling Action plan which has a long

term vision to address the cooling requirement across sectors and lists out actions which can help reduce the cooling demand.

• Cooling requirement is cross sectoral and an essential part for economic growth and is required across different sectors of the economy such as residential and commercial buildings, cold-chain, refrigeration, transport and industries

India Cooling Action Plan (ICAP)• The overarching goal of ICAP is to provide sustainable cooling and thermal comfort for all while securing

environmental and socio-economic benefits for the society.

• This will also help in reducing both direct and indirect emissions.” said Dr. Vardhan.

• India Cooling Action Plan (ICAP) provides an integrated vision towards cooling across sectors encompassing inter alia reduction of cooling demand, refrigerant transition, enhancing energy efficiency and better technology options with a 20 year time horizon.

The India Cooling Action seeks to

(i) reduce cooling demand across sectors by 20% to 25% by 2037-38,

(ii) reduce refrigerant demand by 25% to 30% by 2037-38,

(iii) Reduce cooling energy requirements by 25% to 40% by 2037-38

(iv) Recognize “cooling and related areas” as a thrust area of research under national S&T Programme,

Page 40: JOIN THE DOTS! - Career Launcher · dots! A current affairs series for UPSC Examination’ series which will help you pick up relevant news items of the day from various national

Join the dots! – March 2019 Page: 34

(v) Training and certification of 100,000 servicing sector technicians by 2022-23, synergizing with Skill India Mission.

• Cooling is also linked to human health and productivity. Linkages of cooling with Sustainable Development Goals (SDGs) are well acknowledged.

• The cross-sectoral nature of cooling and its use in development of the economy makes provision for cooling an important developmental necessity.

• The development of ICAP has been a multi-stakeholder inclusive process encompassing different Government Ministries/Departments/Organizations, Industry and Industry Associations, Think tanks, Academic and R&D institutions.

India signs $3 billion contract with Russia for lease of a nuclear submarine

Chakra III submarine

What’s the NEWS

• India and Russia have signed an agreement for the leasing of a nuclear-powered attack submarine for the Indian Navy for a period of 10 years.

Know! more about the agreement

• Russia will be delivering the Akula class submarine, to be known as Chakra III, to the Indian Navy by 2025.

• India had earlier leased two nuclear submarines from Russia. They are INS Chakra leased in 1988 under a three-year lease and second INS Chakra was taken on lease in 2012 for a period of 10 years.

• Russia will lease Akula class submarine for the period of 10 years at the cost of USD 3 billion.

• Chakra III will be equipped with Indian communication systems and sensors, including the indigenously-developed USHUS integrated sonar system and Panchendriya sonar.

• Chakra III get a unified submarine sonar and tactical control system, which are already in use on the INS Arihant.

• The Indian Navy also operates the home-built, nuclear-propelled submarine INS Arihant, which is equipped with nuclear ballistic missiles. A second nuclear submarine, INS Aririghat, will be commissioned later this year, with two more currently under construction.

• India has already commissioned French Scorpene submarine INS Kalvari and is in the process of inducting another French Scorpene submarine INS Khanderi

Page 41: JOIN THE DOTS! - Career Launcher · dots! A current affairs series for UPSC Examination’ series which will help you pick up relevant news items of the day from various national

Page: 35 Join the dots! – March 2019

SC appoints 3-member panel to mediate in Ayodhya dispute: Committee will start work in Faizabad in a week, and file status report in 4 weeks

Relevance IN - Prelims (about the judgement) + Mains (GS II structure organisation

and functioning of the executive and the judiciary ministers and departments of the

government

What’s the NEWS

• A Constitution Bench of the Supreme Court referred the Ayodhya dispute for mediation in a bid to heal minds

and hearts.

Know! all about the judgement

• The five-judge Bench, led by Chief Justice of India Ranjan Gogoi, appointed a panel of mediators, comprising

former Supreme Court judge F.M.I. Kalifulla as chairman, Art of Living founder Sri Sri Ravi Shankar,

and Sriram Panchu, a senior advocate with experience in alternative dispute resolution.

• The mediation would start in a week in Faizabad district of Uttar Pradesh — of which the disputed area is a

part — with the process conducted in-camera.

• A court-monitored mediation in the Ram Janmabhoomi-Babri Masjid land dispute case to arrive at a permanent

solution to the politically and religiously sensitive issue.

Know! all about the dispute

• The Babri Masjid-Ram Janmabhoomi case is a property dispute over the land where the Babri Masjid, a

16thcentury mosque once stood.

• The mosque was razed by kar sevaks on 6 December 1992. The Hindu groups claim that the exact site of Lord

Ram’s birthplace is where the Babri Masjid was once located.

• They argue that the Mughals demolished a Hindu shrine that marked the spot of Lord Ram’s birthplace and

constructed a mosque in its place.

• Those oppose to this view argue that such arguments arose only in the 18th century, and that there is no

evidence for the spot being the birthplace of Rama.

Page 42: JOIN THE DOTS! - Career Launcher · dots! A current affairs series for UPSC Examination’ series which will help you pick up relevant news items of the day from various national

Join the dots! – March 2019 Page: 36

India Bags First Prize at The International ‘Golden City Gate Tourism Awards 2019’

Relevance IN – Prelims(about the award)

What’s the NEWS

• The Ministry of Tourism, Government of India has won the First Prize in the category of TV Cinema Spot at the prestigious international Golden City Gate Tourism Awards 2019.

• Promotional films / television commercials produced by the Ministry as part of its Incredible India 2.0 Campaign received the awards

Know! more about the award

• The Golden City Gate Tourism Multi-media Awards are given annually in various categories related to the Tourism and Hospitality sectors.

• The ‘Golden City Gate’ is a creative multi-media international competition for countries, cities, regions and hotels.

• The entries received for the awards are judged by an international jury comprising film and tourism experts. The annual award ceremony takes place at ITB Berlin, the world’s leading tourism trade show.

Page 43: JOIN THE DOTS! - Career Launcher · dots! A current affairs series for UPSC Examination’ series which will help you pick up relevant news items of the day from various national

Page: 37 Join the dots! – March 2019

Incredible India 2.0 Campaign • The Ministry of Tourism has launched the Incredible India 2.0 Campaign in September 2017.

• The 2.0 Campaign marks a shift from generic promotions across the world to market specific promotional plans and content creation.

• Thematic creatives on different Niche tourism products have been produced and are being used in the Campaign, to cater to diverse consumer interests.

• These include the above Television Commercials on Yoga, Wellness, Wildlife, Luxury and Cuisine, which have been very well received the world over, registering about 155 million views on social media.

Only 3.32 lakh MSME jobs created in last four years, finds CII survey

Relevance IN Prelims (about the findings of the survey and about CII)

What’s the NEWS• The number of net jobs created in the Micro, Small and Medium Enterprises (MSME) sector in the last four

years stood at just 3,32,394, which is 13.9% higher than the base four years ago, according to a CII survey of more than one lakh companies.

• The findings for the four-year period — beginning 2015-16 — pale in comparison with that from the government’s Ministry of Micro, Small & Medium Enterprises for an earlier period, which shows 11,54,293 MSME jobs were created in the three years ended 2014-15.

• The survey shows just three States — Maharashtra, Gujarat, and Telangana — accounted for over 50% of the jobs created in this period (2015-16 to 2018-19).

• It also shows that 73% of the jobs were created by micro enterprises.

Know! about CII• The Confederation of Indian Industry (CII) is a business association in India.

• CII is a non-government, not-for-profit, industry-led and industry-managed organization. Founded in 1895, it has over 9,000 members, from the private as well as public sectors, including SMEs and MNCs, and an indirect membership of over 300,000 enterprises from around 265 national and regional sectoral industry bodies.

• CII works with the Government on policy issues. CII has been a catalyst of change in India’s economic policy reforms

• CII played a very important role during economic liberalisation in 1991 which knocked down the high walls of protection between Indian industry and the rest of the world.

• With 65 offices, including 9 Centres of Excellence, in India, and 11 overseas offices CII serves as a reference point for Indian industry and the international business community.

• The CII Theme for 2018-19 is ‘India RISE : Responsible. Inclusive. Sustainable. Entrepreneurial.

The Seven Croreth LPG connection under the Pradhan Mantri Ujjwala Yojana (PMUY)

Relevance IN – Prelims (about PMUY) + Mains (GS II social justice)

What’s the NEWS• The Union Minister for Petroleum and Natural Gas Dharmendra Pradhan handed over the Seven Croreth LPG

connection under the Pradhan Mantri Ujjwala Yojana (PMUY).

Know! all about PMUY• The PMUY was launched by the Prime Minister Narendra Modi on 1st May 2016 with an initial target of five

crore LPG connections, which was later revised upward to eight crores connections.

• Pradhan Mantri Ujjwala Yojana Pradhan Mantri Ujjwala Yojana (PMUY) - Under this scheme, LPG connections will be provided to BPL families with a support of Rs.1600 per connection.

• The connection is provided in the name of the adult women of the family. Further, an option is provided to provide a loan at zero interest to bear the cost of the cooking stove and first refill which has to be paid by the beneficiary.

Page 44: JOIN THE DOTS! - Career Launcher · dots! A current affairs series for UPSC Examination’ series which will help you pick up relevant news items of the day from various national

Join the dots! – March 2019 Page: 38

• 7 crore LPG connections have been distributed over a span of 34 months.

Explicitly demarcate trees grown in forests from those grown outside

Relevance IN – (Prelims – about the state forest report findings) + Mains (GS III environment conservation)

What’s the NEWS

• A high-power committee constituted by the Ministry of Environment, Forest and Climate Change (MoEFCC) has recommended that forest surveys — the biennial exercise by the government to estimate forest cover — explicitly demarcate trees grown in forests from those grown outside, that is, in plantations and private lands.

• Currently, the government counts both towards estimating the portion of India’s geographical area covered by forest.

What’s the need for this demarcation

• Independent critics have for long pointed out that including both isn’t an ecologically sound principle but this is a first instance of government-constituted committee recommending so.

• India posted a marginal 0.21% rise in the area under forest between 2015 and 2017, according to the India State of Forest Report (SFR) 2017, which was made public in February 2018.

Page 45: JOIN THE DOTS! - Career Launcher · dots! A current affairs series for UPSC Examination’ series which will help you pick up relevant news items of the day from various national

Page: 39 Join the dots! – March 2019

• The document says that India has about 7,08,273 sq. km. of forest, which is 21.53% of the geographic area of the country (32,87,569 sq. km.).

• Getting India to have at least 33% of its area under forest has been a long-standing goal of the government since 1988.

• Various editions of the SFR have over the years reported the area under forests as hovering around 21%.

• So the government also includes substantial patches of trees outside areas designated as forests, such as plantations or greenlands, in its assessment.

• The total tree cover, according to this assessment, was 93,815 sq. km. or a 2% rise from the approximately 92,500 sq. km. in 2015.

Twin Success for Guided PINAKA

Relevance IN – Prelims (about PINAKA) + Mains (GS II security challenges)

What’s the NEWS • Defence Research and Defence Organisation (DRDO) today successfully test fired the Guided PINAKA from

Pokhran ranges.

Page 46: JOIN THE DOTS! - Career Launcher · dots! A current affairs series for UPSC Examination’ series which will help you pick up relevant news items of the day from various national

Join the dots! – March 2019 Page: 40

• The weapon system is equipped with state-of-the-art guidance kit comprising of an advanced navigation and control system.

• In both the missions, the weapon systems impacted the intended targets with high precision and achieved desired accuracies.

• Telemetry Systems tracked and monitored the vehicle all through the flight path. All the mission objectives have been met.

• The indigenously developed Guided Pinaka by DRDO will significantly boost the capability of the artillery to make precision hits.

Know! more about Pinakha• Pinaka is a multiple rocket launcher produced in India and developed by the Defence Research and Development

Organisation (DRDO) for the Indian Army.

• The system has a maximum range of 40 km for Mark-I and 75 km for Mark-II, and can fire a salvo of 12 HE rockets in 44 seconds.

• The system is mounted on a Tatra truck for mobility. Pinaka saw service during the Kargil War, where it was successful in neutralising enemy positions on the mountain tops. It has since been inducted into the Indian Army in large numbers

Idukki’s Marayoor Jaggery gets GI tag

Relevance IN – Prelims (about Idukki’s Marayoor)

What’s the NEWS• The Marayoor jaggery, the traditional and handmade product from Idukki district, has finally received the

Geographical Indication (GI) tag from central government.

• Produced in Marayoor, this jaggery is made from sugarcane and no chemicals are added during the manufacturing process. Workers from Onakkallur, a village in Udumalpet in Tamil Nadu, are engaged in the production of the Marayoor jaggery.

Know! all about Marayoor jaggery • The Marayoor jaggery got the GI status after two years of effort by the Agricultural department. The GI tag will

provide more windows of opportunity to the traditional sugarcane farmers in Marayoor

• The Marayoor Jaggery largely produced in the regions of Marayur and Kanthallur grama panchayats of Kerala has got the GI tag.

Page 47: JOIN THE DOTS! - Career Launcher · dots! A current affairs series for UPSC Examination’ series which will help you pick up relevant news items of the day from various national

Page: 41 Join the dots! – March 2019

• Geographical Indication Geographical Indication (GI) is a name or sign used on products which correspond to a specific geographical location or origin.

• Geographical Indication act as a certification that the product possesses certain qualities, is made according to traditional methods or enjoys a certain reputation, due to its geographical origin.

• Marayur Jaggery Marayoor in the Idukki district of Kerala is known for its tensive sugarcane cultivation.

• In the regions of Marayur and Kanthallur, more than 2500 acres of land is under sugarcane cultivation.

• The peculiar geographical location of Marayoor amid the forests of the Western Ghats gives the sugar cane a distinct geographical identity.

• The local people have integrated the age-old tradition have imparted age-old specialized skill to make Marayur Jaggery a distinct product in itself.

• The distinct features of the Marayur Jaggery are high sweetness with less saltiness, high content of iron and less sodium. The produce is free of impurities and the sugar cane fields are free of chemical pesticides and fertilisers.

14th CII-EXIM Bank Conclave on India-Africa Project Partnerships

Relevance IN – Mains (GS II international relations)

What’s the NEWS• Ministry of Commerce & Industry will be organising the 14th CII-EXIM Bank Conclave ON India-Africa Project

Partnerships, in association with Confederation of Indian Industry and EXIM Bank of India in New Delhi from March 17-19, 2019.

Know! more about the conclave• The event will mark the deepening of India-Africa economic and business ties and pave the way for a whole

range of cross-border project partnerships.

• The annual Conclave, since its inception in 2005, brings senior Ministers, policy makers, officials, business leaders, bankers, technologists, start-up entrepreneurs and other professionals from India and Africa on a common platform in a spirit of partnership.

• More than 31senior Ministers from 21 African countries apart from business delegates from 37 countries would be participating at this event.

• The Conclave will mark the pre-eminence of India-Africa partnership in the area of ‘South-South Cooperation’, at a time when the global economy is faced with intractable challenges that stem from rising protectionism and trade conflicts.

• The India-Africa bilateral partnership is augmented by India’s ascendency as the fastest growing major economy, as well as Africa’s new economic dynamism illustrated by some of the Sub-Saharan economies which are among the top 10 fastest growing economies in the world.

• The Conclave coheres into the Indian Government’s broader vision of long-term engagement with Africa.

• The Government of India’s unflinching commitment to expanding the canvas of India-Africa economic partnership which is evident from the increase in bilateral trade between India and Africa by nearly 22% from last year touching USD 62.66 billion in the year 2017-18.

• The Conclave is expected to see the participation of 400 plus delegates from Africa and around 300 delegates from India. The B2B meetings at this Conclave are expected tobe held on more than 500 project proposals from Africa.

Vice President concludes his visit to Paraguay and Costa Rica

Relevance IN – Prelims (about the tour outcomes + Mains GS II – international relations)

What’s the NEWS• The Vice President of India, Shri M. Venkaiah Naidu visited the Republic of Paraguay from 5-7 March 2019 and

the Republic of Costa Rica from 7-9 March 2019.

• This was the first high-level visit from India to both these countries.

Page 48: JOIN THE DOTS! - Career Launcher · dots! A current affairs series for UPSC Examination’ series which will help you pick up relevant news items of the day from various national

Join the dots! – March 2019 Page: 42

Paraguay Highlights • Shri Naidu applauded Paraguay’s decision to join the International Solar Alliance soon, an initiative by India

and France, aimed at combating the threats of climate change and global warming.

• Shri Naidu urged Paraguay to facilitate the convening of the next round of expert level discussions between MERCOSUR and India to take forward the shared agenda of the expansion of the Preferential Trade Agreement.

Costa Rica highlights • The visit of Vice President to the Republic of Costa Rica was the first ever high-level visit from India to the

nation.

Honorary Doctorate from University for Peace• The Vice President was also conferred an Honorary Doctorate by the University for Peace founded by the United

Nations for his contribution “to the Rule of Law, democracy and sustainable development in India”.

• The university has stated that the degree of “Doctor Honoris Causa”(Doctor of Philosophy) was conferred on the Indian Vice President recognising his contribution to the Rule of Law, democracy and sustainable development in India.

• Shri. Venkaiah Naidu is the first Indian to receive an honorary doctorate from the University of Peace.

Know! about University of Peace• The University of Peace was established in accordance with the Resolution 35/55 passed by the United Nations

General Assembly in 1980.

• The main campus of the University of Peace is located in Costa Rica, a country of Central America.

• The University of Peace aims to be a forward-thinking, transformational and inspirational educational institution dedicated to the goals of quality teaching, research and service for serving humanity in building a peaceful world.

• The stated mission of the University of Peace is to provide humanity with an international institution of higher education for peace and with the aim of promoting, among all human beings

Know! about MERCOSUR• Mercosur, officially Southern Common Market is a South American trade bloc established by the Treaty of

Asunción in 1991 and Protocol of Ouro Preto in 1994.

• Its full members are Argentina, Brazil, Paraguay and Uruguay. Venezuela is a full member but has been suspended since December 1, 2016.

• Associate countries are Bolivia, Chile, Colombia, Ecuador, Guyana, Peru and Suriname. Observer countries are New Zealand and Mexico.

• Mercosur’s purpose is to promote free trade and the fluid movement of goods, people, and currency. It currently confines itself to a customs union, in which there is free intra-zone trade and a common trade policy between member countries.

Pulse Polio Programme 2019

Relevance IN – Prelims(about polio and other similar initiatives by the government)

What’s the NEWS• In order to sustain polio eradication drive from the

country, government on Saturday launched Pulse Polio programme for 2019.

Know! all about the programme • President of India Ram Nath Kovind, launched the

program by administering polio drops to children less than five years old, at the Rahstrapati Bhawan in New Delhi.

Page 49: JOIN THE DOTS! - Career Launcher · dots! A current affairs series for UPSC Examination’ series which will help you pick up relevant news items of the day from various national

Page: 43 Join the dots! – March 2019

• More than 17 crore children of less than five years across the country will be given polio drops as part of the drive.

• Universal Immunization Programme is focusing to protect children from more diseases than ever before and has introduced several new vaccines like Pneumococcal Conjugate Vaccine, Rotavirus vaccine, and Measles-Rubella vaccine in the recent past

• To provide additional protection to our children, Government has also introduced the injectable Inactivated Polio Vaccine into its routine immunization program

• Along with Universal Immunization Programme of the country, the government has also launched Mission Indradhanush to accelerate the goal to achieve more than 90% full immunization coverage.

• More than 3.39 crore children and 87 lakh pregnant women have been vaccinated through Mission Indradhanush drives

• The government claimed that strengthening of immunization programme has contributed significantly to the decline of Infant Mortality Rate from 39 in 2014 to 32 per 1000 live births in 2017.

• The polio eradication programme in India aims to protect children from the crippling disease by conducting two nationwide mass polio vaccination campaigns and two to three sub-national campaigns each year.

• The last reported cases of wild polio in India were in West Bengal and Gujarat on 13 January 2011.

• On 27 March 2014, the World Health Organization (WHO) declared India a polio free country.

Know! about Polio • Polio also known as poliomyelitis is a highly contagious viral disease caused due to the attacks the nervous

system and children younger than 5 years old are more likely to contract the virus than any other group.

• Poliovirus usually spreads from person to person through infected faecal matter entering the mouth.

• Poliovirus also spreads by food or water containing human faeces and less commonly from infected saliva

Arecanut gets ‘Sirsi supari’ geographical indication tag

Relevance IN – Prelims(about sirsi supari)

What’s the NEWS• Arecanut (also known as supari or betel nut) is the latest entrant in the list of agricultural produce with a GI

(geographical indication) tag

Know! more about Sirsi Supari • On March 4, the Registrar of Geographical Indications, Government of India, accorded a GI tag, ‘Sirsi Supari’,

to arecanut grown in Sirsi, Siddpaur and Yellapur taluks of Uttara Kannada district in Karnataka.

• It is cultivated in Yellapura, Siddapura and Sirsi taluks. Totgars’ Cooperative Sale Society Ltd., Sirsi, is the registered proprietor of the GI.

Page 50: JOIN THE DOTS! - Career Launcher · dots! A current affairs series for UPSC Examination’ series which will help you pick up relevant news items of the day from various national

Join the dots! – March 2019 Page: 44

• The Registrar of Geographical Indications, under the Union government, Chennai, issued the certificate to the society on March 4, 2019.

• According to it, the particular arecanut “is medium in size, somewhat flat and rounded in shape, somewhat ash coloured, and has a hard seed.

• The arecanut grown in these taluks have unique features like a round and flattened coin shape, particular texture, size, cross-sectional views, taste, etc.

• These features are not seen in arecanut grown in any other regions.

• The World Intellectual Property Organisation defines GI as a sign used on products that have a specific geographical origin and possess qualities or a reputation that are due to that origin.

Wood snake, last seen in 1878, rediscovered by scientists

The species is endemic to the Meghamalai forests and Periyar Tiger Reserve area

What’s the NEWS • A species of wood snake that wasn’t seen for 140 years has resurfaced in a survey conducted by scientists in the

Meghamalai Wildlife Sanctuary.

Know! more about this species• The species, endemic to the Meghamalai forests and the Periyar Tiger Reserve landscape, was recently

rediscovered

• The findings of the surveys, conducted over two years (2014-2016), were published in the Journal of the Bombay Natural History Society last month.

• The snake is a ‘point endemic’ (found only in Meghamalai). It was found in the same region that Colonel Beddome alluded to, and the morphological characters match with his specimen

• The local population of wood snakes was last spotted and recorded by British military officer and naturalist Colonel Richard Henry Beddome in 1878

• The rediscovery of the snake indicated that the quality of the habitat was good.

SC may send plea challenging quota Bill to Constitution Bench

Relevance IN – Prelims (about EWS provisions) + Mains (GS II role of judiciary + social issues)

What’s the NEWS• The Constitution(103rd) Amendment Act introducing Articles 15(6) and 16(6) in the Constitution to provide for

economic reservation does not affect the basis structure of the Constitution, said the Centre in its affidavit filed in Supreme Court countering the petitions challenging economic reservation.

• The Supreme Court decided to consider the question of whether the challenge to the 10% economic reservation law should be heard by a Constitution Bench.

Page 51: JOIN THE DOTS! - Career Launcher · dots! A current affairs series for UPSC Examination’ series which will help you pick up relevant news items of the day from various national

Page: 45 Join the dots! – March 2019

• A three-judge Bench, led by Chief Justice of India Ranjan Gogoi, scheduled the hearing for March 28.

• The court, however, refused to pass any interim order to stay or hamper the implementation of the Constitution (103rd Amendment) Act that provides for 10% reservation in government jobs and educational institutions to the economically backward in the unreserved category.

What’s the issue

• The issue arose when senior advocate Rajeev Dhavan pointed out that the 50% quota limit was part of the Basic Structure of the Constitution, and the new amendment tinkered with it.

• The Act amends Articles 15 and 16 of the Constitution, adding clauses empowering the government to provide reservation on the basis of economic backwardness

• The Act violated the basic features of the Constitution. The petitioners argued that the 50% ceiling was “engrafted as a part of the Basic Structure of the Constitution’s equality code” by the court.

• The challenge on the ground that EWS quota will breach 50% limit of reservation is rebutted by stating that this limit imposed by Supreme Court in Indira Sawhney case is not applicable after constitution amendment.

Know! all about Indira Sawhney case

• In the famous Mandal case (Indra Sawhney v. Union of India 1992), the scope and extent of Article 16(4), which provides for reservation of jobs in favour of backward classes, has been examined thoroughly by the Supreme Court.

• Though the Court has rejected the additional reservation of 10% for poorer sections of higher castes, it upheld the constitutional validity of 27% reservation for the OBCs with certain conditions

- The advanced sections among the OBCs (the creamy layer) should be excluded from the list of beneficiaries of reservation.

- No reservation in promotions; reservation should be confined to initial appointments only. Any existing reservation in promotions can continue for five years only (i.e., upto 1997).

- The total reserved quota should not exceed 50% except in some extraordinary situations. This rule should be applied every year.

- The ‘carry forward rule’ in case of unfilled (backlog) vacancies is valid. But it should not violate 50% rule.

- A permanent statutory body should be established to examine complaints of over-inclusion and under-inclusion in the list of OBCs.

Page 52: JOIN THE DOTS! - Career Launcher · dots! A current affairs series for UPSC Examination’ series which will help you pick up relevant news items of the day from various national

Join the dots! – March 2019 Page: 46

India is world’s 2nd largest arms importer: Figures decreased by 24% between 2009-13 and 2014-18, partly due to delays in delivery

Relevance IN – Prelims (about SIPRI report) + Mains (GS III international organisations)

What’s the NEWS• India was the world’s second largest arms importer from 2014-18, ceding the long-held tag as largest importer

to Saudi Arabia, which accounted for 12% of the total imports during the period.

• India was the world’s second largest importer of major arms in 2014–18 and accounted for 9.5% of the global total,” according to the latest report published by the Stockholm International Peace Research Institute (SIPRI)

Know! more about the SIPRI report• Indian imports decreased by 24% between 2009-13 and 2014-18, partly due to delays in deliveries of arms

produced under licence from foreign suppliers, such as combat aircraft ordered from Russia in 2001 and submarines ordered from France in 2008, the report stated.

• Russia accounted for 58% of Indian arms imports in 2014–18, compared with 76% in 2009-13.

• Israel, the U.S. and France all increased their arms exports to India in 2014-18.

• However, the Russian share in Indian imports is likely to sharply go up for the next five-year period as India signed several big-ticket deals recently, and more are in the pipeline.

• These include S-400 air defence systems, four stealth frigates, AK-203 assault rifles, a second nuclear attack submarine on lease, and deals for Kamov-226T utility helicopters, Mi-17 helicopters and short-range air defence systems.

• The report noted that despite the long-standing conflict between India and Pakistan, arms imports decreased for both countries in 2014-18 compared with 2009-13.

• Pakistan stood at the 11th position accounting for 2.7% of all global imports.

• Its biggest source was China, from which 70% of arms were sourced, followed by the U.S. at 8.9% and, interestingly, Russia at 6%.

• The five largest exporters in 2014-18 were the United States, Russia, France, Germany and China together accounting for 75% of the total volume of arms exports in 2014-18.

• The flow of arms increased to the Middle East between 2009-13 and 2014-18, while there was a decrease in flows to all other regions

• China, which has emerged as a major arms exporter, has increased its share by 2.7% for 2014-18 compared to

Page 53: JOIN THE DOTS! - Career Launcher · dots! A current affairs series for UPSC Examination’ series which will help you pick up relevant news items of the day from various national

Page: 47 Join the dots! – March 2019

2009-13. Its biggest customers are Pakistan and Bangladesh.

Know! about SIPRI

• SIPRI is an independent international institute dedicated to research into conflict, armaments, arms control and disarmament.

• Established in 1966, SIPRI provides data, analysis and recommendations, based on open sources, to policymakers, researchers, media and the interested public.

• Based in Stockholm, SIPRI is regularly ranked among the most respected think tanks worldwide.

Vision and mission

• SIPRI’s vision is a world in which sources of insecurity are identified and understood, conflicts are prevented or resolved, and peace is sustained.

SIPRI’s mission is to:

• undertake research and activities on security, conflict and peace;

• provide policy analysis and recommendations;

• facilitate dialogue and build capacities;

• promote transparency and accountability; and

• deliver authoritative information to global audiences.

The DRDO Develops ‘combat drugs’ to reduce casualties in Pulwama type attacks, warfare

Relevance IN – Prelims (about the combat drugs) + Mains (GS III security challenges and internal security)

Highlights

• The spectrum includes bleeding wound sealants, super absorptive dressings and glycerated salines, all of which can save lives in the event of warfare in jungle and high altitude areas as well as in terror attacks

What’s the NEWS

• With 90 per cent of gravely wounded security personnel succumbing to injuries within a few hours, DRDO’s medical laboratory has come up with a range of ‘combat casualty drugs’ that can extend the golden hour till the trooper is shifted to hospital.

• This range of drugs can bring down the death toll during instances like the recent Pulwama Terrorist Attack.

• According to developers of the drugs at the Institute of Nuclear Medicine and Allied Sciences, a laboratory of the Defence Research and Development Organisation, chances of survival and minimum disability are highest when effective first aid care is given within the golden hour.

Know! about theses combat casualty drugs

• Among the drugs developed is glycerated saline, a battlefield intravenous fluid that does not freeze till -18 degrees Celsius and is useful in handling trauma cases in high altitude areas.

• Glycerated saline, unlike normal saline, reduces inflammation. The drug can be life saving, particularly if the traumatic edema, collection of fluid in tissues and cavities of the body, is in the brain or lungs.

• Glycerated saline has life-saving capacities as it gives more time to the medical personnel to shift the wounded patient to a higher care facility.

• INMAS has also developed a special medicated dressing material which is 200 times more absorptive than normal dressings during bleeding wounds.

• This range of casualty drugs are aimed at enhancing the chances of survival and minimum disability by providing effective first aid care is given within the golden hour

Page 54: JOIN THE DOTS! - Career Launcher · dots! A current affairs series for UPSC Examination’ series which will help you pick up relevant news items of the day from various national

Join the dots! – March 2019 Page: 48

PM Modi, Bangladesh PM Sheikh Hasina, jointly unveil e-plaques for development projects in Bangladesh

Relevance IN - Prelims(about various launched projects) + Mains (GS II bilateral relations)

What’s the NEWS• Prime Minister Narendra Modi, and Bangladesh Prime Minister Sheikh Hasina, jointly unveiled e-plaques for

development projects in Bangladesh, through video conference.

Know! more about the launched development projects• Both leaders unveiled e-plaques for supply of buses and trucks, inauguration of 36 community clinics,

11 water treatment plants, and extension of National Knowledge Network to Bangladesh.

• These unveilings will boost not just transport connectivity, but also knowledge connectivity.

• The National Knowledge Network will connect scholars and research institutes in Bangladesh, to India and the world.

• The buses and trucks would assist the efforts towards affordable public transportation; water treatment plants will help supply clean water; and community clinics will benefit about 2 lakh people, in Bangladesh.

Know! more about National Knowledge Network • National Knowledge Network (NKN) is a state-of-the art pan-Indian resource sharing network aimed at

digitally connecting all national universities, colleges and research establishments to create country-wide virtual classrooms.

• National Knowledge Network project was initiated in 2009 for a period of 10 years. At present, the National Knowledge Network programme is a component of the umbrella “Digital India” programme.

• NKN facilitates advanced distance education in specialized fields like engineering, science, medicine etc. as well as enable an ultra-high speed e-Governance backbone.

INDO-OMAN Joint Ex Al Nagah 2019

Relevance IN – Prelims (about the exercise)

What’s the NEWS• Indo Oman Joint Exercise Al Nagah III 2019, a joint military exercise between Indian and Royal Army of Oman

(RAO), commenced at HQ Jabel Regiment, Nizwa, Oman

Know! more about the exercise• The Omani contingent was represented by Jabel Regiment of the RAO while the Indian side was represented by

troops of Tenth Battalion The Garhwal Rifles Regiment.

• The Indian Army and RAO contingents have been specifically selected for the exercise based on expertise and

Page 55: JOIN THE DOTS! - Career Launcher · dots! A current affairs series for UPSC Examination’ series which will help you pick up relevant news items of the day from various national

Page: 49 Join the dots! – March 2019

professional competence and will take part in the two-week-long event that will see them hone their tactical and technical skills in joint counter insurgency and counter terrorist operations in semi-urban scenario in mountainous terrain under UN mandate.

• Both sides will jointly train, plan and execute a series of well-developed tactical drills for neutralization of likely threats that may be encountered in such a scenario.

• Ex Al Nagah 2019 will contribute immensely in developing mutual understanding and respect for each other’s military as also facilitate in tackling the worldwide phenomenon of terrorism.

Cabinet approves proposal for accession of India to The Nice Agreement The Vienna agreement and The Locarno Agreement

Relevance IN – Prelims (about these accession) + Mains (GS III awareness in the field of IPR)

What’s the NEWSThe Union Cabinet has approved the proposal for accession of India to

(i) The Nice Agreement concerning the International classification of Goods and Services for the purposes of registration of marks

(ii) The Vienna Agreement establishing an International Classification of the figurative elements of marks, and

(iii) The Locarno Agreement establishing an International classification for industrial designs.

Know! the benefits of these accessions • Accession to the Nice, Vienna and Locarno Agreements will help the Intellectual Property Office in India to

harmonise the classification systems for examinational of trademark and design applications, in line with the classification systems followed globally.

• It would give an opportunity to include Indian designs, figurative elements and goods in the international classification systems.

• The accession is expected to instill confidence in foreign investors in relation to protection of IPs in India.

• The accession would also facilitate in exercising rights in decision making processes regarding review and revision of the classifications under the agreement.

No names removed based on draft NRC: Election Commission

Relevance IN – Prelims (about NRC developments) + Mains (GS II internal security + GS III structure, organisation and functioning of the judiciary)

What’s the NEWS• The Election Commission of India (ECI) assured the Supreme Court that names have not been deleted from

the Assam electoral roll on the basis of their exclusion from the draft National Register of Citizens (NRC), which was published in July last year.

Know! more about the developments• A question looms large in the light of the preparation of the final NRC, to be published on July 31, 2019. What

would happen to those who do not appear there but are included in the electoral roll?

• The court further directed the ECI to furnish details of the names included in the electoral roll on January 1, 2017, 2018 and 2019 in Assam.

• The Election Commission of India (ECI) has stated before the Supreme Court that exclusion of names of persons from draft National Registry of Citizens (NRC) in Assam will not affect their voting rights in the upcoming Lok Sabha polls provided their names feature in the electoral rolls.

Know! about the petition • Petition before the Supreme Court Petition led by Gopal Seth and Susanta Sen, residents of Assam feared that

people would lose their voting rights due to the ongoing exercise of National Registry of Citizens.

Page 56: JOIN THE DOTS! - Career Launcher · dots! A current affairs series for UPSC Examination’ series which will help you pick up relevant news items of the day from various national

Join the dots! – March 2019 Page: 50

• The petition highlighted five categories of people: Persons whose names were deleted from the voter list which includes names of those appeared in the draft NRC published on July 30, 2018.

• People whose names were not included in the complete draft NRC, but they subsequently led claims for inclusion of their names in it.

• People who have been declared as foreigners by the foreigners’ tribunal as well as the Guwahati High Court.

• The order which has been stayed by the apex court. People who had already been declared foreigners by the foreigners’ tribunal and such declarations were set aside by the apex court.

• People whose names have not been included in the draft NRC, but other members of their families, including parents, have been included in the NRC and they have led their claims for the inclusion of their names.

• The Supreme Court asked the ECI to provide data about the addition and deletion of names from voters’ list as revised in January for 2017, 2018 and 2019. The case for posted for further hearing on March 28

Data for the Index of Industrial Production (IIP)

Relevance IN – Prelims (about IIP) + Mains (GS III economic developments)

What’s the NEWS

• The Central Statistics Office (CSO) has released the data for the Index of Industrial Production (IIP) for the month of January.

• The IIP data shows that: Industrial output growth stood at 1.7 per cent in January against the 2.6% growth recorded in December 2018.

• The CSO has revised the industrial production growth for December 2018 has been revised upwards from 2.4% to 2.6%.

• The growth of output of manufacturing sector moderated to 1.3% and the electricity generation rose a mere 0.8% in January 2019.

• The mining output rebounded 3.9% in January 2019, snapping 1% decline in December 2018.

• Capital goods output declined 3.2% in January 2019 and the output of intermediate goods also fell by 3.0% in January 2019.

• The output of primary goods increased by 1.4%, while that of infrastructure/ construction goods moved up 7.9% in January 2019 when compared to January 2018.

• The output of consumer durables moved up 1.8%, while that of consumer non-durable durables also rose 3.8% in January 2019 when compared to January 2018.

• Eleven out of the twenty-three industry groups in the manufacturing sector have shown positive growth during the month of January 2019 as compared to January 2018.

• The electricity generation output growth also improved to 5.8%, while mining output growth accelerated to 5.3% in April-January FY2019.

Know! about IIP • Index of Industrial Production Index of Industrial Production (IIP) is a composite indicator that measures the

changes in the volume of production of a basket of industrial products during a given period with respect to the volume of production in a chosen base period. The base year for the IIP is 2011- 12

Page 57: JOIN THE DOTS! - Career Launcher · dots! A current affairs series for UPSC Examination’ series which will help you pick up relevant news items of the day from various national

Page: 51 Join the dots! – March 2019

Retail inflation rises to 4-month high of 2.57% in February 2019

The Central Statistical Office (CSO) has released the data of Inflation for the month of February 2019.

What’s the NEWS• The retail inflation based on Consumer Price Index (CPI) stood at 1.97 per cent in January and 4.44 per cent in

February 2018

• Retail inflation was at a four month high of 2.57 per cent in February 2019 due to costlier food articles.

Know! more about the CSO findings • The retail inflation was highest since October 2018 when it stood at 3.38 per cent.

• Consumer Price Index-based inflation for January 2019 was revised down to a 19-month low of 1.97 per cent from an earlier estimate of 2.05 per cent.

• Food inflation at (-) 0.66 per cent in February 2019 was lower against 3.26 per cent in the same month last year.

• The consumer food price index increased by 0.15 per cent in February against January 2019.

• With the headline inflation reading at 2.57 per cent and industrial production growth on the downside at 1.7 per cent, economists see a case and space for one more rate cut of 25 bps by RBI in April to support growth.

World Gold Council Report

Relevance IN – Prelims (about GCR findings)

Know! about the findings of the report• India has the 11th largest gold reserve and the current holding pegged at 607 tonnes.

• India’s would have been at the tenth position had the list included only countries.

• International Monetary Fund (IMF) ranks third on the list with total gold reserves of 2,814 tonnes.

• The top spot is occupied by US gold reserves of 8,133.5 tonnes, followed by Germany with 3,369.7 tonnes.

• The third and fourth slot is occupied by Germany and France with reserves of around 2,400 tonnes each.

• China and Japan have more reserves of the precious metal when compared to India.

• The report notes that Gross purchases of 48 tonnes and gross sales of 13 tonnes led to an increase in global gold reserves by 35 tonnes on a net basis in January, with sizeable increases from nine central banks.

Know! about WGC

• The World Gold Council is the market development organisation for the gold industry and it aims to stimulate and sustain demand for gold, provide industry leadership, and be the global authority on the gold market.

• The members of the World Gold Council include gold mining companies

Page 58: JOIN THE DOTS! - Career Launcher · dots! A current affairs series for UPSC Examination’ series which will help you pick up relevant news items of the day from various national

Join the dots! – March 2019 Page: 52

Twin Success for Man Portable Anti Tank Guided Missile

Relevance IN – Prelims (about MPATGM)

What’s the NEWS• In a major boost for Army, Defence Research and Development Organisation (DRDO) successfully test fired

indigenously developed, low weight, fire and forget Man Portable Anti-Tank Guided Missile (MPATGM) for the second time in the ranges of Rajasthan desert.

Know! about the successful trial of MPATGM• MPATGM is incorporated with advanced features including state-of-the-art Imaging Infrared Radar (IIR)

Seeker with integrated avionics.

• The first test was conducted on 13th March 2019. In both the missions, the missiles hit the designated targets precisely at different ranges. All the mission objectives have been met

• MPATGM incorporates advanced features, including, image infrared radar seeker with integrated avionics

• Indian defence scientists have scored a twin success in demonstrating the effectiveness of the Man Portable Anti-tank Guided Missile (MPATGM).

• This follows the hat trick of sharp tests carried out on the short range guided Pinaka weapon system early in the week.

• Both these weapons, developed indigenously by scientists of the Defence Research and Development Organisation (DRDO), were tested in the deserts of Pokhran in Rajasthan.

• They enhance the hit and destruction capacity of the armed forces in short distance battle situations considerably.

• The MPATGM missile incorporates many advanced features, including, image infrared radar (IIR) seeker with integrated avionics.

Know! more about MANPATS• Man-portable anti-tank systems (MANPATS or MPATS) are shoulder-launched anti-tank rockets.

• They are typically unguided weapons and are a threat to armored vehicles, low-flying aircraft (especially helicopters), and field fortifications.

• Generally MANPATS fall into three distinct categories. The first consist of a small, disposable preloaded launch tube firing a high explosive anti-tank warhead operated by a single soldier.

• The second is a firing system onto /into which a rocket is loaded, operated by a single soldier.

• The third are manufactured prepacked and issued as a single unit of ammunition with the launcher discarded after a single use.

Page 59: JOIN THE DOTS! - Career Launcher · dots! A current affairs series for UPSC Examination’ series which will help you pick up relevant news items of the day from various national

Page: 53 Join the dots! – March 2019

Indo-Bangladesh joint Military Exercise Sampriti – 2019

Relevance IN – Prelims(about Sampriti) + Mains (GS II bilateral relations)

What’s the NEWS• Exercise Sampriti-VIII, a joint Indo-Bangladesh military exercise which witnessed participation of a company

group of 9th Battalion the Rajputana Rifles from the Indian Army and the Company of 36 East Bengal Battalion, Bangladesh Army concluded at Tangail, Bangladesh

Know! more about the exercise• The exercise was the 8th edition in the Sampriti series, which started in 2009.

• Exercise Sampriti strengthens and broadens interoperability and cooperation between the Indian and Bangladesh Armies.

• It is the fourth Indo-Bangladesh exercise at Tangail, Bangladesh and compliments number of other exchanges and exercise between the two forces.

• The commanders and staff officers of both sides were exercised to work in close coordination to receive and collate intelligence and to issue suitable operational orders to the joint field training components who executed these orders on ground in simulated realistic situations.

• The joint training culminated in a validation exercise conducted at Bangabandu Senanibas Cantonment, Tangail in which subunits of both armies executed the plans.

First Workshop on India Energy Modelling Forum Held

Relevance IN - Mains (GS III energy conservation and energy security)

What’s the NEWS• The NITI Aayog and the United States Agency for International Development (USAID) organized the first

workshop on development of the India Energy Modelling Forum (IEMF), which has been envisaged as a pan-stakeholder platform for debating ideas, scenario-planning & discussing the India’s energy future.

• The two-day workshop, being held with the support of the Pacific Northwest National Laboratory (PNNL), was organized under the Sustainable Growth Pillar of the India-U.S. Strategic Energy Partnership.

Know! more about the forum India Energy Modelling Forum (IEMF)• The IEMF seeks to provide a platform for leading experts and policy makers to study important energy and

environmental issues and ensure induction of modelling and analysis in informed decision making process.

Page 60: JOIN THE DOTS! - Career Launcher · dots! A current affairs series for UPSC Examination’ series which will help you pick up relevant news items of the day from various national

Join the dots! – March 2019 Page: 54

• The Forum aims to improve cooperation and coordination between modeling teams, the Government of India, knowledge partners and think-tanks, build capacity of Indian institutions, and identify issues for joint modeling activities and future areas of research.

• The panelists particularly laid focus on bridging the rural-urban divide and factoring in energy pressures from the informal economy within models.

• There was a need expressed to ensure that holistic perspective of energy consumption and ground realities must be inducted to produce practical and feasible energy models, converging land and water use patterns within energy models.

• The shift towards electric mobility, an increasing emphasis on mainstreaming of renewable energy options and overarching environmental concerns were also stated as key factors for determining India’s energy future.

• The workshop included extensive discussions about the framework of an India Energy Modelling Forum and its institutional, coordination and funding mechanisms.

Health Ministry takes stock of the public health measures for controlling West Nile Virus

Relevance IN – Prelims (about WNV)

What’s the NEWS• A section of the media has reported that a seven year old boy from Malappuram District of Kerala is suffering from

a West Nile Virus (WNV), a mosquito-borne disease, mostly reported in the continental United States.

Know! all about WNV• West Nile virus (WNV) is the leading cause of mosquito-borne disease in the continental United States.

• It is most commonly spread to people by the bite of an infected mosquito. Cases of WNV occur during mosquito season, which starts in the summer and continues through fall.

• There are no vaccines to prevent or medications to treat WNV in people. Fortunately, most people infected with WNV do not feel sick.

• About 1 in 5 people who are infected develop a fever and other symptoms.

• About 1 out of 150 infected people develop a serious, sometimes fatal, illness. You can reduce your risk of WNV by using insect repellent and wearing long-sleeved shirts and long pants to prevent mosquito bites.

China places hold on listing Azhar as designated terrorist

Relevance IN – Prelims (about UNSC role) + Mains (GS II international organisations + GS III security challenges)

What’s the NEWS• China has again blocked the bid to designate Pakistan-based terror group Jaish-e-Mohammed Chief Azhar

Masood as a global terrorist in the United Nations Security Council 1267 list.

Know! more about the proposed bid• The proposal was moved by France the UK and the US on February 27, on the backdrop of Pulwama Terror

Attack.

• Just before the deadline of the objections was about to end, China which is the permanent member of the United Nations Permanent Council led the objection.

• Implications of the listing by the 1267 Sanctions committee

• The 1267 and Al-Qaeda Sanctions Committee was established under the UNSC resolution 1267.

The listing would have resulted in:• Assets freeze The Asset Freeze mandates all states to freeze without delay the funds and other financial assets

or economic resources of designated individuals and entities.

• Travel ban The travel ban implies preventing the entry of designated individuals into or transit by all states through their territories

Page 61: JOIN THE DOTS! - Career Launcher · dots! A current affairs series for UPSC Examination’ series which will help you pick up relevant news items of the day from various national

Page: 55 Join the dots! – March 2019

• Arms embargo Arms embargo requires all states to prevent the direct or indirect supply, sale and transfer from their territories or by their nationals outside their territories, or using their ag vessels or aircraft, of arms and related material of all types, spare parts, and technical advice, assistance, or training related to military activities, to designated individuals and entities.

Festival of Innovation & Entrepreneurship (FINE)

Relevance IN – Prelims (about FINE 2019) + Mains (GS III economic development + GS II government policy and interventions)

What’s the NEWS• The President of India, Shri Ram Nath Kovind, inaugurated the Festival of Innovation and Entrepreneurship

in Gandhinagar, Gujarat . He also presented the 10th Biennial National Grassroots Innovation Awards.

• The Festival of Innovation & Entrepreneurship (FINE) {previously known as Festival of Innovation FOIN)} is a unique initiative of the Office of the President of India to recognise, respect and reward grassroots innovations and foster a supportive ecosystem.

Know! all BOUT fine 2019• Hosted in the month of March at The President’s House, the FOIN has become a national celebration of creativity

and innovation at and for grassroots.

• This year it has been decided to organize Festival of Innovation and Entrepreneurship (FINE) from March 15-17, 2018

• This Festival is a celebration of creativity, innovation and entrepreneurship. Till 2018, it had been hosted at Rashtrapati Bhavan.

• This year it was decided to organise it outside Rashtrapati Bhavan by the President’s Secretariat in association with National Innovation Foundation-India and Department of Science & Technology.

• FINE is a celebration of country’s Innovation potential, particularly those ideas which stem from grassroots level including the citizen at the last mile and also a reflection of power of children’s creativity.

• FINE would provide platform to the innovators for building the linkages with potential stakeholders whose support can improve their prospects in coming years for the larger social good.

• It will also help in promoting lateral learning and linkages among the innovators to enrich the ecosystem for new India.

• It would also be a great opportunity to create awareness about the importance of various Ministries of Government of India attaches to their effort and participation in the FINE.

Page 62: JOIN THE DOTS! - Career Launcher · dots! A current affairs series for UPSC Examination’ series which will help you pick up relevant news items of the day from various national

Join the dots! – March 2019 Page: 56

• It is imperative that India becomes a growth engine for the world and provides a new model of inclusive development by providing a large number of open technological and other solutions for the developing and developed world.

• In sync with the policies of the government of India, FINE will provide a window to the creative and innovative solutions for social development through grassroots innovations, student ideas and other technologies for agriculture, rural development, sanitation, health, women and child development, biotechnology and medical innovation for grassroots.

A climate vulnerability index for India on the anvil

Relevance IN – Prelims (about the vulnerability index) + Mains (GS III environment conservation)

What’s the NEWS• The Department of Science and Technology (DST) will be commissioning a study to assess the climate risks

faced by States in India.

• This follows an assessment of the global warming risks faced by 12 Himalayan States — and discussed at last year’s U.N. climate change conference in Poland — that found States such as Assam, Arunachal Pradesh and Uttarakhand vulnerable to climate change.

• DST also planning to have a climate portal, whereby users can zoom in on any district in the country and get a sense of what kind of risks — climate, socio-economic — are present

Know! about the Common methodology and about the index outcome• Last year the Indian Institutes of Technology (IIT) at Mandi and Guwahati, and the Indian Institute of Science

(IISc), Bengaluru, coordinated with State authorities in Assam, Manipur, Meghalaya, Mizoram, Nagaland, Tripura, Arunachal Pradesh, Sikkim, the hill districts of West Bengal, Himachal Pradesh, Uttarakhand and Jammu and Kashmir, to evolve a common methodology, and determine how districts there are equipped to deal with the vagaries of climate change

• The researchers prepared a ‘vulnerability index’ of each of these States based on district-level data.

• Vulnerability would be a measure of the inherent risks a district faces, primarily by virtue of its geography and socio-economic situation.

Vulnerability score bases on eight parameters • The scientists conducted workshops with the States and culled eight key parameters on the basis of which a

vulnerability score could be generated.

• They included: percentage of area in districts under forests, yield variability of food grain, population density, female literacy rate, infant mortality rate, percentage of population below poverty line, average man-days under MGNREGA (Mahatma Gandhi National Rural Employment Guarantee Act), and the area under slope > 30%.

Page 63: JOIN THE DOTS! - Career Launcher · dots! A current affairs series for UPSC Examination’ series which will help you pick up relevant news items of the day from various national

Page: 57 Join the dots! – March 2019

• On a scale ranging 0-1, 1 indicating the highest possible level of vulnerability, at the top of the scale were Assam with a score of 0.72 and Mizoram at 0.71, whereas Sikkim, with an index score of 0.42 was relatively less vulnerable.

India-Africa joint field training exercise to kick off on March 18: AFINDEX -2019

Relevance IN – Prelims (about AFINDEX -19) + Mains (GS II international relations)

What’s the NEWS

• A 10-day long Africa-India Joint Field Training Exercise (AFINDEX-19) between the Indian Army and 16 African nations will be conducted in Pune from March 18 to 27.

• As many as 10 personnel, each from the participating nations from the African continent and personnel of Maratha Light Infantry of Indian Army, will participate in the joint exercise which will be conducted at two locations in Pune’s Foreign Training Node at Aundh Military Station and College of Military Engineering in Kirkee.

Know! all about AFINDEX-19

• AFINDEX-19 aims to train the participating contingents in Humanitarian Mine Assistance (HMA) and Peace Keeping Operations (PKO) under the United Nations Charter through practical and comprehensive discussions and tactical exercises Contingents from Egypt, Ghana, Nigeria, Senegal, Sudan, South Africa, Tanzania, Namibia, Mozambique, Uganda, Niger & Zambia are part of the joint exercise together with officers from Rwanda, Democratic Republic of Congo and Madagascar as Observers.

• The joint exercise will also focus on achieving interoperability, learning each other’s methodologies and tactics through synchronised operational level planning and tactical level training.

Global Environment Outlook (GEO)What’s the NEWS

• The sixth edition of the Global Environmental Outlook (GEO), prepared by the United Nations Environment Programme.

• The Global Environment Outlook (GEO) is the UN Environment Programme’s (UNEP) flagship environmental assessment.

Know! about the findings of the Report

• The report warns that deadly emissions, chemicals polluting drinking water, and the accelerating destruction of ecosystems crucial to the livelihoods of billions of people are driving a worldwide epidemic that hampers the global economy.

• The report highlights the growing divide between rich and poor as rampant overconsumption, pollution and food waste in the developed world leads to hunger, poverty and disease elsewhere.

• The report notes that as greenhouse gas emissions continue to rise amid a preponderance of droughts, floods and superstorms made worse by climbing sea levels, there is a growing political consensus that climate change poses a future risk to billions.

• The report expresses concern that the health impacts of pollution, deforestation and the mechanised food chain are less well understood.

• The report notes that poor environmental conditions cause approximately 25% of global disease and mortality and resulted in around 9 million deaths in 2015 alone.

• Due to lack of access to clean drinking supplies, 1.4 million people die each year from preventable diseases such as diarrhoea and parasites linked to pathogen riddled water and poor sanitation.

• The report notes that chemicals pumped into the seas causes potentially multi-generational adverse health effects, and land degradation through mega farming and deforestation occurs in areas of Earth home to 3.2 billion people.

Page 64: JOIN THE DOTS! - Career Launcher · dots! A current affairs series for UPSC Examination’ series which will help you pick up relevant news items of the day from various national

Join the dots! – March 2019 Page: 58

• The report advises adopting less-meat intensive diets, and reducing food waste in both developed and developing countries, would reduce the need to increase food production by 50% to feed the projected 9-10 billion people on the planet in 2050. At present, 33% of global edible food is wasted, and 56% of waste happens in industrialised countries.

About the role OF India

• India’s stated commitment is to lower emissions intensity of its GDP by 33-35% compared to 2005 levels by 2030; increase total cumulative electricity generation from fossil free energy sources to 40% by 2030, and create additional carbon sink of 2.5 to 3 billion tons through additional forest and tree cover.

• India is on track to achieve two of these goals — of emissions intensity and electricity generation — according to independent climate-watch site Climate Tracker.

• For India to leapfrog onto a 1.5-degree pathway it would have to “abandon plans to build new coal-fired power plants,” said Climate Tracker’s most updated analysis as of Dec 2018.

• The landmark Paris Agreement of 2015 aims to keeping a global temperature rise this century well to “…below 2 degrees Celsius above pre-industrial levels and to pursue efforts to limit the temperature increase even further to 1.5 degrees Celsius.”

• Currently environmental ministers and delegates from around the world are participating in a UN conference in Nairobi to discuss issues such as stopping food waste, promoting the spread of electric mobility, and tackling the crisis of plastic pollution.

Young Global Leaders 2019: WEF recognises Nara Lokesh as Young Global Leader for 2019

Relevance IN – Prelims(about young global leader) + Mains (GS III economic development + GS II international organisations)

What’s the NEWS

• The World Economic Forum (WEF) on recognised Andhra Pradesh IT Minister Nara Lokesh as their Young Global Leader for 2019.

• The WEF announced the 2019 Young Global Leaders Nara Lokesh which included 127 individuals from across the globe.

Know! more about it

• Lokesh has been recognised as Young Global Leader for fifth consecutive time this year. The forum of Young Global Leaders 2019 witnessed the participation of social activists, business leaders, public servants, artists and technologists under the age of 40.

• Launched by the World Economic Forum (WEF) in 2004, the Global Young Leaders forum is a community of young leaders from all walks of life and from every region of the world, including Heads of governments to UN Goodwill Ambassadors and Nobel Prize winners.

• According to WEF, more than half of the new members who made it to the list are females.

• Most of the young leaders are from developing countries with enhanced understanding and promoting innovative ideas in the fields of art, business, civil society, energy, government and health.

• The individuals are expected to take part in a five year programme that will help them identify new ways to learn more innovative ideas.

• These leaders to take forward the challenge of improving the state of the world. In offering opportunities and experiences to transform their understanding of the possibilities presented by the Fourth Industrial Revolution, WEF are investing in them to drive a future where all can flourish

Know! more about the forum

• The Forum of Young Global Leaders, the World Economic Forum’s foundation for remarkable leaders under 40, was founded to fuel new models of leadership.

Page 65: JOIN THE DOTS! - Career Launcher · dots! A current affairs series for UPSC Examination’ series which will help you pick up relevant news items of the day from various national

Page: 59 Join the dots! – March 2019

• Young Global Leaders participate in the Annual Meeting of the New Champions, established in 2007 and known informally as “Summer Davos”, alongside Global Growth Companies and other delegations to the World Economic Forum.

• Young Global Leaders are united by the belief that the urgent problems of today present an opportunity to forge a better future across sectors, generations and borders.

Know! about WEF

• The World Economic Forum is the International Organization for Public-Private Cooperation.

• The Forum engages the foremost political, business and other leaders of society to shape global, regional and industry agendas.

• It was established in 1971 as a not-for-profit foundation and is headquartered in Geneva, Switzerland. It is independent, impartial and not tied to any special interests.

• The Forum strives in all its efforts to demonstrate entrepreneurship in the global public interest while upholding the highest standards of governance. Moral and intellectual integrity is at the heart of everything it does.

• The WEF hosts annual meeting at the end of January in Davos, a mountain resort in Graubünden, in the eastern Alps region of Switzerland.

• The meeting brings together some 2,500 business leaders, international political leaders, economists, celebrities and journalists for up to four days to discuss the most pressing issues facing the world.

ISRO set for next PSLV-C45 launch

Relevance IN – Prelims (about PSLV and its stages and configurations) + Manis (GS III awareness in the field of space)

What’s the NEWS

• ISRO is now set for the launch of Polar Satellite Launch Vehicle (PSLV)-C45 on April 1.

Know! more about this PSLV C- 45 launch

• This PSLV will be ISRO’s first three-orbit mission that will launch satellites in three different orbits.

• It is set to launch the advanced electronic intelligence satellite EMISAT with 28 commercial satellites.

• This PSLV-C45 is the first three-orbit mission of ISRO and the first to use solar panel in rocket fourth stage (PS4).

• Earlier PSLV launched satellites in two different orbits. This PSLV will be a special launch as it will launch satellites in three different orbits.

• After launch of electronic intelligence satellite EMISAT into orbit at 780 km, it will inject 28 guest satellites into orbit at 504 km.

• Then rocket fourth stage (PS4) instead of burning out, will automatically reorient itself to establish an orbital platform at 485 km orbit to carry out scientific experiments

• It will have a small satellite developed by Indian Institute of Space Science and Technology (IIST) to carry out scientific experiments on micro-gravity and will be in orbit for six months.

• Following that, PSLV-C46 is set to launch earth observation satellite Cartosat-3 and PSLV-C47 mission for launch of RISAT 2BR 1, a Radar Imaging Satellite for reconnaissance and strategic surveillance. Dates for these launches are yet to be finalised.

• Normally, PSLV is a four-stage launch vehicle with alternating solid and liquid stages. This PSLV-C45 mission is unique as it is the first PSLV fourth stage that uses solar panels for required thrust.

Know! some facts about Kalamsat, Microsat-R satellites on PSLV-C44 rocket

• The Indian Space Research Organisation (ISRO) launched a students’ satellite Kalamsat and an imaging satellite Microsat-R from the Satish Dhawan Space Centre in Sriharikota, Andhra Pradesh on January 24, 2019, marking its first launch in 2019.

• The national space agency’s rocket, PSLV C44 carried the satellites into the orbit.

Page 66: JOIN THE DOTS! - Career Launcher · dots! A current affairs series for UPSC Examination’ series which will help you pick up relevant news items of the day from various national

Join the dots! – March 2019 Page: 60

Kalamsat Satellite• The Kalamsat is a 10 cm cube nanosatellite weighing about 1.2 kg and has a life span of about two months.

• Kalamsat, prepared by a student and Chennai-based Space Kidz India, is a small satellite (10 x 10 x 10 cm) and is meant for HAM radio services.

• The Kalamsat satellite was the first to use fourth stage (PS4) of the launch vehicle PSLV-C44 as an orbital platform.

• The fourth stage moved to higher circular orbit so as to establish an orbital platform for carrying out experiments.

• With this launch, India became the first country to use the fourth stage of a space rocket as an orbital platform for micro-gravity experiments.

Configuration of PSLV-C44 rocket• The PSLV is a four-stage engine expendable rocket

with alternating solid and liquid fuel.

• One of the specialties of the launch was the configuration of the rocket. ISRO used the aluminium tank for the first time in the fourth stage of the launch of PSLV C 44.

• In its normal configuration, the rocket would have six strap-on motors hugging the its first stage.

• However, for the launch of Kalamsat and Microsat-R satellites, the rocket carried only two strap-on motors by the sides of the first fuel stage at the bottom.

• This was the first time the launch vehicle is built in this configuration known as PSLV-DL.

• This will reduce the weight and increase the mass in the four-stage engine expendable rocket with alternating solid and liquid fuel.

• After parking the satellites in the intended orbits, the fourth stage of the rocket will be taken to a circular orbit in space for carrying out certain experiments by the scientists.

• Normally, the fourth stage is kept deserted in space after the injection of the satellites. This time, it will be kept ‘live’ for carrying out innovative studies.

World Consumer Rights Day

Relevance IN – Prelims (about consumer rights day)

What’s the NEWS• The consumer movement marks 15th March with World Consumer Rights Day every year, as a means of raising

global awareness about consumer rights and needs.

• Celebrating the day is a chance to demand that the rights of all consumers are respected and protected, and to protest against market abuses and social injustices which undermine those rights.

Know! more about World Consumer Rights day• World Consumer Rights Day was inspired by President John F Kennedy, who sent a special message to the US

Congress on 15th March 1962, in which he formally addressed the issue of consumer rights. He was the first world leader to do so.

Page 67: JOIN THE DOTS! - Career Launcher · dots! A current affairs series for UPSC Examination’ series which will help you pick up relevant news items of the day from various national

Page: 61 Join the dots! – March 2019

• The consumer movement first marked that date in 1983 and now uses the day every year to mobilise action on important issues and campaigns.

• The theme of the World Consumer Rights Day 2019 is “Trusted Smart Products”.

• The theme aims to highlight what consumers want and need from a connected world and how important it is to put them at the heart of the development of these digital products and services.

• World Consumers Day aims to highlight that rights of all consumers are respected and protected and to protest against market abuses and social injustices which undermine those rights.

Consumer Rights in India • The Consumer Protection Act 1986 gives 6 basic rights to the consumers: Right to choose the product. Right to

be protected from all kind of hazardous goods.

• Right to be informed about the performance and quality of all products. Right to be heard in all decision-making processes related to consumer interests. Right to seek redressal, whenever consumer rights have been infringed. Right to complete consumer education.

Erode turmeric gets GI tag after an 8-year process

Relevance IN – Prelims (about Erode turmeric)

What’s the NEWS• After an eight-year-long process, Erode turmeric finally got a Geographical Indication (GI) tag from the

Geographical Indication Registry.

• The tag is a name or sign used on products that correspond to a specific location. Erode city located in Tamil Nadu state

• The Erode Manjal Vanigargal Matrum Kidangu Urimaiyalargal Sangham applied for the GI tag on January 4, 2011, at the office of the Deputy Registrar of GI Registry in Chennai.

• A Geographical Indication is a name or sign used on certain products that correspond to a specific geographical location or origin.

Know! more about Erode turmeric• Erode turmeric is a rhizome, both finger and bulb obtained from the Erode local cultivar.

• In its claim for uniqueness, the application said the mean length of the fingers of Erode turmeric was about 4.15cm and the mean circumference was about 3.03cm.

• The mean bulb length of the mother rhizome is about 4.54cm and the mean circumference is 6.54cm.

• Quality parameters of the turmeric included 2.5 to 4.5% of curcumin content, a golden yellow colour and resistance to pests after boiling.

Fourth United Nations Environment AssemblyWhat’s the NEWS• In a significant first, India piloted resolutions on two important global environment issues relating to Single-use

Plastics and Sustainable Nitrogen management at the fourth session of United Nations Environment Assembly (UNEA) which was held in Nairobi from 11th to 15th March 2019.

Page 68: JOIN THE DOTS! - Career Launcher · dots! A current affairs series for UPSC Examination’ series which will help you pick up relevant news items of the day from various national

Join the dots! – March 2019 Page: 62

Know! more about UNEA• UNEA adopted both the resolutions with consensus.

• The theme of the UNEA this year was Innovative Solutions for environmental challenges and sustainable production and consumption.

Global challenges • The global nitrogen use efficiency is low, resulting in pollution by reactive nitrogen which threatens human

health, eco system services, contributes to climate change and stratospheric ozone depletion.

• Only a small proportion of the plastics produced globally are recycled with most of it damaging the environment and aquatic bio-diversity.

• Both these are global challenges and the resolutions piloted by India at the UNEA are vital first steps towards addressing these issues and attracting focus of the global community.

Issue of climate finance• Climate finance is an important lever for climate action related to both mitigation and adaption in the developing

countries.

• The contributions to climate finance need to be in consonance with the basic principles of common but differentiated responsibility and respective capabilities (CBDR-RC).

Know! about UNEA• The United Nations Environment Assembly is the world’s highest-level decision-making body on the environment.

• It addresses the critical environmental challenges facing the world today.

• Understanding these challenges and preserving and rehabilitating our environment is at the heart of the 2030 Agenda for Sustainable Development.

• The Environment Assembly meets biennially to set priorities for global environmental policies and develop international environmental law.

• Through its resolutions and calls to action, the Assembly provides leadership and catalyses intergovernmental action on the environment.

• Decision-making requires broad participation, which is why the Assembly provides an opportunity for all peoples to help design solutions for our planet’s health.

Know! the genesis of United Nations Environment Assembly• The United Nations Environment Assembly was created in June 2012, when world leaders called for UN

Environment to be strengthened and upgraded the United Nations Conference on Sustainable Development, also referred to as RIO+20.

• The Fourth Environment Assembly recently held in Nairobi 11-15 March 2019, focusing on the theme “Innovative solutions for environmental challenges and sustainable consumption and production”.

Only 26% of rural toilets use twin-leach pits

Relevance IN – Prelims (about the outcomes of clean India mission and about twin pits and septic tanks) + Mains (GS III government policies and interventions for the development in various sectors

What’s the NEWS• A government-commissioned survey shows that just over a quarter of rural toilets use this twin-pit system.

• The waste from the remainder of rural toilets could create a new nightmare — harmful to health and the environment, and even pushing a new generation into manual scavenging.

Know! about twin - pit system• Under the twin-pit system, two pits are dug with honeycombed walls and earthen floors which allow liquid to

percolate into the surrounding soil.

• When one pit is filled and closed off, waste flow is transferred to the second pit, allowing waste in the first pit to be converted into manure after a year or two.

Page 69: JOIN THE DOTS! - Career Launcher · dots! A current affairs series for UPSC Examination’ series which will help you pick up relevant news items of the day from various national

Page: 63 Join the dots! – March 2019

• Data from the National Annual Rural Sanitation Survey 2018-19, shows that just 26.6% of rural households use the recommended twin-pit system to dispose of excreta from their toilets.

• Septic tanks are the most popular option, with 28% of toilets connected to a septic tank with a soak pit and 6% to a tank without a soak pit.

Contempt of Courts act 1971

Relevance IN – Prelims(about contempt of court act 1971) + Mains (GS II structure, organisation and functioning of the judiciary)

What’s the NEWS

• The Supreme Court has put on hold the Meghalaya High Court’s judgment holding The Shillong Times editor Patricia Mukhim and publisher Shobha Chaudhuri guilty of contempt and fining them Rs 2 lakh each.

Know! about the judgement of Meghalaya H.C

• The Shillong Times has published an article titled “When judges judge for themselves” in pursuant to a high court order directing the government to amend rules so that spouses and children of retired judges become eligible for medical treatment.

• The High Court had found the editor and publisher guilty for publishing the article “When judges judge for themselves”.

Why were they penalized

• The matter comes within the purview of section 15 of the contempt of court Act. 1971. There is a compelling case to use the contempt law sparingly. Otherwise it supresses free speech or dissent

An overview of Contempt of Courts ACT 1971

• Contempt of court refers to actions which defy a court’s authority, cast disrespect on a court, or impede the ability of the court to perform its function.

• The Contempt of Court provisions in India are enshrined under Articles 129 and 215 of the constitution for Supreme Court and High Court respectively and Contempt of Courts Act, 1971.

Civil Contempt

• It is defined as willful disobedience to any judgment, decree, direction, order, writ or other processes of a court or wilful breach of an undertaking given to a court.

Page 70: JOIN THE DOTS! - Career Launcher · dots! A current affairs series for UPSC Examination’ series which will help you pick up relevant news items of the day from various national

Join the dots! – March 2019 Page: 64

Criminal Contempt • It is defined as the publication (whether by words, spoken or written, or by signs, or by visible representation,

or otherwise) of any matter or the doing of any other act which: Scandalises or tends to scandalise, or lowers or tends to lower the authority of, any court, or Prejudices, or interferes or tends to interfere with the due course of any judicial proceeding

What Scandalising the court means• Scandalising the Court broadly refers to statements or publications which have the effect of undermining public

confidence in the judiciary

New hydro policy to help meet renewables target

Relevance IN – Prelims (about the new hydrocarbon policy) + Mains (GS III infrastructure development- energy sector + environment conservation)

What’s the NEWS• The government decided to re-classify large hydroelectric projects as renewable energy

• This will certainly help the sector, the move will also go a long way in meeting the targets set by it for the sector.

Know! about the new hydrocarbon policy• The Union Cabinet approved a new hydroelectricity policy that, among other things, included large hydro

projects within the ambit of renewable energy.

• Prior to the policy, only small hydro projects of a capacity of less than 25 MW were treated as renewable energy. Large hydro projects were treated as a separate source of energy.

• India’s renewable energy sector had an installed capacity of 75,055.92 MW as of February 2019, according to data with the Central Electricity Authority.

• This made up about 21.4% of the overall energy mix, with the rest coming from thermal, nuclear and large hydro sources.

• With the inclusion of large hydro in renewable energy, the energy mix changes drastically.

• Renewable energy capacity would now be 1,20,455.14 MW or 34.4% of the overall energy mix.

Know! more about this policy change • No additional resources have been created through this policy. It is a reclassification of existing capacity.

• The policy has meant a drastic change in the renewable energy mix as well. Whereas earlier, wind energy contributed nearly 50% of all renewable energy capacity, it will now make up only 29.3%.

• Similarly, solar energy’s share will fall from 34.68% to 21.61%.

• The hydro sector, however, will see its share grow from just over 6% to over 41%.

Page 71: JOIN THE DOTS! - Career Launcher · dots! A current affairs series for UPSC Examination’ series which will help you pick up relevant news items of the day from various national

Page: 65 Join the dots! – March 2019

The thermal – hydro imbalance• There has been a huge imbalance in the thermal-hydro mix for the last few years because of a sharp growth in

thermal and complete stagnation in hydro.

• The basic idea is to ramp up hydro because it provides grid stability which a renewable source like wind and solar do not.

• The key reasoning seems to be providing grid stability and a better energy mix.

One more reason for bringing in such a re-classification• The 175 GW renewable energy target of the government by 2022

• This has been quite a high-profile target because the government has been trumpeting it at every chance. It will look bad if it fails to reach it.

• So, apart from the good to the sector, one main reason for the re-classification of hydro as renewables is to add all that capacity to the renewable energy kitty.

‘Green’ coffee: A carbon-neutral project in Kerala

Relevance IN – Prelims (about the various coffee types)

What’s the NEWS• The State Industries Department is gearing up to set up a carbon neutral farm project in Wayanad district of

Kerala .

• The department had assigned the task of executing the project spread across 100 acres of land at Vayarad in the district, to the Kerala Industrial Infrastructure Development Corporation (KINFRA).

Know! about the neutral village• A carbon neutral village coffee park would be set up for which the State government had earmarked ₹150 crore

for first phase construction works.

• As many as 1.5 lakh coffee plants would be planted as part of making the region a carbon-neutral zone

• The project would be executed with the technical support of the Coffee Board.

• Common processing facilities for coffee would also be established. All technical assistance, including for acquiring organic certification and other requirements, would be provided by the KINFRA to farmers

• Wayanad in north Kerala grows about 50,000 to 60,000 tonnes of robusta coffee annually, making it a prominent region in the country cultivating this variety.

The Coffee Board had recived GI status for Wayanad coffee, along with Baba Budan Giri, Araku Valley, Coorg and Chikmagalur coffee recently

Know! more about it• Geographical Indication (GI) tags have been granted to Coorg Arabica Coffee, Wayanad Robusta Coffee,

Chikmagalur Arabica Coffee, Araku Valley Arabica Coffee and Bababudangiris Arabica Coffee, by the Geographical Indications Registry

• Coorg Arabica Coffee is grown in Kodagu, Karnataka.

• Wayanad Robusta Coffee is grown in Wayanad, Kerala.

• Chikmagalur Arabica Coffee and Bababudangiris Arabica Coffee are grown in Chikmagalur, Karnataka.

Know! about Coffee Board of India • The Coffee Board of India is an organisation managed by the Ministry of Commerce and Industry of the

government of India to promote coffee production in India. Head Office is in Bangalore.

• The Coffee Board of India was established by an act of Parliament in 1942.

• Until 1995 the Coffee Board marketed the coffee of many growers from a pooled supply, but after that time coffee marketing became a private-sector activity due to the economic liberalisation in India.

• The Coffee Boards tradition duties included the promotion of the sale and consumption of coffee in India and abroad, conducting coffee research, financial assistance to establish small coffee growers, safeguarding working conditions for laborers, and managing the surplus pool of unsold coffee.

Page 72: JOIN THE DOTS! - Career Launcher · dots! A current affairs series for UPSC Examination’ series which will help you pick up relevant news items of the day from various national

Join the dots! – March 2019 Page: 66

Pinaki Chandra Ghose set to be India’s first Lokpal: PM-led selection panel clears the former SC judge’s name

Relevance IN – Prelims (about Lokpal act and its provisions) + Mains (GS II important aspects of governance, transparency and accountability)

What’s the NEWS

• Former Supreme Court judge and current member of the National Human Rights Commission (NHRC), Pinaki Chandra Ghose, is likely to be India’s first anti-corruption ombudsman, or Lokpal, after his name was cleared and recommended by the high-level selection committee chaired by Prime Minister Narendra Modi.

Know! about the high level selection committee

• Apart from Prime Minister other members of the committee are Chief Justice of India Ranjan Gogoi, Lok Sabha Speaker Sumitra Mahajan and eminent jurist Mukul Rohatgi.

• Leader of the Opposition in the Lok Sabha Mallikarjun Kharge, who is part of the committee, did not attend the meeting after he was invited as “special invitee.”

• Mr. Kharge had refused to attend earlier meetings too, protesting against his being invited as a “special invitee.”

• The government was prompted to make the selection after the Supreme Court set the February-end deadline.

Know! about the lokpal act

• The Lokpal Act, which was passed in 2013 after a nationwide anti-corruption movement, provides for setting up of Lokpal at the centre and Lokayuktas in the States to probe corruption complaints against top functionaries and public servants, including the Prime Minister and the Chief Ministers.

SBI, ICICI, HDFC Bank to remain D-SIBs: Domestic Systemically Important Banks (D-SIBs)

Relevance In – Prelims (about D-SIBs) + Mains (GS III economic development)

What’s the NEWS

• The Reserve Bank of India (RBI) has announced that SBI, ICICI and HDFC Banks would continue as Domestic Systemically Important Banks (D-SIBs) for 2018.

• Inclusion in the list gives additional comfort to investors that these banks won’t be allowed to fail and therefore, borrowing costs of these banks from the markets are cheaper than their peers

Know! about D -SIBs

• Domestic Systemically Important Banks (D-SIBs) recognition implies that banks are too big to fail.

• This creates an expectation of government support for them in times of financial distress.

• As per the norms, these banks will have to set aside more capital for their continued operation.

• RBI comes with the list every year since 2015. Inclusion in D-SIB indicates that failure of any of these banks would have a cascading effect on Indian financial system.

• As a result, banks enjoy certain advantages in funding markets. Inclusion in the list gives additional comfort to investors that these banks won’t be allowed to fail and therefore, borrowing costs of these banks from the markets are cheaper than their peers.

• Inclusion in D-SIB also implies that the failure of any of these banks would have a cascading effect on the Indian financial system.

Page 73: JOIN THE DOTS! - Career Launcher · dots! A current affairs series for UPSC Examination’ series which will help you pick up relevant news items of the day from various national

Page: 67 Join the dots! – March 2019

3rd Indo-Japan Workshop on Disaster Risk Reduction held in New Delhi

Relevance IN – Mains (GS III disaster management)

What’s the NEWS• The 3rd Indo-Japan Workshop on Disaster Risk Reduction was held recently

• The workshop was attended by about 140 delegates from Japan and India including experts from both the governments, top premium research institutes, city administrators, specialized Disaster Management agencies and private sector.

Know! more about the workshop • The Government of India and the Government of Japan had signed a Memorandum of Cooperation (MoC) in the

field of Disaster Risk Reduction (DRR) in September 2017.

• The 3rd Indo-Japan workshop is the follow-up of the deliberations held during the 1st Indo-Japan Workshop on DRR held on March 18-19, 2018 in New Delhi as well as during the 2nd Indo-Japan workshop on DRR held on October 13-15, 2018 in Tokyo, Japan.

• The 3rd workshop was organized with an objective of enhancing collaboration between research institutes, cities and the private sector in the field of Disaster Risk Reduction.

• The significance of three themes set for this workshop as collaboration amongst research institutes, among cities and among private companies, from the point of view that Disaster Risk Reduction should involve various stakeholders, and expressed Japan’s continuous support to India’s challenges towards DRR in every possible way.

International Workshop on Disaster Resilient Infrastructure

What’s the NEWS• An International Workshop on Disaster Resilient Infrastructure (IWDRI) inaugurated today

• The two-day workshop is being organised by the National Disaster Management Authority (NDMA) in collaboration with United Nations Office for Disaster Risk Reduction (UNISDR), and in partnership with the Global Commission on Adaptation, United Nations Development Programme and the World Bank.

Know! all about the workshop The workshop aims to

i) identify good practices of disaster risk management in key infrastructure sectors

ii) identify specific areas and pathways for collaborative research on DRI (Transport, Energy, Telecom and Water)

iii) discuss and co-create the broad contours of the Coalition for Disaster Resilient Infrastructure (CDRI) as well as a notional roll-out plan for the next three years, and

iv) build a forum for members to work on areas of common interest and make specific commitments.

• It will bring together countries from different parts of the world, multilateral development banks, UN agencies, academia and research institutions, the private sector, academics and policy think tanks to discuss and collaborate on promoting policies and practices towards achieving disaster resilience of large infrastructure systems (transport, telecom, energy, water).

• This will also be a great opportunity to learn from the unique experiences of different countries.

Know! about the various international agreements • Various international agreements have also reiterated the importance and long-term benefits of investing in

resilient infrastructure.

• The Sendai Framework for Disaster Risk Reduction (SFDRR), 2015-2030, which is the first major agreement of the post-2015 development agenda, identifies investing in Disaster Risk Reduction (DRR) for resilience and to build back better in reconstruction as priorities for action towards reducing disaster risk.

• Similarly, Goal 9 of the Sustainable Development Goals (SDGs) recognizes disaster resilient infrastructure as a crucial driver of economic growth and development.

• The first International Workshop on Disaster Resilient Infrastructure (IWDRI 2018) was held in January 2018.

Page 74: JOIN THE DOTS! - Career Launcher · dots! A current affairs series for UPSC Examination’ series which will help you pick up relevant news items of the day from various national

Join the dots! – March 2019 Page: 68

• This workshop will further build upon some of the ideas generated at the IWDRI 2018 as a crucial milestone towards the establishment of the Coalition for Disaster Resilient Infrastructure (CDRI).

• The CDRI is envisaged as a knowledge exchange and capacity development partnership. India announced the creation of a CDRI soon after the Asian Ministerial Conference on Disaster Risk Reduction, which was held in New Delhi in 2016.

Theatre Level Readiness and Operational Exercise (TROPEX)

Relevance IN – Prelims(about TROPEX)

What’s the NEWS• Chief of Naval Staff Admiral Sunil Lanba arrived in Kochi for a debriefing of the recently concluded annual

Theatre Level Readiness and Operational Exercise (TROPEX) held earlier this year in the Arabian Sea and north Indian Ocean.

• The day-long review of TROPEX 2019 was undertaken to examine the conduct of the exercise and to assess the operational readiness of the Indian Navy.

Know! about the Exercise • The exercise that began in January graduated smoothly into providing the Navy a high operational readiness

posture post the Pulwama attack on February 14.

• Around 60 Naval ships, 12 ships of Indian Coast Guard and 60 aircraft, participated in TROPEX 19.

• The exercise also included a Tri-services Amphibious Exercise that saw the participation of the Army and the Air Force.

• Commanders-in-Chief of all the three naval commands along with a number of senior operational commanders and representatives from the Indian Army, Air Force, and the Indian Coast Guard participated in the discussions held at Naval Base, Kochi.

Know! more about TROPEX • The Annual Integrated Theatre Level Operational Readiness Exercise - in short TROPEX - the largest

maritime exercise of the Indian Navy was conducted in the month of February in the Arabian Sea and North Indian Ocean.

• TROPEX 2019 was thus far the largest in terms of geographical extent covering the IOR, and also with regard to number of units participating.

• The exercise was conducted from 07 Jan 19 and graduated smoothly into providing the IN a high operational readiness posture post the Pulwama attack on 14 Feb 19.

• About 60 ships of Indian Navy, 12 ships of Indian Coast Guard and 60 aircraft participated in TROPEX 19.

• The exercise also included a Tri-services Amphibious Exercise that saw the participation of Army and Air Force personnel and assets.

• As a prelude to TROPEX, the largest coastal defence exercise over codenamed ‘Sea Vigil’, was conducted on 22 and 23 Jan 19 with participation of all coastal states and union territories along with all maritime stake holders.

Opening ceremony: Africa-India Field Training Exercise-2019

What’s the NEWS• The inaugural Africa-India Field

Training Exercise-2019 for India and African nations called AFINDEX-19 scheduled from 18 March to 27 March 2019 started with a grand opening ceremony on 18 March 2019 at Aundh Military Station, Pune.

Page 75: JOIN THE DOTS! - Career Launcher · dots! A current affairs series for UPSC Examination’ series which will help you pick up relevant news items of the day from various national

Page: 69 Join the dots! – March 2019

Know! more about AFINDEX -19• Contingents of the 17 African Nations i.e. Benin, Botswana, Egypt, Ghana, Kenya, Mauritius, Mozambique,

Namibia, Niger, Nigeria, Senegal, South Africa, Sudan, Tanzania, Uganda, Zambia and Zimbabwe came together for the opening ceremony alongwith a contingent of Maratha Light Infantry representing India.

• The aim of the exercise is to practice the participating nations in planning and conduct of Humanitarian Mine Assistance and Peace Keeping Operations under Chapter VII of United Nations Peace Keeping Operations.

• The exercise will focus on exchange of best practices between the participating nations, team building and tactical level operations in conduct of United Nations mandated tasks to include establishment of a new mission, siting of a United Nations Headquarters for Peace Keeping operations, siting of Military Observer sites during the peace keeping missions, protection of civilians, nuances of standing combat deployment, convoy protection, patrolling aspects and aspects related to Humanitarian Mine Assistance.

Tropical Cyclone Idai

Relevance IN – Prelims (about Idai cyclone) + Mains – GS I Important geophysical phenomena

What’s the NEWS• Flooding caused by the Tropical

Cyclone Idai weather system since early March had affected more than 1 million people

• Idai has caused huge deaths and economic losses in Mozambique, Malawi, Zimbabwe, and South Africa.

• Cyclone Idai made landfall on Mozambique and later moved in a westerly direction.

• Idai is the deadliest Tropical cyclone witnessed in 2019.Idai originated from a tropical depression that formed off the eastern coast of Mozambique and made landfall in Mozambique.

Know! about Tropical Cyclone• A tropical cyclone is a rapidly rotating

storm system characterized by a low-pressure center, a closed low-level atmospheric circulation, strong winds, and a spiral arrangement of thunderstorms that produce heavy rain.

• Depending on its location and strength, a tropical cyclone is referred to by different names, including hurricane typhoon ,tropical storm, cyclonic storm, tropical depression, and simply cyclone.

• A hurricane is a tropical cyclone that occurs in the Atlantic Ocean and northeastern Pacific Ocean, and a typhoon occurs in the northwestern Pacific Ocean; in the south Pacific or Indian Ocean, comparable storms are referred to simply as “tropical cyclones” or “severe cyclonic storms

• Tropical Cyclone is an intense low-pressure area or a whirl in the atmosphere over tropical or sub-tropical waters, with organised convection, circulating either anti-clockwise (in the northern hemisphere) or clockwise (in the southern hemisphere).

Page 76: JOIN THE DOTS! - Career Launcher · dots! A current affairs series for UPSC Examination’ series which will help you pick up relevant news items of the day from various national

Join the dots! – March 2019 Page: 70

Conditions which favour Tropical Cyclone Formation• A source of warm, moist air derived from tropical oceans with sea surface temperature normally near to or in

excess of 27 ₹C

• Winds near the ocean surface blowing from different directions converging and causing air to rise and storm clouds to form

• Winds which do not vary greatly with height known as low wind shear. This allows the storm clouds to rise vertically to high levels.

• Coriolis force/spin induced by the rotation of the Earth. The formation mechanisms vary across the world, but once a cluster of storm clouds starts to rotate, it becomes a tropical depression.

• If it continues to develop it becomes a tropical storm, and later a cyclone/ super cyclone. Pressure increases outwards from the centre of a cyclonic storm.

• The amount of the pressure drop in the centre and the rate at which it increases outwards gives the intensity of the cyclones and the strength of winds.

Steps taken by India• The Indian Navy had launched a Humanitarian Assistance and Disaster Relief (HADR) operation in coordination

with local officials to evacuate about 5,000 people stranded at Buzi near Port Beira in Mozambique

World Happiness Report: India drops even further, ranks 140 on index of 156 countries: Pakistan, Bhutan, Sri Lanka and Bangladesh ranked higher than India.

Relevance IN – Prelims (about the report)

What’s the NEWS• India was ranked at 140 on the United Nation’s latest World Happiness Report that gauged 156 countries, a

decline of seven spots from the last edition of the survey.

• India was also among the five countries that experienced the highest decline since 2005-2008 in the index.

• Finland topped the list for the second consecutive year, followed by Denmark and Norway. South Sudan was ranked at the bottom of the index.

Know! more about the report• The World Happiness Report is an annual publication of the United Nations Sustainable Development

Solutions Network.

Page 77: JOIN THE DOTS! - Career Launcher · dots! A current affairs series for UPSC Examination’ series which will help you pick up relevant news items of the day from various national

Page: 71 Join the dots! – March 2019

• It contains articles, and rankings of national happiness based on respondent ratings of their own lives, which the report also correlates with various life factors. Finland was ranked the happiest country in the world twice in row

• The World Happiness Report is a landmark survey of the state of global happiness that ranks 156 countries by how happy their citizens perceive themselves to be.

• This year’s World Happiness Report focuses on happiness and the community: how happiness has evolved over the past dozen years, with a focus on the technologies, social norms, conflicts and government policies that have driven those changes.

• The report said the countries which had suffered such a decline had some combination of economic, political, and social stresses.

• The study based its findings on individuals’ assessment of their lives in relation to the Gross Domestic Product per capita income, social support, healthy life expectancy, freedom, generosity and absence of corruption.

• Meanwhile, India’s South Asian neighbours ranked higher with Pakistan pegged at 67, Bhutan at 95, Bangladesh at 125 and Sri Lanka at 130.

• The report said the large gaps in happiness between countries will continue to create pressure to migrate.

India-Indonesia Coordinated Patrol (Ind-Indo Corpat) Commences

Relevance IN – Prelims (about Corpat)

What’s the NEWS• 33rd Edition of India-Indonesia Coordinated Patrol (Ind-Indo Corpat) Commences at Port Blair

• Indonesian Naval Ship KRI Sultan Thaha Syaifuddin and Maritime Patrol Aircraft CN-235 arrived at Port Blair, Andaman & Nicobar Islands, India for the Opening Ceremony of the 33rd edition of the India-Indonesia Coordinated Patrol (IND-INDO CORPAT) to be held from 19 Mar to 04 Apr 2019.

Know! more about the exercise • The ship and aircraft from both the countries would undertake patrolling on the respective sides of 236 nautical

miles long International Maritime Boundary line. The patrolling would be conducted in three phases from 22-31 March 2019.

Page 78: JOIN THE DOTS! - Career Launcher · dots! A current affairs series for UPSC Examination’ series which will help you pick up relevant news items of the day from various national

Join the dots! – March 2019 Page: 72

• The IND-INDO CORPAT Series of bilateral seek to underscore India’s peaceful presence and solidarity with friendly Maritime neighbours countries to ensure good order in the maritime domain, consolidate interoperability and strengthen existing bonds of friendship between India and Indonesia.

• Indian Naval assets have been increasingly deployed in the recent times to address the maritime concerns on the region.

• In addition, as part of the Indian Government’s vision of SAGAR (Security and Growth for All in the Region), the Indian Navy has also been involved in assisting countries in the Indian Ocean Region with EEZ Surveillance, Search and Rescue, and other capacity-building and capability-enhancement activities.

• The 33rd IND-INDO CORPAT, also coinciding with 70 years of India-Indonesia diplomatic ties, will contribute towards the Indian Navy’s efforts to consolidate inter-operability and forge strong bonds of friendship across the seas.

Indo-Sri Lanka joint Exercise Mitra shakti-VI

Relevance IN – Prelims (about Mitra shakti)

What’s the NEWS • Exercise MITRA SHAKTI is conducted annually as part of military diplomacy and interaction between armies

of India & Sri Lanka.

• The joint exercise for the year 2018-19 will be conducted from 26 March to 08 April 2019 in Sri Lanka.

• Troops from 1st Battalion the BIHAR Regiment of the Indian Army and Gemunu Watch Battalion of Sri Lankan Army would be jointly undertaking the exercise.

Know! more about the exercise• The aim of the exercise is to build and promote close relations between armies of both the countries and to

enhance ability of joint exercise commander to take military contingents of both nations under command.

• The exercise will involve tactical level operations in an international Counter Insurgency and Counter Terrorist environment under United Nations mandate.

• Exercise MITRA SHAKTI-VI will go a long way in further cementing relationship between both the nations and will act as a catalyst in bringing synergy and cooperation at grassroots levels between both the armies.

34th meeting of the GST Council

Relevance IN – Prelims(about the changes in tax slabs) + Mains (GS II economic developments + GS II centre state relations)

What’s the NEWS• GST Council in the 34th meeting discussed the operational details for implementation of the recommendations

made by the council in its 33rd meeting for lower effective GST rate of 1% in case of affordable houses and 5% on construction of houses other than affordable house.

Option in respect of ongoing projects• The promoters shall be given a one -time option to continue to pay tax at the old rates (effective rate of 8% or

12% with ITC) on ongoing projects (buildings where construction and actual booking have both started before 01.04.2019) which have not been completed by 31.03.2019.

• The option shall be exercised once within a prescribed time frame and where the option is not exercised within the prescribed time limit, new rates shall apply.

New tax rates:• The new tax rates which shall be applicable to new projects or ongoing projects which have exercised the above

option to pay tax in the new regime are as follows.

New rate of 1% without input tax credit (ITC) on construction of affordable houses shall be available for,

• all houses which meet the definition of affordable houses as decided by GSTC (area 60 sqm in metros / 90 sqm in non- metros and value upto RS. 45 lakhs), and

Page 79: JOIN THE DOTS! - Career Launcher · dots! A current affairs series for UPSC Examination’ series which will help you pick up relevant news items of the day from various national

Page: 73 Join the dots! – March 2019

• affordable houses being constructed in ongoing projects under the existing central and state housing schemes presently eligible for concessional rate of 8% GST (after 1/3rd land abatement).

New rate of 5% without input tax credit shall be applicable on construction of,-

• all houses other than affordable houses in ongoing projects whether booked prior to or after 01.04.2019. In case of houses booked prior to 01.04.2019, new rate shall be available on instalments payable on or after 01.04.2019.

• all houses other than affordable houses in new projects.

• commercial apartments such as shops, offices etc. in a residential real estate project (RREP) in which the carpet area of commercial apartments is not more than 15% of total carpet area of all apartments.

Census of otters

Relevance IN – Prelims (about otters)

What’s the NEWS

• For the first time, Uttar Pradesh is taking a census of otters in its protected areas. The exercise that began in the Pilibhit Tiger Reserve (PTR), will be completed by the end of this month.

Know! all about Otters and census

• Otters are an important part of the forest ecosystem. A thriving population of otters means a healthy ecosystem

• A mammal, an otter spends much of its time in or close to water bodies. Otters live on fish.

• Pilibhit Tiger Reserve (PTR), is in the foothills of the Himalayas, south of Nepal. Covering an area of approximately 800 square kilometres, the reserve sprawls across parts of Pilibhit, Lakhimpur Kheri and Bahraich districts.

• With the Sharda and Ghaghara rivers encircling a considerable part of the reserve, it is rich in water bodies.

Know! more about Otters (as natural indicator)

• A thriving population of otters means a healthy ecosystem. A growing or healthy population of otters means the water bodies are pollution-free.

• Clean water bodies mean a healthy ecosystem of the forest.

• Otters thriving and getting sufficient food to eat means the water bodies in the reserve are in a fine state and the aquatic life in them is healthy.

India is home to 3 of the 13 species of otters found worldwide.

• Eurasian Otter (Lutra lutra) – IUCN status: Near Threatened.

• Smooth-coated Otter (Lutra perspicillata) – IUCN status: Vulnerable.

• Small-clawed Otter (Aonyx cinereus) – IUCN status: Vulnerable.

Page 80: JOIN THE DOTS! - Career Launcher · dots! A current affairs series for UPSC Examination’ series which will help you pick up relevant news items of the day from various national

Join the dots! – March 2019 Page: 74

Bomb cyclone

Relevance IN – Prelims (about bomb cyclone)

What’s the NEWS

• The bomb cyclone swept has through the US Great Plains resulting in blizzard conditions, hurricane-like winds,

snow and heavy rain and subsequent flooding.

Know! about Bomb Cyclone

• Bomb Cyclone or Bombogenesis Bomb Cyclone or bombogenesis refer to a rapidly intensifying area of low-

pressure winter storm when there is a pressure drop by at least 24 millibars in 24 hours.

• The decrease of the pressure intensifies the storm. This pressure drop intensification and subsequent winter

storms result in strong winds, beach erosion and coastal flooding, especially with high tide.

• Bombogenesis storm can be tropical or non-tropical and proximity to a large body of water, particularly an

ocean, during cold months contributes to the creation of a bomb cyclone.

• Bomb Cyclone is caused by a collision of warm air and cold air which develop into rotating storm-like pattern

and lead to an explosive deepening of pressure. Worldwide about 40 to 50 ‘bomb cyclones brew each year.

Indian Navy - first Responder to Cyclone ‘IDAI’ in Mozambique

Relevance IN – Prelims (about IDAI cyclone)

What’s the NEWS

• The ships of First Training Squadron of Indian Navy (Sujata, Sarathi and Shardul) operating in the Southern

Indian Ocean were diverted to Port Beira, Mozambique based on request received from the Government of

Mozambique to provide Humanitarian Assistance and Disaster Relief (HADR) to the local population post the

devastation caused by cyclone ‘IDAI’ which struck the coast of Mozambique on 15 Mar 19.

• Cyclone ‘IDAI’ made landfall at Beira, Mozambique in early hours of 15 Mar 19 causing widespread damage and

loss of human life in the Central and Northern provinces of the country.

• The disembarkation of HADR stores including food, medicines and clothing to the Mozambique Defence

authorities has been completed. Arrangements are in progress to disembark potable water.

Page 81: JOIN THE DOTS! - Career Launcher · dots! A current affairs series for UPSC Examination’ series which will help you pick up relevant news items of the day from various national

Page: 75 Join the dots! – March 2019

World Water Day

Relevance IN – Prelims (about World water day) + Mains (GS III environment conservation)

What’s the NEWS• World Water Day is an annual UN observance day (always on

22 March) that highlights the importance of freshwater.

• The day is used to advocate for the sustainable management of freshwater resources.

Know! about World Water Day• World Water Day is celebrated around the world with a variety

of events. These can be educational, theatrical, musical or lobbying in nature.

• The day can also include campaigns to raise money for water projects. The first World Water Day, designated by the United Nations, was commemorated in 1993.Each year many countries celebrate World Water Day.

• UN-Water is the convener for World Water Day and selects a theme for each year in consultation with UN organizations that share an interest in that year’s focus.

• The theme for 2019 is “Leaving no one behind” and encourages people to consider marginalized groups as these are often overlooked and discriminated against when they try to access safe water.

• The focus on universal access to clean water, sanitation and hygiene (WASH) is in line with the targets of Sustainable Development Goal 6.

• The UN World Water Development Report (WWDR) is released each year around World Water Day.

Know! more about World Water Day• Sustainable Development Goal 6 is crystal clear: water for all by 2030. By definition, this means leaving

no one behind. But today, billions of people are still living without safe water – their households, schools, workplaces, farms and factories struggling to survive and thrive.

• Marginalized groups – women, children, refugees, indigenous peoples, disabled people and many others – are often overlooked, and sometimes face discrimination, as they try to access and manage the safe water they need.

• This World Water Day, 22nd March, is about tackling the water crisis by addressing the reasons why so many people are being left behind.

Indian Air Force to participate in Langkawi International Maritime Aero Expo (LIMA) 2019

Relevance IN – Prelims (about LIMA 2019)

What’s the NEWS• Langkawi International Maritime Aero Expo (LIMA-2019) is planned in

Langkawi, Malaysia from 26 March 2019 to 30 March 2019.

• Indian Air Force is participating in the Maritime Aero Expo for the first time, during which it will showcase its indigenously developed LCA fighter aircraft.

• Participation of IAF in LIMA-2019 will provide an opportunity to air-warriors to interact with their Royal Malaysian Air Force (RMAF) counterparts and foster close relationship between the two services.

• This will serve as a foundation for any future interaction with the Malaysian Air Force. It will also provide an opportunity to RMAF to assess the capabilities of LCA.

Page 82: JOIN THE DOTS! - Career Launcher · dots! A current affairs series for UPSC Examination’ series which will help you pick up relevant news items of the day from various national

Join the dots! – March 2019 Page: 76

Know! more about LIMA• The Langkawi International Maritime & Aerospace Exhibition (LIMA) is the largest show of its kind within the

Asia Pacific region.

• Its impressive list of international exhibitors and suppliers is more than matched by the supportive presence of industry elites, which range from senior Government officials, and military and civil delegates, to industry movers and shakers and more.

• Held biennially since its debut in 1991, LIMA is an ideal platform where industry stakeholders could engage and expand their networks towards forging new partnerships and business agreements.

• Its insightful round tables and conferences, as well as business forums offer access to invaluable information that enables businesses to stay ahead of the competition and seize new opportunities in the dynamic market of Asia Pacific.

Navroz Festival

Relevance IN – Prelims (about the Navroz fest)

What’s the NEWS• Navroz is a 3,000-year-old Zoroastrian tradition, a ritual celebration that signals the start of Spring and the

Persian new year.

Know! all about Navroz• In more modern times, in A.D. 1079, a king of Iran named Jalaluddin Malekshah started observing it on March 21.

• In the 18th century, a rich tradesman from Surat, Nusservanji Kohyaji, who often travelled to Iran, came to know about Navroz and began celebrating the day back home; that brought the festival to India.

• Over a period of time, the festival was introduced in India on a wide scale by members of the Parsi community who eventually connected it to Jamshed, the illustrious king of Iran. Thus, the day came to be known as Jamshedi Navroz.

• King Jamshed, like other ancient kings of Iran, was known for his truthfulness and righteousness. For Parsis, celebrating Navroz in its true sense means living truthfully and walking on the righteous path.

• Navroz is the Iranian New Year celebrated by several ethnolinguistic communities around the world irrespective of their religious background.

• In India Parsis who follow Zoroastrianism celebrate Navroz. Navroz was celebrated on March 21st.

• Navroz is also the day of the Spring equinox and the rituals are performed based on the movements of the sun during the course of the day

World Down Syndrome Day

Relevance IN – Prelims (about World Down Syndrome)

What’s the NEWS• World Down Syndrome Day (WDSD) is on

March 21. On this day, people with Down syndrome and those who live and work with them throughout the world organize and participate in activities and events to raise public awareness and create a single global voice for advocating for the rights, inclusion and well being of people with Down syndrome.

Know! all about Down syndrome• Down syndrome is a naturally occurring chromosomal arrangement that has always been a part of the human

condition, exists in all regions across the globe and commonly results in variable effects on learning styles, physical characteristics or health.

Page 83: JOIN THE DOTS! - Career Launcher · dots! A current affairs series for UPSC Examination’ series which will help you pick up relevant news items of the day from various national

Page: 77 Join the dots! – March 2019

• Adequate access to health care, to early intervention programmes and to inclusive education, as well as appropriate research, are vital to the growth and development of the individual.

• In December 2011, the General Assembly declared 21 March as World Down Syndrome Day

• The General Assembly decided, with effect from 2012, to observe World Down Syndrome Day on 21 March each year, and Invites all Member States, relevant organizations of the United Nations system and other international organizations, as well as civil society, including non-governmental organizations and the private sector, to observe World Down Syndrome Day in an appropriate manner, in order to raise public awareness of Down syndrome.

For WDSD 2019, Down Syndrome International focuses on: • Leave no one behind: All people with Down syndrome must have opportunities to live fulfilling lives, included

on a full and equal basis with others, in all aspects of society.

• The 2030 UN Agenda for Sustainable Development, a global plan of action for people, planet and prosperity, pledges that “no one will be left behind”.

Know! more about Down Syndrome• Down Syndrome is a genetic disorder which results in delayed physical and mental growth in individuals.

• The genetic disorder has been named after British doctor John Langdon Down who fully described the syndrome in 1866.

• In normal conditions, people have only two copies of Chromosome 21 whereas those with Down’s syndrome there is a presence of a part or full of the third copy of chromosome 21.

• Even though Down’s Syndrome is incurable, the quality of life of those suffering could be considerably improved by giving proper medical support and education.

• World Down Syndrome Day 21 March is observed as World Syndrome Day to increase global awareness about the syndrome by the United Nations since 2012

• March 21st was chosen to signify uniqueness of the triplication of the 21st chromosome which causes Down syndrome

International Day of Forests 2019

Relevance IN – Prelims (about IDF 2019)

What’s the NEWS• The United Nations General Assembly proclaimed 21 March the International Day of Forests (IDF) in 2012.

• The Day celebrates and raises awareness of the importance of all types of forests. On each International Day of Forests, countries are encouraged to undertake local, national and international efforts to organize activities involving forests and trees, such as tree planting campaigns.

• The theme for each International Day of Forests is chosen by the Collaborative Partnership on Forests. The theme for 2019 is Forests and Education.

Page 84: JOIN THE DOTS! - Career Launcher · dots! A current affairs series for UPSC Examination’ series which will help you pick up relevant news items of the day from various national

Join the dots! – March 2019 Page: 78

Know! more about forest day

• Every 21 March the United Nations raises awareness of the importance of all types of forests. This year the International Day of Forests promotes education to Learn to Love Forests.

• It underscores the importance of education at all levels in achieving sustainable forest management and biodiversity conservation. Healthy forests mean healthy, resilient communities and prosperous economies.

• The theme ‘Forests and Education’ underscores the importance of education at all levels in achieving sustainable forest management and biodiversity conservation.

Evidence of water, particle plumes discovered on asteroid Bennu: NASA

Relevance IN – Prelims (about Bennu and OSIRIS-Rex)

What’s the NEWS• Shortly after the discovery of the particle plumes on January 6, the mission science team increased the frequency

of observations, and subsequently detected additional particle plumes during the following two months.

Know! more about the recent observations• A NASA spacecraft, that will return a sample of a near-Earth asteroid named Bennu to Earth in 2023, has

discovered plumes erupting from the cosmic body’s surface — among a numerous other findings including evidence of water-bearing minerals.

• Bennu also revealed itself to be more rugged than expected, challenging the mission team to alter its flight and sample collection plans, due to the rough terrain.

Know! about NASA’S OSIRIS-Rex mission• OSIRIS-REx launched in 2016 to explore Bennu, which is the smallest body ever orbited by spacecraft.

• Studying Bennu will allow researchers to learn more about the origins of our solar system, the sources of water and organic molecules on Earth, the resources in near-Earth space, as well as improve our understanding of asteroids that could impact Earth.

• Bennu is the target of NASA’s Origins, Spectral Interpretation, Resource Identification, Security-Regolith Explorer (OSIRIS-REx) mission, which began orbiting the asteroid on December 31 last year.

• Bennu, which is only slightly wider than the height of the Empire State Building, may contain unaltered material from the very beginning of our solar system.

• Scientists have also discovered evidence of abundant water-bearing minerals on the surface of Bennu. Using early spectral data from NASA’s OSIRIS-REx spacecraft orbiting the asteroid, the team identified infrared properties similar to those in a type of meteorite called carbonaceous chondrites.

• Scientists are interested in the composition of Bennu because similar objects may have seeded the Earth with water and organic materials

Page 85: JOIN THE DOTS! - Career Launcher · dots! A current affairs series for UPSC Examination’ series which will help you pick up relevant news items of the day from various national

Page: 79 Join the dots! – March 2019

NITI Aayog to organise FinTech Conclave 2019

Relevance In – Prelims (about FinTech) + Mains (GS III economic developments)

What’s the NEWS• NITI Aayog is organising a day-long FinTech Conclave

• The objective is to shape India’s continued ascendancy in FinTech, build the narrative for future strategy and policy efforts, and to deliberate steps for comprehensive financial inclusion

• The Conclave will be featuring representatives from across the financial space – central ministries, regulators, bankers, startups, service providers and entrepreneurs.

Know! more about the conclave• Conclave will host more than 300 representatives from the leading Financial Institutions including HDFC

Bank, IndusInd, ICICI Bank, SBI Card, Tata Capital and FinTechs including BankBazaar, PhonePe, Capital Float, Zerodha, PayTM, MobiKwik, PayU, leading venture capital investors, state governments, MSMEs and industry subject matter experts.

• Government of India’s efforts focused on Digital India and developing India Stack including Voluntary Aadhaar for financial inclusion have evoked significant interest from various stakeholders in the area of Financial Technology (FinTech).

Know! more about India’s Fin Tech market• India is one of the fastest growing FinTech markets globally and industry research has projected that USD 1

Trillion or 60% of retail and SME credit, will be digitally disbursed by 2029.

• The Indian FinTech ecosystem is the third largest in the globe, attracting nearly USD 6 billion in investments since 2014.

• The Indian FinTech industry is creating cutting edge intellectual property assets in advanced risk management and artificial intelligence that will propel India forward in the global digital economy while simultaneously enabling paperless access to finance for every Indian.

Know! about Financial technology• Financial technology, often shortened to FinTech or fintech, is the new technology and innovation that aims to

compete with traditional financial methods in the delivery of financial services.

• It is an emerging industry that uses technology to improve activities in finance. The use of smartphones for mobile banking, investing services and cryptocurrency are examples of technologies aiming to make financial services more accessible to the general public.

Page 86: JOIN THE DOTS! - Career Launcher · dots! A current affairs series for UPSC Examination’ series which will help you pick up relevant news items of the day from various national

Join the dots! – March 2019 Page: 80

• Financial technology companies consist of both startups and established financial institutions and technology companies trying to replace or enhance the usage of financial services provided by existing financial companies.

• Many existing financial institutions are implementing Fintech solutions and technologies in order to improve and develop their services, as well as gaining an improved competitive stance.

Indian Forest Act, 1927 proposed amendments

Relevance IN – Prelims (about Indian forest act and new amendments) + Mains (GS III environment conservation)

What’s the NEWS

• The first draft of the comprehensive amendments to the Indian Forest Act, 1927 has been finalised by the Ministry of Environment, Forest and Climate Change (MoEF&CC)

• A proposed legislation accords significant powers to India’s forest officers — including the power issue search warrants, enter and investigate lands within their jurisdictions, and to provide indemnity to forest officers using arms to prevent forest-related offences.

• The Indian Forest Act, 2019 is envisaged as an amendment to the Indian Forest Act, 1927 and is an attempt to address contemporary challenges to India’s forests.

Know! about the amendments • Amendment proposed to provide indemnity to Forest-officer using arms etc, to prevent the forest offence

• Forest-officer not below the rank of a Ranger shall have power to hold an inquiry into forest offences and shall have the powers to search or issue a search warrant under the Code of Criminal Procedure, 1973

• Any Forest-officer not below the rank of a Forester may, at any time enter and inspect any land within his area of jurisdiction

• The draft amendment defines community as “a group of persons specified on the basis of government records living in a specific locality and in joint possession and enjoyment of common property resources, without regard to race, religion, caste, language and culture”

• Under the proposed amendment Forest is defined to include “any government or private or institutional land recorded or notified as forest/forest land in any government record and the lands managed by government/community as forest and mangroves, and also any land which the central or state government may by notification declare to be forest for the purpose of this Act.

• Village forests”, according to the proposed Act, may be forestland or wasteland, which is the property of the government and would be jointly managed by the community through the Joint Forest Management Committee or Gram Sabha.

• The amendment provides that if the state government, after consultation with the central government, feels that the rights under Forest Rights Act will hamper conservation efforts, then the state may commute such rights by providing compensation to maintain the social organisation of the forest dwelling communities

Page 87: JOIN THE DOTS! - Career Launcher · dots! A current affairs series for UPSC Examination’ series which will help you pick up relevant news items of the day from various national

Page: 81 Join the dots! – March 2019

Trump says US will recognize Israel’s sovereignty over Golan Heights: Israel captured Golan Heights from Syria after war in 1967

Relevance IN – Prelims (about Golan heights)

What’s the NEWS• US President Trump has announced that the US will

recognize Israel’s sovereignty over the Golan Heights.

• Previous US administrations have treated Golan Heights as occupied Syrian territory, in line with UN security council resolutions.

• By defying a 52-year-old unanimously adopted UN resolution on “inadmissibility of the acquisition of territory by war”, Trump has also broken the postwar norm of refusing to recognise the forcible annexation of territory – which has underpinned western and international opposition to the Russian annexation of Crimea.

Know! all about Golan Heights • Until 1967 Golan Heights was part of Syria. Israel

occupied the Golan Heights during the Six Day war (Third Arab Israeli war) held in 1967.

• Israel annexed the region unilaterally in 1981. This unilateral annexation was not recognised by the international community and the Golan Heights was seen as Occupied Syrian Territory.

Importance of Golan Heights for Israel• Israel argues that civil war in Syria demonstrates the need to keep the plateau as a buffer zone between Israeli

towns and the instability of its neighbour.

• Israel also fears that Iran is seeking to establish itself permanently on the Syrian side of the border in order to launch attacks on Israel.

• Israel advanced into the Golan Heights gradually in the years following the 1948 war Arab-Israeli war, and occupied it entirely in the 1967 war.

• That year, UN security council resolution 242 stressed the “inadmissibility of the acquisition of territory by war and the need to work for a just and lasting peace in which every state in the area can live in security”.

• United Nations Disengagement Observer Force (UNDOF) is stationed in camps and observation posts along the Golan. There is a 400-square-km “Area of Separation” called a demilitarized zone between the Israeli and Syrian armies.

The International Day for the Elimination of Racial Discrimination 2019• 2019 Theme: Mitigating and countering rising nationalist populism and extreme supremacist ideologies

Present context of racial discrimination • Racist extremist movements based on ideologies that seek to promote populist, nationalist agendas are spreading

in various parts of the world, fueling racism, racial discrimination, xenophobia and related intolerance, often targeting migrants and refugees as well as people of African descent.

Role of United Nations• In its recent resolution on eliminating racism, the United Nations General Assembly reiterated that all human

beings are born free and equal in dignity and rights and have the potential to contribute constructively to the development and well-being of their societies.

• The resolution also emphasized that any doctrine of racial superiority is scientifically false, morally condemnable, socially unjust and dangerous and must be rejected, together with theories that attempt to determine the existence of separate human races.

Page 88: JOIN THE DOTS! - Career Launcher · dots! A current affairs series for UPSC Examination’ series which will help you pick up relevant news items of the day from various national

Join the dots! – March 2019 Page: 82

• The report on glorification of Nazism online, identified recent trends and manifestations of glorification of Nazism, neo-Nazism and other practices that contribute to fuelling contemporary forms of racism, racial discrimination, xenophobia and related intolerance.

• The report highlighted States’ obligations under human rights law to counter such extreme ideologies online, as well as the responsibilities of technology companies in the light of human rights principles.

• The recent Christchurch Mosque Shootings in New Zealand is one such event. The International Day for the Elimination of Racial Discrimination aims to fight against all forms of racial intolerance.

Why it was proclaimed on 21 March• On March 21st, 1960 police opened re and killed 69 people at a peaceful demonstration against the apartheid

“pass laws” in Sharpeville, South Africa.

• The United Nations General Assembly in 1966 proclaimed March 21st as International Day for the Elimination of Racial Discrimination 2019 and called on the international community to redouble its efforts to eliminate all forms of racial discrimination.

Unlawful Activities Prevention Act (UAPA)

Relevance IN – Prelims(about UAPA) + Mains (GS III security challenges and their management)

What’s the NEWS• The Central Government has banned the Yasin Malik-led Jammu Kashmir Liberation Front (JKLF) as an

unlawful association under the provisions of Unlawful Activities Prevention Act (UAPA).

Know! about JKLF• Jammu Kashmir Liberation Front Jammu Kashmir Liberation Front (JKLF) is spearheading the separatist

ideology in the Kashmir valley and has been at the forefront of the separatist activities and violence since 1988.

• It is also alleged that JKLF was involved in murders of Kashmiri Pandits in 1989 which led to their exodus from the valley.

• It is also said that JKLF is posing a security threat to the country and is posing threat to the territorial integrity and sovereignty of India.

Know! about Unlawful Activities (Prevention) Act (UAPA), 1967• UAPA is anti-terrorist law aimed at effective prevention of unlawful activities associations in India.

• UAPA’s objective is to provide necessary powers to the investigating agencies to act against activities directed against the integrity and sovereignty of India.

• UAPA bans certain terrorist associations, penalises membership and association with such organizations and punishes terrorist activities.

• UAPA imposes reasonable restrictions on the exercise of freedom of speech and expression, to assemble peaceably without arms and to form associations in interests of sovereignty and integrity of India

Justice Pinaki Chandra Ghose takes oath as first Lokpal chief of India

Relevance IN – Prelims(about Lokpal)+ Mains (GS II and GS IV important aspects of governance, transparency and accountability)

What’s the NEWS• Justice Ghose, the former Supreme Court judge, was appointed as the country’s first Lokpal

• President Ram Nath Kovind administered the Oath of office to Justice Ghose after a long delay of five years.

• The Lokpal Act had received the assent of the President on January 1, 2014.

• The appointment comes over five years after the former United Progressive Alliance government passed the Lokpal and Lokayuktas Act, 2013.

• The Lokpal Act provides for an anti-corruption panel that will supervise cases of corruption against certain categories of public servants.

Page 89: JOIN THE DOTS! - Career Launcher · dots! A current affairs series for UPSC Examination’ series which will help you pick up relevant news items of the day from various national

Page: 83 Join the dots! – March 2019

Know! the recent developments• In September, the Centre had named an eight-member search committee, led by Desai and also comprising

former State Bank of India chief Arundhati Bhattacharya and Prasar Bharati chairperson A Surya Prakash.

• On January 17, the Supreme Court had asked the Lokpal search committee to recommend names for the office by the end of February.

• A three-judge bench led by Chief Justice of India Ranjan Gogoi asked the Centre to provide the search panel the infrastructure and manpower to help it complete its work. The Centre initiated the process to fill the long-vacant posts of chairperson and members of anti-corruption ombudsman on January 30.

Know! about the Lokpal (rules and regulations)• According to the rules, the applicant to the post of chairperson of the Lokpal must be a former chief justice of

India or a Supreme Court judge. The applicant for the post of a member should be a former Supreme Court judge or a chief justice of a High Court.

• The applicant must be at least 45 years of age. The rules also state that at least 50% of the members of the Lokpal should be persons belonging to the Scheduled Castes, Scheduled Tribes, backward classes and women.

• The chairperson and members will hold office for a term of five years or till they attain 70 years of age.

• The salary and allowances of the chairman are same as that of the Chief Justice of India. The salary and allowances of members are same as that of a judge of the Supreme Court

Know! about the members of Lokpal • The Lokpal act provides for a chairperson and a maximum of eight members in the Lokpal panel and of these,

four need to be judicial members.

Judicial Members of Lokpal • Former Chief Justices of different high courts Justices Dilip B Bhosale, Pradip Kumar Mohanty, Abhilasha

Kumari together with sitting Chief Justice of Chhattisgarh High Court Ajay Kumar Tripathi have been appointed as judicial members in the Lokpal.

Non-judicial members• The non-judicial members of the Lokpal are first woman chief of Sashastra Seema Bal Archana Ramasundaram,

ex-Maharashtra chief secretary Dinesh Kumar Jain, former IRS officer Mahender Singh and Gujarat cadre ex-IAS officer Indrajeet Prasad Gautam.

Young Scientist Programme (Yuvika)

Relevance IN – Prelims (about Yuvika)

What’s the NEWS

• Indian Space Research Organisation has launched a special programme for School Children called “Young Scientist Programme” “YUva VIgyani KAryakram” (;qfodk) from this year, in tune with the Government’s vision “Jai Vigyan, Jai Anusandhan”.

Page 90: JOIN THE DOTS! - Career Launcher · dots! A current affairs series for UPSC Examination’ series which will help you pick up relevant news items of the day from various national

Join the dots! – March 2019 Page: 84

Know! about the programme • The Program is primarily aimed at imparting basic knowledge on Space Technology, Space Science and Space

Applications to the younger ones with the intent of arousing their interest in the emerging areas of Space activities.

• The program is thus aimed at creating awareness amongst the youngsters who are the future building blocks of our Nation.

• This will further help them to appreciate what they are being taught in the school and its real application in Space Science & Technology. ISRO has chalked out this programme to “Catch them young”.

• The programme will be of around two weeks duration during summer holidays and the schedule will include invited talks, experience sharing by the eminent scientists, facility and lab visits, exclusive sessions for discussions with experts, practical and feedback sessions.

• It is proposed to select 3 students each from each State/ Union Territory to participate in this programme every year covering CBSE, ICSE and State syllabus. Those who have finished 8th standard and currently studying in 9th standard will be eligible for the programme.

• The selection is based on the academic performance and extracurricular activities, which is clearly mentioned in the selection criteria already circulated to Chief Secretaries of States/ Administrators of UTs. Students belong to the rural area have been given special weightage in the selection criteria.

Induction of Chinook Helicopters in Indian Air Force

Relevance IN – Prelims (about Chinook)

What’s the NEWS• The IAF formally inducted the CH 47 F(I)- Chinook heavy lift helicopters into its inventory at Air Force Station

Chandigarh.

• IAF had signed a contract with M/s Boeing Ltd in September 2015 for 15 Chinook helicopters.

• The first batch of four helicopters has been delivered on schedule and the last batch is to be delivered by March next year.

• These helicopters will be deployed in the Northern and Eastern regions of India.

Know! more about Chinook• The addition of heavy-lift CH 47 F(I) helicopter is a significant step towards modernisation of Indian Air Force’s

helicopter fleet.

• The helicopter has been customized to suit IAF’s future requirements and capability roadmap.

• The helicopter has a fully integrated digital cockpit management system, advanced cargo handling capabilities and electronic warfare suite that complement the aircraft’s performance.

• The helicopter is capable of airlifting diverse military and non military loads into remote locations.

Page 91: JOIN THE DOTS! - Career Launcher · dots! A current affairs series for UPSC Examination’ series which will help you pick up relevant news items of the day from various national

Page: 85 Join the dots! – March 2019

• The aircraft is all-weather capable and state-of-the-art night vision goggles will permit all operations even at night, he informed.

• The Chinook, with its operating ceiling of 20,000 feet, will redefine heli-lift not only in operations like inter-valley transport of troops, airlifting artillery guns and heavy under slung loads for the Border Roads Organisation, but also for humanitarian assistance

Japan gives UN World Food Programme $69m

Relevance IN – Prelims(about UN’s WEF programme)

What’s the NEWS• Japan has donated 69 million dollars to the United Nations World Food Programme to provide vital aid to 28

countries in the Middle East, Africa, and Asia, with the biggest shares of the money earmarked for Yemen and Iraq, WFP said Thursday in a statement.

Know! about United Nation’s World Food Programme • The World Food Programme (WFP) is the food-assistance branch of the United Nations and the world’s largest

humanitarian organization addressing hunger and promoting food security.

• According to the WFP, it provides food assistance to an average of 91.4 million people in 83 countries each year.

• From its headquarters in Rome and from more than 80 country offices around the world, the WFP works to help people who cannot produce or obtain enough food for themselves and their families. It is a member of the United Nations Development Group

Know! about the inception of WEP• In the 1960 Food and Agriculture Organization (FAO) Conference, there were calls for setting up a multilateral

food aid programme.

• In line these demands, the World Food Programme was established in 1961 by the FAO and the United Nations General Assembly.

• The Food for Work programmes of the World Food Programme promotes environmental and economic stability and agricultural production.

• The World Food Programme strives to eradicate hunger and malnutrition, with the ultimate goal to eliminate the need for food aid itself.

• The objectives of the World Food Programme are: Save lives and protect livelihoods in emergencies. Support food security and nutrition and (re)build livelihoods in fragile settings and following emergencies.

• Reduce risk and enable people, communities and countries to meet their own food and nutrition needs. Reduce under-nutrition and break the intergenerational cycle of hunger.

• Zero Hunger in 2030. World Food Programme also aims to fight micronutrient deficiencies, reduce child mortality, improve maternal health, and combat disease, including HIV and AIDS

World Meteorological Day 2019

Page 92: JOIN THE DOTS! - Career Launcher · dots! A current affairs series for UPSC Examination’ series which will help you pick up relevant news items of the day from various national

Join the dots! – March 2019 Page: 86

What’s the NEWS• The World Meteorological Day was observed on March 23

Know! about the theme

• The theme “The Sun, the Earth and the weather”. The theme recognises the critical role the Earth’s nearest star plays in what transpires on the planet.

• The energy from the Sun is taken up by all life forms to perform their biological functions. The Solar energy also regulates global climate, which, in turn, influences local weather conditions and the same energy is also absorbed by the oceans which carry it around the planet, creating further conditions for diverse marine life to exist.

World Meteorological Day • World Meteorological Day is observed on March 23rd every year to commemorate the coming into force of the

Convention establishing the World Meteorological Organization on 23 March 1950.

• The World Meteorological Day showcases the essential contribution of National Meteorological and Hydrological Services to the safety and wellbeing of society and is celebrated with activities around the world.

• WMO provides world leadership and expertise for international cooperation in the delivery and use of high quality, authoritative weather, climate, hydrological and related environmental services by its Members, for the improvement of the well-being of societies of all nations.

World Tuberculosis Day 2019

Relevance IN – Prelims (about World TB day) + Mains (GS II issues relating to development and management of social sector/services relating to health)

What’s the NEWS• Each year, we commemorate World Tuberculosis

(TB) Day on March 24 to raise public awareness about the devastating health, social and economic consequences of TB, and to step up efforts to end the global TB epidemic.

• The date marks the day in 1882 when Dr Robert Koch announced that he had discovered the bacterium that causes TB, which opened the way towards diagnosing and curing this disease.

Know! some statics about TB• TB remains the world’s deadliest infectious killer.

Each day, nearly 4500 people lose their lives to TB and close to 30,000 people fall ill with this preventable and curable disease.

• Global efforts to combat TB have saved an estimated 54 million lives since the year 2000 and reduced the TB mortality rate by 42%.

• To accelerate the TB response in countries to reach targets – Heads of State came together and made strong commitments to end TB at the first-ever UN High Level Meeting in September 2018.

• The theme of World TB Day 2019 - ‘It’s time’ – puts the accent on the urgency to act on the commitments made by global leaders

• The theme of World Tuberculosis Day this year is “It’s Time”. Consistent with the spirit of this theme, India has renewed its commitments and intentions to end TB by 2025, five years ahead of the global targets.

Page 93: JOIN THE DOTS! - Career Launcher · dots! A current affairs series for UPSC Examination’ series which will help you pick up relevant news items of the day from various national

Page: 87 Join the dots! – March 2019

• This is an ambitious timeline and, given our motivation to eliminate the disease, it is achievable.

• The World Health Organization (WHO) has launched a joint initiative “Find. Treat. All. #EndTB” with the Global Fund and Stop TB Partnership, with the aim of accelerating the TB response and ensuring access to care, in line with WHO’s overall drive towards Universal Health Coverage.

Know! about TB• Tuberculosis Tuberculosis (TB) is an infectious disease caused by bacteria Mycobacterium tuberculosis (MTB).

• Tuberculosis affects the lungs and can also infect other parts of the body.

• Early symptoms of TB include bad cough which lasts for 3 weeks or longer. It causes pain in the chest and one might also end up coughing blood in case of tuberculosis

PSLV-C45 project will mark several firsts for ISRO: Emisat mission scheduled for launch on April 1

Relevance IN – Prelims (about PSLV -C 45)

What’s the NEWS• The PSLV-C45/Emisat mission scheduled for an April 1 lift-off from Sriharikota will be a memorable one for the

Indian Space Research Organisation (ISRO).

Know! about the uniqueness of the mission • For one, it will be ISRO’s first attempt at placing payloads in three different orbits.

• The chief payload — the 436 kg Emisat — will be injected into a 749 km orbit. After that, the fourth stage of the rocket will be manoeuvred to a 504 km orbit for releasing 28 international satellites.

• Once that job is over, the fourth stage will be restarted and guided to an altitude of 485 km.

• For the next six months, this stage will serve as an orbital platform for space-based experiments. This is another first for the ISRO. Normally, the spent stage simply becomes space junk.

• The orbital platform will also sport solar panels, which too is a first,

• The launch vehicle itself is a new variant, designated PSLV-QL. For the first time, ISRO will be employing four XL strap-on motors on the first stage.

• Again, the PSLV-C45 mission marks a milestone for ISRO’s Indian Institute of Space Science and Technology (IIST). One of the three experiments aboard the orbital platform is the IIST’s Advanced Retarding Potential Analyser for Ionospheric Studies (ARIS).

Page 94: JOIN THE DOTS! - Career Launcher · dots! A current affairs series for UPSC Examination’ series which will help you pick up relevant news items of the day from various national

Join the dots! – March 2019 Page: 88

• This is the first time that an IIST payload is flying aboard an ISRO mission, ARIS will study the structure and composition of the ionosphere.

• The other two experimental payloads aboard the orbital platform are the Automatic Identification System (AIS), an ISRO payload for maritime satellite applications, and the Automatic Packet Repeating System (APRS), meant to assist amateur radio operators.

Know! about Emisat

• Emisat, the chief payload on PSLV-C45, is meant for electromagnetic spectrum measurements, according to the ISRO.

• It will released into an orbit at 749 km, the C-45, which is set for lift-off from the second launchpad at Sriharikota, will mark the 47th flight of the PSLV.

• The 436-kg satellite would serve as the country’s roving device for detecting and gathering electronic intelligence from enemy radars across the borders as it circles the globe roughly pole to pole every 90 minutes or so

Foreign co-passengers

• As many as 28 small foreign co-passenger satellites will also travel to space with it, but to a lower orbit at 504 km.

• They include 24 small satellites from the U.S., among them 20 which are part of previous customer Planet Labs’ earth observation constellation. The other four customers are from Lithuania, Spain and Switzerland.

The Global Energy Transition index 2019: India ranks 76th on WEF’s global Energy Transition index; Sweden on top

Relevance IN (Prelims – about WEF) + Mains (GS III energy conservation)

What’s the NEWS

• The Global Energy Transition index 2019 report has been released by the World Economic Forum (WEF).

• The index compares the energy sectors of 115 countries and analyses their readiness for energy transition.

• The index benchmarks the countries energy systems based on an “energy triangle”, comprised of energy security and access, economic development and growth, environmental sustainability and how well they are set-up to succeed in the future.

Page 95: JOIN THE DOTS! - Career Launcher · dots! A current affairs series for UPSC Examination’ series which will help you pick up relevant news items of the day from various national

Page: 89 Join the dots! – March 2019

• The index takes into account six individual indicators: capital and investment, regulation and political commitment, institutions and governance, institutions and innovative business environment, human capital and consumer participation, and energy system structure.

Know! about the findings of WEF report w.r.t India

• India is amongst the countries with high pollution levels and has a relatively high CO2 intensity in its energy system

• India has moved up two places to rank 76th on a global energy transition index, which has ranked 115 economies on how well they are able to balance energy security and access with environmental sustainability and affordability.

• Sweden remains on the top on this annual list compiled by Geneva-based World Economic Forum (WEF) and is followed by Switzerland and Norway in the top three, as per its latest report released on Monday.

• The WEF said energy systems have globally become less affordable and less environmentally sustainable than they were five years ago, though access to energy has improved with less than 1 billion now living without access to electricity.

• It said India is amongst the countries with high pollution levels and has a relatively high CO2 intensity in its energy system.

• “Despite this, India has made significant strides to improve energy access in recent years, and currently scores well in the area of regulation and political commitment towards energy transition

• While India has scored low in terms of system performance (ranking 97 and 86, respectively), it ranks considerably higher when it comes to readiness (45 and 61, respectively). Overall, India has moved up two places from 78th last year.

• China is ranked even lower than India at 82nd position, though it ranks very high at seventh place in the world for regulation and political commitment.

• Despite its low ranking, India is the second best in the BRICS block of emerging economies, with Brazil being the best at 46th place globally. However, India is the only amongst the five economies to improve its rank since last year.

Know! more about the findings of the report

• The WEF said its index considers both the current state of the countries’ energy system and their structural readiness to adapt to future energy needs.

• Small economies have achieved higher scores on readiness, with the UK being the only G7 economy in the top 10.

• The biggest challenge facing attempts to future proof global energy is the lack of readiness among the world’s largest emitters. The 10 countries that score the highest in terms of readiness account for a mere 2.6 per cent of global annual emissions, the study found.

• It said continued use of coal for power generation in Asia, increasing commodity prices and slower-than-needed improvements in energy intensity have contributed to this year’s stagnation in performance.

• The ‘transition readiness’ component of the index has taken into account six individual indicators: capital and investment, regulation and political commitment, institutions and governance, institutions and innovative business environment, human capital and consumer participation, and energy system structure.

• The WEF said fossil fuels’ share of total primary energy supply at 81 per cent has been constant over the past three decades.

• Also, the global CO2 emissions are expected to have increased by more than 2 per cent in 2018, the highest since 2014.

Page 96: JOIN THE DOTS! - Career Launcher · dots! A current affairs series for UPSC Examination’ series which will help you pick up relevant news items of the day from various national

Join the dots! – March 2019 Page: 90

AUSINDEX: Australia, India’s Naval Exercise To Focus On Anti-Submarine Warfare

Relevance IN – Prelims (about AUSINDEX)

What’s the NEWS• Third instalment of the

“AUSINDEX” exercise will be held off the coast of Vishakhapatnam from April 2 to 16 and will see the largest Australian defence forces group deployment in India.

• The joint exercise will focus on anti-submarine warfare, as part of efforts to ramp up bilateral naval cooperation in the strategic Indian Ocean region.

Know! more about AUSINDEX 2019 • Over 1,000 Australian

defence personnel, HMAS Canberra (landing helicopter dock), Auxiliary Oiler Replenishment vessel, and two frigates -- HMAS Newcastle and HMAS Parramatta -- will take part in the exercise.

• Another interesting feature of the exercise will be the deployment of Australian personnel on Indian ships and vice versa. Also, P8I and P8 maritime patrol aircraft will be involved in the exercise.

Pakistan Approves Plan to Open Sharda Temple Corridor in PoK for Hindu Pilgrims: Highlight

The Sharda Peeth corridor, when opened, will be the second religious tract after Kartarpur corridor in Pakistan-controlled territory that will connect the two neighbouring nations

Page 97: JOIN THE DOTS! - Career Launcher · dots! A current affairs series for UPSC Examination’ series which will help you pick up relevant news items of the day from various national

Page: 91 Join the dots! – March 2019

Relevance IN – Prelims (about sharda peeth corridor) + Mains (GS II bilateral relations)

What’s the NEWS• The Pakistan government approved a proposal to establish a corridor that will allow Hindu pilgrims from India

to visit Sharda Peeth, an ancient Hindu temple and cultural site in Pakistan-occupied Kashmir.

• The Sharda Peeth corridor, when opened, will be the second religious tract after Kartarpur corridor in Pakistan-controlled territory that will connect the two neighbouring nations.

• India had already sent a proposal to Pakistan to open the temple corridor

Know! about Sharda Peeth• Established in 237 B.C. during the reign of Emperor Ashoka, the 5,000-year-old Sharada Peeth is an abandoned

temple and ancient centre of learning dedicated to the Hindu goddess of learning.

• Between the 6th and 12th centuries C.E, Sharada Peeth was one of the foremost temple universities of the Indian subcontinent.

• Sharada Peeth is believed to be a foremost centre of learning Between the 6th and 12th centuries CE hosting scholars such as Kalhana, Adi Shankara, Vairostana, Kumarajiva, and Thonmi Sambhota.

• The temple is about 150 km from Muzaffarabad capital of PoK and 130 Km from Srinagar currently under the control of Pakistan.

• It is also one of the three famous holy sites for Kashmiri Pandits, the other two being the Martand Sun Temple in Anantnag and the Amarnath temple.

Know! about Kartarpur corridor • In November last year, the Pakistani premier had laid the foundation stone for the 4-km Kartarpur corridor at

Shakargarh in Narowal district of Pakistan’s Punjab province.

• The corridor is expected to be completed by 2019 and will connect Darbar Sahib in Pakistan’s Kartarpur -- the final resting place of Sikh faith’s founder Guru Nanak Dev -- with Dera Baba Nanak shrine in India’s Gurdaspur district and facilitate visa-free movement of Indian Sikh pilgrims, who will have to just obtain a permit to visit Kartarpur Sahib, which was established in 1522 by Guru Nanak Dev.

‘Mission Shakti’, India’s homegrown anti-satellite missile: India 4th nation to enter elite space power club with anti-satellite weapon, India had demonstrated anti-satellite missile capability by shooting down a live low earth orbit satellite.

Relevance IN – Prelims (about mission shakti) + Mains (GS III awareness in the field of space)

Page 98: JOIN THE DOTS! - Career Launcher · dots! A current affairs series for UPSC Examination’ series which will help you pick up relevant news items of the day from various national

Join the dots! – March 2019 Page: 92

What’s the NEWS

• Mission Shakti, as the operation is known, was marked successful after it destroyed a test satellite 300 kilometers in space.

Know about Mission Shakti

• It successfully tested India’s first A-SAT weapon system, destroying a test satellite 300 km in space.

• The A-SAT is entirely made in India, by the Defence Research and Development Organisation (DRDO).

• The mission destroyed the test satellite with “remarkable precision” in just three minutes.

• It makes India is the fourth country to successfully test an Anti-Satellite weapon system, after the US, Russia and China.

• Target destroyed by India’s A-SAT missile was an out-of-service Indian satellite

• Since the weaponising of space is prevented by the Outer Space Treaty of 1967 now, the technology has been in development since as early as the 1950s. The Soviet Union had, in fact, tested its first A-SAT system in 1958.

Know! how ASAT weapons work

• A-SATs are space weapons that are designed to destroy satellites for strategic military purpose.

• A-SAT weapons are basically long-range missiles with a kinetic kill vehicle attached to them. They can be locked on to the target, intercept, engage and destroy in one swoop.

• The range of these A-SAT missiles depends on their launch location. The US and Russia have experimented with ship, land and space launches. India appears to have used land.

• India’s ABM program, used in missile systems like the Agni, could have been tweaked into an A-SAT weapon

• ISRO now has the capability to launch two tonnes of payload, an important prerequisite for launching any weapon system.

• A-SAT projects were adapted for ABM use. The ABMs could also be used to develop A-SAT missiles, Saraswat said in 2012.

• India has used a three-stage Ballistic Missile Defence program (BMD) to test its A-SAT system.

Few more points on Mission Shakti

• India conducted Mission Shakti, an anti-satellite missile test, from the Dr A P J Abdul Kalam Island launch complex.

• This was a technological mission carried out by the DRDO. The satellite used in the mission was one of India’s existing satellites operating in lower orbit.

• The test was fully successful and achieved all parameters as per plans. The test required an extremely high degree of precision and technical capability.

• The significance of the test is that India has tested and successfully demonstrated its capability to interdict and intercept a satellite in outer space based on complete indigenous technology.

• With this test, India joins an exclusive group of space-faring nations consisting of the USA, Russia and China.

• The DRDO’s Ballistic Missile Defence interceptor was used, which is part of the ongoing ballistic missile defence programme.

• The test was done in the lower atmosphere to ensure that there is no space debris. Whatever debris is generated will decay and fall back on to the earth within weeks. The test was done to verify that India has the capability to safeguard our space assets.

• No country in the world has so far used anti-satellite weapons against satellites belonging to another country. The US, Russia and China - and now, India - all blew up their own satellites. The US and Russia have also shot down their own defunct satellites on multiple occasions.

Page 99: JOIN THE DOTS! - Career Launcher · dots! A current affairs series for UPSC Examination’ series which will help you pick up relevant news items of the day from various national

Page: 93 Join the dots! – March 2019

India’s carbon dioxide emissions up 5%: IEA report shows China, U.S. & India together accounted for nearly 70% of the rise in energy demand

Relevance IN – Prelims(about IEA and report findings) + Mains (GS III environment conservation)

What’s the NEWS

• India emitted 2,299 million tonnes of carbon dioxide in 2018, a 4.8% rise from the previous year, according to a report by the International Energy Agency (IEA).

Know! about the report findings

• India’s emissions growth was higher than that of the United States and China — the two biggest emitters in the world — and this was primarily due to a rise in coal consumption.

• China, the United States, and India together accounted for nearly 70% of the rise in energy demand.

• India’s per capita emissions were about 40% of the global average and contributed 7% to the global carbon dioxide burden. The U.S., the largest emitter, was responsible for 14%.

• As per its commitments to the United Nations Framework Convention on Climate Change, India has promised to reduce the emissions intensity of its economy by 2030, compared to the 2005 levels.

• It has also committed to having 40% of its energy from renewable sources by 2030 and, as part of this, install 100 GW of solar power by 2022.

• The IEA report, showed that India’s energy intensity improvement declined 3% from the previous year even as its renewable energy installations increased 10.6%.

Reasons for rise in energy demand

• Reason Global energy consumption in 2018 increased at nearly twice the average rate of growth since 2010, driven by a robust global economy and higher heating and cooling needs in some parts of the world.

• Demand for all fuels increased, led by natural gas, even as solar and wind posted double digit growth.

• Higher electricity demand was responsible for over half of the growth in energy needs.

• Energy efficiency saw lacklustre improvement. As a result of higher energy consumption, carbon dioxide emissions rose 1.7% last year and hit a new record, the authors of the report said in a press statement.

• The United States had the largest increase in oil and gas demand worldwide. Gas consumption jumped 10% from the previous year, the fastest increase since the beginning of IEA records in 1971.

Page 100: JOIN THE DOTS! - Career Launcher · dots! A current affairs series for UPSC Examination’ series which will help you pick up relevant news items of the day from various national

Join the dots! – March 2019 Page: 94

Know! about IEA• The International Energy Agency is a Paris-based autonomous intergovernmental organization

established in the framework of the Organisation for Economic Co-operation and Development (OECD) in 1974 in the wake of the 1973 oil crisis.

• The IEA was initially dedicated to responding to physical disruptions in the supply of oil, as well as serving as an information source on statistics about the international oil market and other energy sectors.

• The IEA acts as a policy adviser to its member states, but also works with non-member countries, especially China, India, and Russia.

• The Agency’s mandate has broadened to focus on the “3Es” of effectual energy policy: energy security, economic development, and environmental protection. The latter has focused on mitigating climate change.

• The IEA has a broad role in promoting alternate energy sources (including renewable energy), rational energy policies, and multinational energy technology co-operation.

Govt. notifies new rules for drugs, clinical trials

Relevance IN – Prelims (about the new drugs and clinical trail rules)

What’s the NEWS• The Union Health Ministry has notified the Drugs and Clinical Trials Rules, 2019, with the government stating

that the move is aimed at promoting clinical research in the country.

• The rules will apply to all new drugs, investigational new drugs for human use, clinical trials, bio-equivalence studies and ethics committees.

Know! more about the notifications• The highlights of the notification include reduction in time for approving applications, which has now come

down to 30 days for drugs manufactured in India and 90 days for those developed outside the country.

• In case of no communication from Drugs Controller General of India, the application will be deemed to have been approved

• As per the new rule, the requirement of a local clinical trial may be waived for approval of a new drug if it is approved and marketed in any of the countries (EU, U.K., Australia, Japan and U.S.) specified by the Drugs Controller General with the approval of the government.

• The new rules will ensure patient safety and an ethics committee will monitor the trials and decide on the amount of compensation in cases of adverse events

Opening Ceremony Indo-Sri Lanka Joint Exercise Mitra Shakti-VI

Relevance IN – Prelims (about the exercise)

What’s the NEWS• Opening ceremony of Exercise MITRA

SHAKTI VI, a 14 days joint training exercise of the Indian Army and the Sri Lankan Army has been held on 27 March at Diyatalawa Parade Ground in Diyatalawa, Badulla District, Sri Lanka.

• This is the sixth edition of the joint exercise between the two nations. The exercise is being conducted from 26 March to 08 April 19.

Page 101: JOIN THE DOTS! - Career Launcher · dots! A current affairs series for UPSC Examination’ series which will help you pick up relevant news items of the day from various national

Page: 95 Join the dots! – March 2019

Know! more about the exercise• The Indian Army contingent comprises of a company group from BIHAR Regiment and a similar strength from

the First Gemunu Watch Battalion of Sri Lankan Army.

• The primary focus of the exercise is to train and equip the contingents to undertake joint counter insurgency and counter terrorist operations in urban/ rural environment under the United Nations flag.

• The exercise provides an ideal platform for both contingents to share their operational experience and expertise while being instrumental in broadening interoperability and cooperation between the armies of India and Sri Lanka.

Cabinet approves five year’s extension of Biomedical Research Career Programme

Relevance IN – Prelims (about the programme) + Mains (GS III awareness in the field of biotechnology)

What’s the NEWS• The Union Cabinet has approved the continuation of the Biomedical Research Career Programme (BRCP),

and Wellcome Trust (WT) / DBT India Alliance beyond its initial 10-year term (2008-09 to 2018-29) to a new five year phase (2019-20 to 2023-24) with Department of Biotechnology (DBT) increasing its commitment to two times that of WT.

Know! about the programme • Over its 10 years of funding in a 1:1partnership, the Programme has fulfilled its objectives of building and

nurturing talent of highest global standards in cutting-edge biomedical research in India, which has led to important scientific breakthroughs and applications to meet societal needs.

• BRCP has made it attractive for high quality Indian scientists working abroad to return to India, and has increased the number of locations geographically within India where world-class biomedical research is undertaken.

• In the extending phase, the Programme would continue to build this capacity as also strengthen clinical research and work towards addressing important health challenges for India. Continuation of the Programme with increased stake from the Government of India is important to bring about these returns.

Know! about Wellcome trust• The Wellcome Trust is a biomedical research charity based in London, United Kingdom. It was established in

1936 with legacies from the pharmaceutical magnate Sir Henry Wellcome to fund research to improve human and animal health.

• The aim of the Trust is to “achieve extraordinary improvements in health by supporting the brightest minds”, and in addition to funding biomedical research it supports the public understanding of science.

• It has an endowment of £25.9 billion (2018) making it the third wealthiest charitable foundation in the world, after the Bill & Melinda Gates Foundation and the INGKA Foundation.

• The Trust has been described by the Financial Times as the United Kingdom’s largest provider of non-governmental funding for scientific research and one of the largest providers in the world.

Electoral bonds will affect transparency, EC tells SC

Relevance IN – Prelims (about electoral bonds) + Mains (GS II important aspects of governance, transparency and accountability + GS IV Ethics, Integrity and Aptitude)

What’s the NEWS• The Election Commission of India (ECI) has told the Supreme Court that electoral bonds, contrary to government

claims, wreck transparency in political funding.

• The ECI ripped apart amendments made to various key statutes through the two consecutive Finance Acts of 2016 and 2017.

Page 102: JOIN THE DOTS! - Career Launcher · dots! A current affairs series for UPSC Examination’ series which will help you pick up relevant news items of the day from various national

Join the dots! – March 2019 Page: 96

• The amendments would pump in black money for political funding through shell companies and allow “unchecked foreign funding of political parties in India which could lead to Indian politics being influenced by foreign companies.

Know! about the amendments done in the finance act

• The Finance Act of 2017 amends various laws, including the Representation of the People (RP)Act of 1951, the Income Tax Act and the Companies Act.

• The Finance Act of 2016 makes changes in the Foreign Contribution (Regulation) Act of 2010.

Know! all about the provisions mentioned in the electoral bonds and various amendments done by the government for making the donation and provisions ambiguous

• The amendment to the RP Act allows political parties to skip recording donations received by them through electoral bonds in their contribution reports to the ECI.

• This is a retrograde step as far as transparency of donations is concerned

• The ECI has no way to ascertain whether the donations were received illegally by the political party from government companies or foreign sources.

• The ECI said the amendment introduced by the government in the Income Tax Act allows anonymous donations.

• Donors to political parties need not provide their names, address or PAN if they have contributed less than Rs. 20,000. Now, “many political parties have been reporting a major portion of the donations received as being less than the prescribed limit of Rs. 20,000

• The Finance Act of 2016, had amended the FCRA 2010 to “allow donations to be received from foreign companies having majority stake in Indian companies

• The earlier 7.5% ceiling on the political donations by companies has been removed and even companies which make losses can now donate via electoral bonds.

Know! all about Electoral bond

What is it?

• An electoral bond is designed to be a bearer instrument like a Promissory Note — in effect, it will be similar to a bank note that is payable to the bearer on demand and free of interest. It can be purchased by any citizen of India or a body incorporated in India.

How do you use it?

• The bonds will be issued in multiples of ₹1,000, ₹10,000, ₹1 lakh, ₹10 lakh and ₹1 crore and will be available at specified branches of State Bank of India.

• They can be bought by the donor with a KYC-compliant account. Donors can donate the bonds to their party of choice which can then be cashed in via the party’s verified account within 15 days.

What are the other conditions?

• Every party that is registered under section 29A of the Representation of the Peoples Act, 1951 (43 of 1951) and has secured at least one per cent of the votes polled in the most recent Lok Sabha or State election will be allotted a verified account by the Election Commission of India.

• The bonds will be available for purchase for a period of 10 days each in the beginning of every quarter, i.e. in January, April, July and October as specified by the Central Government. An additional period of 30 days shall be specified by the Central Government in the year of Lok Sabha elections.

• The electoral bonds will not bear the name of the donor. In essence, the donor and the party details will be available with the bank, but the political party might not be aware of who the donor is.

• The maximum amount of cash donation that a political party can receive be capped at ₹2,000 and that parties be entitled to receive donations by cheque or digital mode, in addition to electoral bonds.

Page 103: JOIN THE DOTS! - Career Launcher · dots! A current affairs series for UPSC Examination’ series which will help you pick up relevant news items of the day from various national

Page: 97 Join the dots! – March 2019

In 2018, Official Secrets Act invoked in five cases

Relevance IN – Prelims (about the official secret act) + Mains (GS III internal security challenges)

What’s the NEWS• The Ministry of Home Affairs (MHA) issued five

prosecution sanction orders last year under the Official Secrets Act (OSA), 1923.

• The colonial-era law meant for ensuring secrecy and confidentiality in governance, mostly on national security and espionage issues, has often been cited by authorities for refusing to divulge information.

• Governments have also faced criticism for misusing the law against journalists and whistleblowers.

Know! about the Act• The Official Secrets Act was first enacted in 1923 and

was retained after Independence.

• The law, applicable to government servants and citizens, provides the framework for dealing with espionage, sedition, and other potential threats to the integrity of the nation.

• The law makes spying, sharing ‘secret’ information, unauthorised use of uniforms, withholding information, interference with the armed forces in prohibited/restricted areas, among others, punishable offences.

• If guilty, a person may get up to 14 years’ imprisonment, a fine, or both.

• The OSA does not define “secret” or “official secrets”. Public servants could deny any information terming it a “secret” when asked under the RTI Act.

• Another contentious issue with the law is that its Section 5, which deals with potential breaches of national security, is often misinterpreted.

• The Section makes it a punishable offence to share information that may help an enemy state. The Section comes in handy for booking journalists when they publicise information that may cause embarrassment to the government or the armed forces.

• Several countries, including the United Kingdom, Malaysia, Singapore, and New Zealand, continue to use the legislation to protect state secrets. In 2001, Canada replaced its OSA with a Security of Information Act. The “official secrets” come under the Espionage Act in the U.S.

Successful anti-satellite missile test puts India in elite club (some more information regarding Mission Shakti)

What’s the NEWS• India successfully conducted an Anti-Satellite (ASAT) missile test, named Mission Shakti, becoming the fourth

country in the world to demonstrate the capability to shoot down satellites in orbit. So far, only the United States, Russia and China have this prowess.

• The satellite downed by the ASAT missile was Microsat-R, an imaging satellite which was launched into orbit on January 24, 2019 using a Polar Satellite Launch Vehicle (PSLV)

Page 104: JOIN THE DOTS! - Career Launcher · dots! A current affairs series for UPSC Examination’ series which will help you pick up relevant news items of the day from various national

Join the dots! – March 2019 Page: 98

• India has built the broad capabilities and building blocks to develop ASAT missiles for some time as part of its Ballistic Missile Defence (BMD) programme.

• A BMD interceptor missile successfully engaged an Indian orbiting target satellite in LEO in a ‘hit to kill’ mode’

• The ASAT missile was a modified exo-atmospheric interceptor missile of the BMD. A LEO of 300 km was chosen to “minimise” debris and it also won’t last more than a few months

• Anti-satellite weapons provide the capability to shoot down enemy satellites in orbit thereby disrupting critical communications and surveillance capabilities.

• ASAT missiles also act as a space deterrent in dissuading adversaries from targeting the country’s satellite network.

Know! about India’s ASAT system• While Mission Shakti may have targeted an object in outer space, India has long developed the ability to

intercept incoming missiles.

• In 2011, a modified Prithvi missile, mimicked the trajectory of a ballistic missile with a 600-km range.

• Radars at different locations swung into action, tracking the “enemy” missile, constructing its trajectory and passing on the information in real time to the Mission Control Centre (MCC) to launch the interceptor, an Advanced Air Defence (AAD) missile.

• It had a directional warhead to go close to the adversarial missile before exploding to inflict damage on it.

Know! about low earth orbit satellites• The Indian satellite that was shot down was a Low Earth Orbit (LEO) satellite.

• These are satellites roughly at an altitude of 2,000 kilometres from the earth and that’s the region where the majority of satellites are concentrated.

Assam launches initiative to educate votersWhat’s the NEWS• Folk song competitions, quiz programmes and appeals by celebrities are some of the initiatives being undertaken

by the election office in Assam for attracting women, first-time voters, people with physical disabilities and senior citizens to cast their votes.

Know! more about the initiative• District election officers have been directed to reach out to the maximum number of voters in the constituencies

under their jurisdiction

• The local aspects of people should be taken care while chalking out the programmes under the Systematic Voters’ Education and Electoral Practices and the aim is to reach out to all sections of voters

• A major component of the SVEEP is to remove gender gap in polls and an initiative ‘Aaideur Chora’ has been taken up in collaboration with the Assam State Rural Livelihood Mission and Assam State Urban Livelihood Mission to sensitise female voters.

• ‘Chandraprabha’, a woman mascot, will be used for spreading the message of participation in the local language.

• The nodal officers of the SVEEP campaigns are organising ‘naam’ (folk song) singing, ‘pitha’ and ‘laru’ (Assamese sweets) making competitions and street plays to involve women in the awareness campaigns.

• Polling stations with low female voter turn-out have been identified and ‘mahila voter rallies’ are being held

India in pact to ease U.S. firms’ compliance

Relevance IN – Prelims (about the benefits of CbC) + Mains (GS II bilateral relations affecting India’s intrest)

What’s the NEWS• India and the U.S. signed an inter-government agreement for the automatic exchange of country-by-country

(CbC) reports, which will reduce the compliance burden for Indian subsidiary companies of U.S. parent companies.

• This is a key step in making India compliant with the Base Erosion and Profit Shifting (BEPS) project, of which it is an active participant.

Page 105: JOIN THE DOTS! - Career Launcher · dots! A current affairs series for UPSC Examination’ series which will help you pick up relevant news items of the day from various national

Page: 99 Join the dots! – March 2019

Know! more about the agreement• This Agreement for Exchange of CbC Reports, along with the Bilateral Competent Authority Arrangement

between the two competent authorities, will enable both the countries to automatically exchange CbC reports filed by the ultimate parent entities of multinational enterprises (MNE) in the respective jurisdictions

• The Base Erosion and Profit Shifting (BEPS) Action Plan adopted by the Organisation for Economic Co-operation and Development (OECD) and G20 countries in 2013 recognised that the way forward to mitigate risk from base erosion and profit shifting was to enhance transparency.

• The signing India-U.S. agreement for exchange of CbC information is a huge respite for subsidiaries of U.S. head-quartered companies.

• The signing of the agreement further revalidates the keen willingness of Indian and U.S. tax authorities to engage and amicably resolve issues for taxpayers.

Know! about Base erosion and profit shifting• Base erosion and profit shifting refers to the activities of multinational corporations to shift their

profits from high tax jurisdictions to lower tax jurisdiction, thereby eroding the tax base of the high tax jurisdictions and depriving them of tax revenue.

• In order to combat this, many countries entered into agreements to share tax information with each other to enhance transparency and make such profit shifting that much harder.

The Union Cabinet has approved the creation of six additional posts in the National Company Law Appellate Tribunal (NCLAT)

Relevance IN – Prelims (about NCLT) + Mains (GS III economic developments)

What’s the NEWS• The proposal involves the creation of three additional posts each for judicial members and technical members in

the NCLAT.

• The creation of additional posts will help meet the mandate provided to NCLAT by the Finance Act 2017, the Companies Act 2013 and the Insolvency and Bankruptcy Code 2016.

National Company Law Appellate Tribunal• National Company Law Appellate Tribunal (NCLAT) was established under Section 410 of the Companies Act,

2013 for hearing appeals against the orders of National Company Law Tribunal (NCLT).

• NCLAT is the Appellate Tribunal for hearing appeals against the orders passed by NCLT under the Insolvency and Bankruptcy Code, 2016 (IBC).

• NCLAT hears appeals against the orders passed by Insolvency and Bankruptcy Board of India.

• NCLAT is also the Appellate Tribunal to hear and dispose of appeals against any direction issued or decision made or order passed by the Competition Commission of India (CCI) – as per the amendment brought to Section 410 of the Companies Act, 2013 by the Finance Act, 2017.

Global Multidimensional Poverty Index 2018 report: India has highest number of poor despite 27 crore moving out of poverty in 10 years: Report

In India, poverty reduction among children, the poorest states, Scheduled Tribes, and Muslims was fastest

Relevance IN – Prelims (about MPI 2018 report findings) + Mains (GS II social justice + welfare schemes for the vulnerable section of population)

What’s the NEWS• India still had 364 million poor in 2015-16, the largest for any country, although it is down from 635 million in

2005-06.

Page 106: JOIN THE DOTS! - Career Launcher · dots! A current affairs series for UPSC Examination’ series which will help you pick up relevant news items of the day from various national

Join the dots! – March 2019 Page: 100

Know! about the findings of the report• India has reduced its poverty rate drastically from 55% to 28% in 10 years, with 271 million people moving

out of poverty between 2005-06 and 2015-16, according to the Global MPI 2018 Report prepared by the United Nations Development Programme (UNDP) and the Oxford Poverty and Human Development Initiative.

• The report, covering 105 countries, dedicates a chapter to India because of this remarkable progress. However, India still had 364 million poor in 2015-16, the largest for any country, although it is down from 635 million in 2005-06.

• The report measures MPI, or multidimensional poverty index, which it says can be broken down to show “who is poor” and “how they are poor”.

• This factors in two measures, poverty rate as a percentage of the population, and intensity as the average share of deprivations that poor people experience. The product of these two is MPI. If someone is deprived in a third or more of 10 weighted indicators, the global index identifies them as “MPI poor”.

• In India, poverty reduction among children, the poorest states, Scheduled Tribes, and Muslims was fastest, the report says. Of the 364 million people who were MPI poor in 2015-16, 156 million (34.6%) were children.

• In 2005-06 there were 292 million poor children in India, so the latest figures represent a 47% decrease or 136 million fewer children growing up in multidimensional poverty.

• Although Muslims and STs reduced poverty the most over the 10 years, these two groups still had the highest rates of poverty. While 80% of ST members had been poor in 2005-06, 50% of them were still poor in 2015-16. And while 60% of Muslims had been poor in 2005-06, 31% of them were still poor in 2015-16.

• Bihar was the poorest state in 2015-16, with more than half its population in poverty. The four poorest states —Bihar, Jharkhand, Uttar Pradesh, and Madhya Pradesh — were still home to 196 million MPI poor people, which was over half of all the MPI poor people in India.

• Jharkhand had the greatest improvement, followed by Arunachal Pradesh, Bihar, Chhattisgarh, and Nagaland. At the other end, Kerala, one of the least poor regions in 2006, reduced its MPI by around 92%.

• Muslims from Assam, UP, Bihar different… Fight is against Bangladeshi Muslims, not Indian Muslims

• Worldwide, the report found, 1.3 billion people live in multidimensional poverty in the 105 developing countries it covered. This represents 23%, or nearly a quarter, of the population of these countries. These people are deprived in at least one-third of overlapping indicators in health, education, and living standards

• While the study found multidimensional poverty in all developing regions of the world, it was seen to be particularly acute in Sub-Saharan Africa and South Asia. These two regions account together for 83% (more than 1.1 billion) of all multidimensionally poor people in the world.

• Additionally, two-thirds of all multidimensionally poor people live in middle-income countries, with 889 million people in these countries experiencing deprivations in nutrition, schooling, and sanitation, just like those in low-income countries.

• The report describes the level of global child poverty as staggering, with children accounting for virtually half (49.9%) of the world’s poor. Worldwide, over 665 million children live in multidimensional poverty. In 35 countries, at least half of all children are MPI poor. In South Sudan and Niger, some 93% of all children are MPI poor.

Dhanush Howitzer guns

Page 107: JOIN THE DOTS! - Career Launcher · dots! A current affairs series for UPSC Examination’ series which will help you pick up relevant news items of the day from various national

Page: 101 Join the dots! – March 2019

Relevance IN – Prelims (about Dhanush artillery)

What’s the NEWS• Four indigenous Dhanush Howitzer guns were inducted to the Indian Army. In February 2018 Ordnance Factory

Board (OFB) had received the bulk production clearance for manufacturing of 114 Guns. Now, Gun Carriage Factory (GCF) Jabalpur is ready with 6 Dhanush guns for issuance to the Army.

Know! about Dhanush Artillery guns• Dhanush is a 155mm x 45mm calibre artillery gun. It has been developed by Ordnance Factory Board (OFB),

Kolkata and manufactured by Jabalpur-based Gun Carriage Factory (GCF).

• It is upgraded version of Swedish 155-mm Bofors howitzers, which India procured in the mid-1980s, based on its original designs.

• It has strike range of 38 kilometres which is about 10 km better than the Bofors guns.

• Its 81% components are indigenously manufactured and it will be scaled up to 90% by the end of 2019.

• Each of this gun costs about Rs 14.50 crore while each shell costs Rs. 1 lakh.

• It also has night firing capability in direct fire mode.

• Its armament system comprises a barrel, muzzle brake, breech mechanism and recoil mechanism to fire 155 mm calibre ammunitions.

• It has several significant advance features, including an all-electric drive, high mobility, quick deployability, auxiliary power mode, advanced communication system and automated command and control system etc.

Importance of Dhanush Artillery guns• This project would enhance the army’s firepower by having more guns of higher calibre. Induction of the gun is

part of the army’s replacing of calibre such as 105 mm and 120 mm with medium guns.

• This move includes the induction of M777 Ultra Light Howitzer and the K-9 Vajra self-propelled gun in November 2018.

• Apart from that, Dhanush artillery gun is equipped with navigation based sighting system, on board ballistic computation and an advanced day and night direct-firing system. The self-propulsion unit allows the gun to deploy itself in mountainous terrains with ease.

President Kovind honoured with Croatia’s highest civilian orderWhat’s the NEWS

• President Ram Nath Kovind was honoured with Croatia’s highest civilian order - The Grand Order of the King of Tomislav

• The President is in the European country as part of his eight-day three-nation visit to Croatia, Bolivia and Chile to further strengthen bilateral ties between India and the three countries.

Page 108: JOIN THE DOTS! - Career Launcher · dots! A current affairs series for UPSC Examination’ series which will help you pick up relevant news items of the day from various national

Join the dots! – March 2019 Page: 102

• Croatian state order is awarded to heads of state for their important contribution towards the development of state relations between Croatia and their respective countries.

The Grand Order of the King of Tomislav• The Grand Order of King Tomislav is the highest civilian order of Croatia.

• It is usually awarded to top foreign officials for their contribution to the improvement of Croatia’s good standing internationally as well as achievements in developing international relations between Croatia and their respective countries.

• It is awarded by the President of Croatia. It is named after King Tomislav of Croatia.

Coffee Board Activates Blockchain Based Marketplace in India

Relevance IN – Prelims (about blockchain based coffee e-marketplace) + Mains – GSIII (science and technology - development of new technologies)

What’s the NEWS• Coffee board launched blockchain based coffee e-marketplace through video conferencing

Know! the benefits of this marketplace• This pilot project will help integrate the farmers with markets in a transparent manner and lead to realisation

of fair price for the coffee producer.

• The blockchain will also reduce the number of layers between coffee growers and buyers and help farmers double their income.

Know! about Indian coffee• India is the only country in the world where entire coffee is grown under shade, handpicked and sun dried.

• It produces one of the best coffees in the world, produced by small coffee growers, tribal farmers adjacent to National Parks and Wild Life Sanctuaries in the Western and Eastern Ghats, which are two of the major bio-diversity hot spots in the world.

• Indian coffee is highly valued in the world market and sold as premium coffees. The share of farmers in the final returns from coffee is very meagre.

Know! about blockchain based market place• Blockchain based market place app for trading of Indian coffees is intended to bring in transparency in the trade

of Indian coffee, maintain the traceability of Indian coffee from bean to cup so as the consumer tastes real Indian coffee and the grower is paid fairly for his coffee produced.

• This initiative will help in creating a brand image for Indian Coffee through traceability in reducing growers dependency an intermediaries by having a direct access to buyers for a fair price for their produce, in finding right coffee suppliers for exporters and within the stipulated time to meet the growing demands and in building a better trust and long term relationship due to increased visibility towards the traceability and transparency of the produce.

• The stakeholder like coffee farmers, traders, coffee curers, exporters, rosters, importers and retailers register on the platform to make trade transactions.

• The coffee farmer registers credentials like place where coffee is grown, details of the crop, elevation, certificates if any and any relevant information.

• A block is created for each of the lot the farmer sells on the Blockchain. The credentials of the block/ lot will be stored on the Blockchain throughout its journey and are immutable.

GI Certification for five varieties of Indian coffee

Relevance IN – Prelims(about the types of coffee and about GI)

What’s the NEWS• The Department for Promotion of Industry and Internal Trade, Ministry of Commerce and Industry, Government

of India has recently awarded Geographical Indication (GI) to five varieties of Indian coffee.

Page 109: JOIN THE DOTS! - Career Launcher · dots! A current affairs series for UPSC Examination’ series which will help you pick up relevant news items of the day from various national

Page: 103 Join the dots! – March 2019

They are:

• Coorg Arabica coffee is grown specifically in the region of Kodagu district in Karnataka.

• Wayanaad Robusta coffee is grown specifically in the region of Wayanad district which is situated on the eastern portion of Kerala.

• Chikmagalur Arabica coffee is grown specifically in the region of Chikmagalur district and it is situated in the Deccan plateau, belongs to the Malnad region of Karnataka.

• Araku Valley Arabica coffee can be described as coffee from the hilly tracks of Visakhapatnam district of Andhra Pradesh and Odisha region at an elevation of 900-1100 Mt MSL.

• The coffee produce of Araku, by the tribals, follows an organic approach in which they emphasise management practices involving substantial use of organic manures, green manuring and organic pest management practices.

• Bababudangiris Arabica coffee is grown specifically in the birthplace of coffee in India and the region is situated in the central portion of Chikmagalur district.

• Selectively hand-picked and processed by natural fermentation, the cup exhibits full body, acidity, mild flavour and striking aroma with a note of chocolate. This coffee is also called high grown coffee which slowly ripens in the mild climate and thereby the bean acquires a special taste and aroma.

Know! more about coffee production in India• The Monsooned Malabar Robusta Coffee, a unique specialty coffee from India, was given GI certification earlier.

• In India, coffee is cultivated in about 4.54 lakh hectares by 3.66 lakh coffee farmers of which 98% are small farmers. Coffee cultivation is mainly done in the Southern States of India:

- Karnataka – 54%

- Kerala – 19%

- Tamil Nadu – 8%

- Coffee is also grown in non-traditional areas like Andhra Pradesh and Odisha (17.2%) and North East States (1.8%).

• India is the only country in the world where the entire coffee cultivation is grown under shade, hand-picked and sun dried.

• India produces some of the best coffee in the world, grown by tribal farmers in the Western and Eastern Ghats,which are the two major bio-diversity hotspots in the world. Indian coffeeis highly valued in the world market and sold as premium coffee in Europe.

• The recognition and protection that comes with GI certification will allow the coffee producers of India to invest in maintaining the specific qualities of the coffee grown in that particular region. It will also enhance the visibility of Indian coffee in the world and allow growers to get maximum price for their premium coffee.

Page 110: JOIN THE DOTS! - Career Launcher · dots! A current affairs series for UPSC Examination’ series which will help you pick up relevant news items of the day from various national

Join the dots! – March 2019 Page: 104

Prelims Practice Question1. Consider the following statements 1. The main objective of FAME is to encourage Faster adoption of Electric and hybrid vehicle by way of

offering upfront Incentive on purchase of Electric vehicles

2. The Indian Space Research Organisation is going to launch an electronic intelligence satellite ‘Emisat’ for the Defence Research and Development Organisation (DRDO), along with 28 third party satellites.

3. The National Science Day is celebrated every year on February 28 in the memory of Sir C.V. Raman. He was an Indian physicist and a Nobel laureate.

Find the correct statement from the option given below (a) 1 and 2 (b) 1 and 3 (c) 1 2 and 3 (d) None

2. Consider the following statements 1. Punjab National Bank has been ranked first among public sector banks in the implementation of ‘reforms

agenda according to the BCG-IBA report on EASE Reforms for Public Sector Banks 2. The Arun River is a trans-boundary river and is part of the Kosi or Sapt Koshi river system in Nepal which

originates in Tibet Autonomous Region of the People’s Republic of China 3. India has even launched the Global Housing Technology Challenge to fast-track the construction of

affordable housing and meet the target of constructing 1.2 crore houses by 2025.

Find the correct statement from the option given below (a) 1 and 2 (b) 2 and 3 (c) 1 2 and 3 (d) None

3. Consider the following statements 1. IFPRI is an international agricultural research center founded in India to improve the understanding of

national agricultural and food policies to promote the adoption of innovations in agricultural technology. 2. The mission of IFPRI is to provide research-based policy solutions that sustainably reduce poverty and end

hunger and malnutrition. 3. The five rhino range nations Indian, Bhutan, Nepal, Indonesia and Myanmar signed a declaration ‘The

Declaration on Asian Rhinos 2019’ for the conservation and protection of the species at the recently held Second Asian Rhino Range Countries

Find the correct statement from the options given below (a) 1 and 2 (b) 2 and 3 (c) 1 2 and 3 (d) None

4. Consider the following statements 1. The GSP programme of which India is the 2nd largest beneficiary allows duty-free entry of 1,784 products

from India into the US, benefitting exporters of textiles, engineering, gems and jewellery and chemical products.

2. Exercise Al Nagah, the inaugural edition of bilateral joint exercise between India and Oman is scheduled to be held from 12 to 25 March 2019 at Jabel Al Akhdar Mountains in Oman.

Find the correct statement from the option given below (a) 1 and 2 (b) 2 only (c) 1 only (d) None

5. Consider the following statements 1. The Ministry of textiles Minister in association with the Ministry of culture and NITI Aayog launched

three projects of NIFT - VisionNXT – Trend Forecasting Initiative, Indian Textiles and Craft Repository and Design Innovation and Incubation.

2. First ladies, 100 women achiever and Web Wonder Women are some of the campaigns of the Ministry of Health and Family welfare

3. The Pradhan MantriBhartiyaJ anaushadhi Pariyojana” (PMBJP) has been implementing by the Bureau of Pharma PSUs of India (BPPI), under the administrative control of Department of Pharmaceuticals, Ministry of Chemicals & Fertilizers.

Page 111: JOIN THE DOTS! - Career Launcher · dots! A current affairs series for UPSC Examination’ series which will help you pick up relevant news items of the day from various national

Page: 105 Join the dots! – March 2019

Find the correct statement from the options given below: (a) 1 and 3 (b) 2 and 3 (c) 1 only (d) 3 only

6. Consider the following statements 1. National space agency ISRO and its French counterpart CNES on Wednesday sealed an agreement to set

up a joint military surveillance system in the country. 2. National Council of Science Museums (NCSM), a premiere institution in the field of science communication,

is an autonomous organization under the Ministry of Science and technology, Government of India. 3. Atal innovation Mission’s objectives are to create and promote an ecosystem of innovation and

entrepreneurship across the country at school, university, research institutions, MSME and industry levels.

Find the correct statement from the option given below (a) 1 and 2 (b) 3 only (c) 2 only (d) All

7. Consider the following statements 1. According to the India State of Forest Report (SFR) 2017, which was made public in February 2018 the

total forest cover as percentage of geographical area is 21.54% 2. India posted a marginal 0.21% rise in the area under forest between 2015 and 2017, according to the India

State of Forest Report (SFR) 2017, which was made public in February 2018. 3. The PMUY was launched by the Prime Minister Narendra Modi on 1st May 2016 with an initial target of

seven crore LPG connections, which was later revised upward to eight crores connections. 4. Pradhan Mantri Ujjwala Yojana Pradhan Mantri Ujjwala Yojana (PMUY) - Under this scheme, LPG

connections will be provided to all rural families with a support of Rs.1600 per connection.

Find the correct statement from the option given below (a) 1 and 2 (b) 1 2 and 4 (c) 1 and 4 (d) All

8. Consider the following statements 1. The indigenously developed Guided Pinaka by DRDO in collaboration with ISRO will significantly boost

the capability of the artillery to make precision hits. 2. Polio also known as poliomyelitis is a highly contagious bacterial disease caused due to the attacks the

nervous system and children younger than 5 years old are more likely to contract the virus than any other group

3. Argentina, Brazil, Paraguay Uruguay and Venezuela are the full time member of MERCOSUR

Find the correct statement from the option given below (a) 1 and 3 (b) All (c) None (d) 2 and 3

9. Consider the following statements 1. The Registrar of Geographical Indications, Government of India, accorded a GI tag, ‘Sirsi Supari’, to

arecanut grown in Sirsi, Siddpaur and Yellapur taluks of Uttara Kannada district in Tamil Nadu. 2. SIPRI is an independent international institute dedicated to research into conflict, armaments, arms

control and disarmament.

Find the correct statements from the option given below (a) 1 and 2 (b) 1 only (c) 2 only (d) None

10. Consider the following statements 1. Index of Industrial Production Index of Industrial Production (IIP) is a composite indicator that measures

the changes in the volume of production of a basket of industrial products by using the base year of 2015-16

2. The inaugural Joint Military exercise AL - Nagah between India and Oman military commenced at Jabel Regiment, Oman

Find the correct statement from the option given below (a) 1 only (b) 2 only (c) None (d) All

Page 112: JOIN THE DOTS! - Career Launcher · dots! A current affairs series for UPSC Examination’ series which will help you pick up relevant news items of the day from various national

Join the dots! – March 2019 Page: 106

11. Consider the following statements

1. The Defence Research and Development Organisation (DRDO) successfully test fired indigenously developed, low weight, fire and forget Man Portable Anti-Tank Guided Missile (MPATGM) for the first time in the ranges of Rajasthan desert.

2. West Nile Virus (WNV) is a mosquito-borne disease, mostly reported in the dense regions of African continent .

3. The 8th edition of exercise Sampriti which was started in 2009 is a joint Indo-Bangladesh military exercise which witnessed participation of a company group of 9th Battalion the Rajputana Rifles from the Indian Army and the Company of 36 East Bengal Battalion, Bangladesh Army concluded at Tangail, Bangladesh

Find the correct statement from the option given below

(a) 2 and 3 (b) 3 only (c) 1 and 3 (d) 2 only

12. Consider the following statements

1. A 10-day long ASEAN -India Joint Field Training Exercise (AFINDEX-19) between the Indian Army and 10 ASEAN nations will be conducted in Pune.

2. AFINDEX-19 aims to train the participating contingents in Humanitarian Mine Assistance (HMA) and Peace Keeping Operations (PKO) under the United Nations Charter through practical and comprehensive discussions and tactical exercises

3. The World Economic Forum is the International Organization for Public-Private Cooperation and it was established in 1971 as a not-for-profit foundation and is headquartered in Geneva, Switzerland.

Find the correct statements from the option given below

(a) 2 and 3 (b) 1 and 3 (c) 1 and 2 (d) All

13. Consider the following statement about this PSLV C- 45 launch

1. This PSLV will be ISRO’s first three-orbit mission that will launch satellites in three different orbits.

2. PSLV will launch the advanced electronic intelligence satellite EMISAT for DRDO along with 28 commercial satellites.

3. This PSLV-C45 is the first three-orbit mission of ISRO and the first to use solar panel in rocket fourth stage (PS4).

Find the correct statement from the option given below

(a) 1 and 2 (b) 1 and 3 (c) 2 and 3 (d) All

14. Consider the following and find out the correctly matched option

1. Coorg Arabica Coffee - Karnataka

2. Wayanad Robusta Coffee - Kerala.

3. Chikmagalur Arabica Coffee - Karnataka

4. Bababudangiris Arabica Coffee - Karnataka

Find the correct one

(a) 1 and 2 (b) 3 and 4 (c) All (d) None

15. Consider the following statements

1. The Reserve Bank of India (RBI) has announced that SBI, ICICI and HDFC Banks would continue as Domestic Systemically Important Banks (D-SIBs) for 2018.

2. Cyclone Idai made landfall on Mozambique and later moved in a westerly direction is the deadliest Tropical cyclone originated from a tropical depression that formed off the eastern coast of Mozambique and made landfall in Mozambique.

Find the correct statement from the option given below

(a) 1 only (b) 2 only (c) Both (d) None

Page 113: JOIN THE DOTS! - Career Launcher · dots! A current affairs series for UPSC Examination’ series which will help you pick up relevant news items of the day from various national

Page: 107 Join the dots! – March 2019

16. Consider the following statements 1. India was ranked at 140 on the latest World Happiness Report that gauged 156 countries, a rise of seven

spots from the last edition of the survey. 2. Finland topped the list for the fifth consecutive year, followed by Denmark and Norway 3. The World Happiness Report is an annual publication of the World Economic Forum 4. Exercise MITRA SHAKTI is an annual military diplomacy and interaction between armies of India &

Maldives.

Find the correct statement from the option given below (a) 1 and 3 (b) 3 and 4 (c) 1, 2 and 4 (d) None

17. Consider the following statement 1. Langkawi International Maritime Aero Expo (LIMA-2019) is planned in Langkawi, Singapore from 26

March 2019 to 30 March 2019. 2. Indian Air Force is participating in the Maritime Aero Expo for the third time, and in this edition India

will showcase its indigenously developed LCA fighter aircraft. 3. World Down Syndrome Day 21 March is observed as World Syndrome Day to increase global awareness

about the syndrome by the United Nations since 2012 4. March 21st was chosen to signify uniqueness of the triplication of the 21st chromosome which causes Down

syndrome

Find the correct statement from the option given below (a) 1 and 2 (b) 2 and 3 (c) 3 and 4 (d) 1 and 4

18. Consider the following statement about Lokpal 1. The chairperson of the Lokpal must be a former chief justice of India or a Supreme Court judge and a

member should be a former Supreme Court judge or a chief justice of a High Court. 2. The chairperson and members will hold office for a term of five years or till they attain 70 years of age. 3. The Lokpal act provides for a chairperson and a maximum of nine members in the Lokpal panel and of

these, four need to be judicial members.

Find the correct statement from the option given below (a) 1 and 2 (b) 2 and 3 (c) All (d) None

19. Consider the following statements 1. The World Food Programme (WFP) is the food-assistance branch of the United Nations and the world’s

largest humanitarian organization addressing hunger and promoting food security. 2. World Meteorological Day is observed on March 23rd every year to commemorate the coming into force of

the Convention establishing the World Meteorological Organization on 23 March 1950. 3. Indian Space Research Organisation has launched a special programme for college pass out students called

“Young Scientist Programme” “YUva VIgyani KAryakram” from this year, in tune with the Government’s vision “Jai Vigyan, Jai Anusandhan”.

Consider the following statements and find the correct one from the options given below (a) 1 and 3 (b) 1 and 2 (c) 2 and 3 (d) All

20. Consider the following statements 1. The PSLV-C45/Emisat mission OFthe Indian Space Research Organisation (ISRO) will be ISRO’s first

attempt at placing payloads in three different orbits. 2. India has moved up two places to rank 76th on a global energy transition index, which has ranked 115

economies on how well they are able to balance energy security and access with environmental sustainability and affordability.

3. India has made significant strides to improve energy access in recent years, and currently scores well in the area of regulation and political commitment towards energy transition

Page 114: JOIN THE DOTS! - Career Launcher · dots! A current affairs series for UPSC Examination’ series which will help you pick up relevant news items of the day from various national

Join the dots! – March 2019 Page: 108

Find the correct statement from the option given below (a) All (b) 1 and 2 (c) 2 and 3 (d) None

21. Consider the following statements about Mission Shakti 1. The successful execution of Mission Shakti makes India the fifth country to successfully test an Anti-

Satellite weapon system, after the US, Russia China and Japan. 2. A-SATs are space weapons that are designed to destroy satellites for strategic military purpose. 3. A-SAT weapons are basically short-range missiles with a kinetic kill vehicle attached to them. They can be

locked on to the target, intercept, engage and destroy in one swoop.

Find the correct statement from the option given below: (a) 2 only (b) 1 and 3 (c) 2 and 3 (d) All

22. Consider the following statement 1. Exercise MITRA SHAKTI is a 14 days joint training exercise of the Indian Army and the Nepal Army 2. Exercise MITRA SHAKTI is the sixth edition of the joint exercise between the two nations and its been

conducting from 26 March to 08 April 19.

Find the correct statement from the option given below (a) 1 only (b) 2 only (c) None (d) all

23. Consider the following statements about electoral bonds 1. An electoral bond is designed to be a bearer instrument like a Promissory Note — in effect, it will be

similar to a bank note that is payable to the bearer on demand and free of interest 2. The bonds will be issued in multiples of ₹1,000, ₹10,000, ₹1 lakh, ₹10 lakh and ₹1 crore and will be available

at specified branches of Public sector banks. 3. Every party that is registered under section 29A of the Representation of the Peoples Act, 1951 (43 of 1951)

and has secured at least four per cent of the votes polled in the most recent Lok Sabha or State election will be allotted a verified account by the Election Commission of India.

Find the correct statements from the option given below) (a) 1 only (b) 2 and 3 (c) 1 and 3 (d All

24. Consider the following statements regarding Mission Shakti 1. India successfully conducted an Anti-Satellite (ASAT) missile test, named Mission Shakti, becoming the

fourth country in the world to demonstrate the capability to shoot down satellites in orbit. 2. The satellite downed by the ASAT missile was Microsat-R, an imaging satellite which was launched into

orbit on January 24, 2019 using a Polar Satellite Launch Vehicle (PSLV) 3. India has built the broad capabilities and building blocks to develop ASAT missiles for some time as part

of its Ballistic Missile Defence (BMD) programme. 4. Anti-satellite weapons provide the capability to shoot down enemy satellites in orbit thereby disrupting

critical communications and surveillance capabilities.

Find the correct statement from the option given below (a) All (b) 1 and 4 (c) 2 and 4 (d) None

25. Consider the following statements regarding Dhanush artillery guns 1. Board (OFB), Kolkata and manufactured by Jabalpur-based Gun Carriage Factory (GCF) along with the

assistance of France 2. It is upgraded version of Swedish 155-mm Bofors howitzers, which India procured in the mid-1980s, based

on its original designs. 3. It is 100% indigenously developed by the Ordnance Factory Board (OFB), Kolkata

Find the correct options from the options given below (a) 1 and 2 (b) 2 only (c) 1 and 3 (d) 2 only

Page 115: JOIN THE DOTS! - Career Launcher · dots! A current affairs series for UPSC Examination’ series which will help you pick up relevant news items of the day from various national

Page: 109 Join the dots! – March 2019

26. Consider the following statements 1. The coffee board launched blockchain based coffee e-marketplace through video conferencing 2. This project will help integrate the farmers with markets in a transparent manner and lead to realisation

of fair price for the coffee producer. 3. The blockchain will reduce the number of layers between coffee growers and buyers and help farmers

double their income. 4. India is the only country in the world where entire coffee is grown under shade, handpicked and sun dried.

Find the correct statements from the option given below (a) all (b) None (c) C and D (d) A and D

Answer Keys1 c 2 a 3 a 4 d 5 d 6 b 7 a 8 c 9 c 10 c

11 b 12 a 13 d 14 c 15 c 16 d 17 c 18 a 19 b 20 a

21 a 22 b 23 a 24 a 25 d 26 a


Recommended